Review Questions

Choose the single BEST option. 3. A 45-year-old man was referred for surgical evalua- tion after a 4.6 cm ascending aortic was 1. In the Arterial Trial (ART), found. demonstrated no sig- patients were randomly assigned to receive either nificant coronary disease. He has a murmur bilateral internal thoracic artery grafts or a standard heard best in the right second interspace and echo single left internal thoracic artery graft during coro- showed a bicuspid with a valve area of nary artery bypass grafting. The intention-to-treat 0.9 cm2. With respect to the patient’s care, which of analysis at 10 years follow-up reported the following statements is true? A. significantly improved all-cause mortality in the A. Aortic pathology is anticipated to show increased bilateral internal thoracic artery grafts group. fibrillin 1 content and decreased matrix metallopro- B. significant between-group difference in the rate of teinase 2 activity. repeat revascularization. B. Aortic alone is the appropriate C. no significant differences between the two groups recommendation for this patient. for the composite outcomes of death from any cause, C. A should be performed if the aor- myocardial infarction, or stroke. tic root is dilated. D. significant between-group differences in cardiac D. Expectant followup of this patient is expected to mortality. show an aortic dilation rate equivalent to patients E. significant between-group differences in stroke rate. with tricuspid aortic valves. 2. A 60-year-old otherwise healthy man underwent suc- E. Reduction aortoplasty and supracoronary tube graft- cessful for severe aortic ste- ing (in addition to AVR) yield comparable long-term nosis with mild insufficiency. Preoperative angiogram results. showed minimal coronary disease. During the opera- 4. One week ago a 40-year-old man patient on chronic tion initial arrest was with cold antegrade cardiople- underwent AVR with a mechanical gia though an aortic cannula, and maintenance was valve for calcific aortic . Preoperative ven- with intermittent cold retrograde . A left tricular function was normal. A routine followup ventricular vent was used. Several hours postop as he transthoracic echo identified a paravalvular leak in was waking up there were multiple witnessed grand the non-coronary sinus with a vena contracta of mal seizures and hemodynamic evidence of right 0.6 cm. Discharge plans are complete and the patient ventricular dysfunction with several episodes of non-­ is asymptomatic. cultures are negative, there is sustained ventricular tachycardia. no evidence of hemolysis and the ejection fraction is The most appropriate management is 45%. Which of the following is the best treatment A. Hundred percent oxygen, induced hypertension and option? hyperdynamic condition, and avoidance of A. Observation until spontaneous resolution or devel- hyperthermia. opment of symptoms. B. Emergent cardiac catheterization and possible per- B. Percutaneous catheter intervention to place an cutaneous interventions. occlude device. C. Hyperbaric therapy with 50% oxygen and 50% C. Reoperation for valve re-replacement using a bio- nitrogen at a pressure of 3 atm. prosthetic valve. D. Immediate reoperation to place multiple coronary D. Reoperation for valve re-replacement using another artery grafts with saphenous segments. mechanical valve. E. IV anticonvulsant therapy, emergent CT scan of the E. Reoperation to close the defect with pledgeted chest and neurosurgical consultation. sutures.

© Springer Nature Switzerland AG 2020 1011 S. G. Raja (ed.), Cardiac , https://doi.org/10.1007/978-3-030-24174-2 1012 Review Questions

5. A 78-year-old man with diabetes, high cholesterol, operative transthoracic shows and well controlled hypertension presented with exer- severe ischemic regurgitation with LVEF tional chest pain and shortness of breath. He has 35% and moderate-to-severe tricuspid valve regurgi- never had a stroke or TIA, he lives independently and tation. Pre-bypass TEE now shows mild mitral regur- remains active. Non-specific ECG changes and crite- gitation and mild tricuspid valve regurgitation. ria for left ventricular hypertrophy were present on Which of the following statements is correct? ECG. Cardiac catheterization demonstrated severe A. CABG and will lower PA pres- multivessel disease with an ejection fraction of 50%, sure, so concomitant tricuspid annuloplasty is not and a carotid duplex showed a 50–70% and 60–75% indicated. stenoses of the right and left internal carotid , B. A complete tricuspid ring annuloplasty is preferred respectively. At the time of surgery the initial TEE to avoid future dilatation of the posterior annulus. demonstrates an aortic valve area of 1.15 cm2 and C. A flexible mitral annuloplasty ring is best to pre- heavy calcification of the annulus and leaflets. serve mitral annular mobility and ventricular Which of the following is the best operation for contractility. this patient? D. The patient’s poor left ventricular function suggests A. Off-pump multivessel bypass grafting alone and sur- that concomitant tricuspid valve repair is prudent. veillance of the carotid stenoses and E. The present TEE findings indicate that CABG alone B. Off-pump multivessel bypass grafting with antici- is adequate—mitral annuloplasty is not required. pated percutaneous transcatheter aortic valve 8. A 63-year-old woman presented to the emergency replacement in the near future. department complaining of shortness of breath and C. On-pump multivessel bypass grafting and surveil- recurrent chest pain. Five days before she had acute lance of the carotid stenoses and aortic stenosis. chest pain that spontaneously resolved. Her past med- D. On-pump multivessel bypass grafting and replace- ical history was notable only for hypertension. Physical ment of the aortic valve with a bioprosthetic valve. examination confirms dyspnoea at rest. Blood pres- E. On-pump multivessel bypass grafting, bioprosthetic sure is 80/50, rate is 100, and her skin is cool and AVR and left carotid . clammy. An ECG shows sinus tachycardia with ST 6. Fifteen years ago a 75-year-old woman underwent elevations in the anterior leads. This woman’s echo- coronary artery bypass grafting with saphenous vein cardiogram shows a large anterior effusion with dia- grafts to the LAD, obtuse marginal, and posterior stolic RV collapse and impaired LV function with descending arteries. She developed unstable angina posterior akinesis suggesting postinfarction free wall and a cath revealed a 90% left main stenosis and ventricular rupture. This patient is best managed by occlusions of the native right coronary and the OM A. , bare metal stenting of the LAD, and and PDA grafts. There was a 90% stenosis of the delayed coronary artery bypass grafting. saphenous vein graft to the LAD. At reoperation, a B. Beta blockers, IV fluids, and septal myomectomy LIMA was placed to the LAD and the old saphenous when stable. vein graft was ligated. New saphenous vein grafts C. Emergent circumflex CABG and myocardial rein- were placed to the obtuse marginal and PDA. The forcement with sutures and felt pledgets. patient cannot be weaned from bypass because of left D. , intra-aortic balloon placement, heart failure and anterior akinesis. TEE shows no and inotropic support. valve pathology, and ultrasonic flow measurements E. Prompt surgical intervention with a glued epicardial of the LAD, OM and PDA grafts are 26, 50, and patch. 35 mL/min respectively. Which of the following is the 9. A 56-year-old man underwent a 3-vessel coronary most appropriate action to take? artery bypass following a recent myocardial infarction. A. Go back on bypass and re-do the LIMA-LAD Grafts included LIMA to LAD, radial artery graft to anastomosis. OM1, and saphenous vein graft to the PDA. This was B. Place a left ventricular assist device. done on with a warm beating C. Remove the ligature from the saphenous vein graft heart. Thirty-six hours postoperatively the patient is to the LAD. noted to be in severe respiratory distress with increased D. Place an intra-aortic balloon pump and add inotropic work of breathing. He is on a face mask with 100% support. inspired oxygen. pressure is 55/35, E. Return to bypass and place a saphenous vein graft to cardiac index is 1.7, and mean arterial pressure is the LAD. 50 mmHg. Physical exam reveals significant rales and 7. A 68-year-old man is about to undergo coronary a systolic murmur. His chest radiograph shows severe bypass grafting and mitral valve surgery. Coronary pulmonary edema. Which of the following is the next reveals severe 3-vessel disease and pre- step in the management of this patient? Review Questions 1013

A. Begin an epinephrine (adrenaline) infusion ular function with anteroapical wall akinesis. A saphe- B. Do a stat nous vein graft was placed to the LAD distal to the C. Initiate a furosemide infusion LIMA anastomosis to no avail. A short-term LVAD D. Obtain a stat echocardiogram was placed with left atrial inflow via the right superior E. Start a phenylephrine infusion pulmonary vein and an outflow graft anastomosed to 10. Three months ago a 56-year-old man suffered an the ascending . The next day his anterior wall myocardial infarction. An echocardio- included: LVAD flow 4.0 L/min, PA 33/25 (mean 29), gram shows an apical aneurysm. Which of the fol- RA 27, heart rate 95. His urine output was 20 mL/h on lowing statements about his condition is true? a furosemide drip at 20 mg/h. Serum creatinine was A. All lesions of this type that exceed 3 cm should be 2.5 mg/dL (rising), and he remained coagulopathic repaired regardless of symptoms. despite massive blood component transfusion. B. Five-year survival for asymptomatic patients with Reoperation is now considered necessary and the this condition is 90% with medical therapy. short-­term device is to be replaced with a long-term C. The leading cause of death for medically managed LVAD as a bridge to transplant. The most life-threat- patients with this condition is heart failure. ening complication anticipated for this patient is D. The majority of these lesions are posterior and at the A. Bleeding from the left ventricular apical LVAD cuff base of the heart. suture line. E. Surgical repair of the condition should be attempted B. Bleeding from the outflow graft anastomosis on the before recommending transplantation. . 11. A 75-year-old man had a history of congestive heart C. Hepatic failure. failure, aortic regurgitation, D. Renal failure. with myocardial infarction, and severe ischemic mitral E. Right ventricular failure. regurgitation. Preoperative echo details included an 13. Regarding using an LVAD as bridge to transplanta- ejection fraction of 30%, normal mitral leaflets, and an tion, which of the following statements is correct? area of akinesis in the lateral wall. CABG, AVR and A. Early RVAD placement is necessary in approxi- mitral repair were done. Bypass and crossclamp times mately 10% of LVAD-bridged patients. were long, and he was weaned on an intra-aortic bal- B. Excessive postoperative bleeding occurs in less than loon pump, milrinone at 0.7 μg/(kg min) and epineph- 20% of patients. rine at 0.1 μg/(kg min). Still in the operating room now, C. The incidence of postoperative stroke and other his BP is 95/50, PA = 60/20, CVP = 17, pH = 7.25, thrombotic complications is about 5%.

CI = 1.5, SVO2 = 40%, and he is making little to no D. Infection is encountered equally with pulsatile and urine. All grafts appear to be functioning based on continuous flow devices. Doppler interrogation and TEE shows normal pros- E. One-year post transplant survival is worse for thesis function, global LV hypokinesis, lateral wall aki- bridged compared to non-bridged patients. nesis (unchanged from preop) and no significant mitral 14. A 26-year-old man was readmitted with heart failure regurgitation. The patient has been closely monitored and fluid overload. Consequences of his dilated cardio- for the last 30 min and there has been no change. myopathy prompted several admissions over the last 6 The most appropriate next step is to months despite optimal medical management and A. Add a vasoconstrictor such as noradrenaline compliance with medications. Recently he had two epi- (norepinephrine). sodes of non-sustained ventricular tachyarrhythmia. B. Cannulate and start venoarterial ECMO and transfer He weighs 250 lb and has blood type O. There is no to the ITU. significant comorbidity. The most recent cath data is: C. Give bicarbonate and inhaled nitric oxide. BP 88/70, CVP 6, PA 56/26 (mean 36), PCWP 19, CO D. Perform additional coronary bypass grafts. 3.8, CI 1.6, transpulmonary gradient (TPG) 17, pulmo- E. Place a temporary ventricular assist device. nary vascular resistance (PVR) 5.0 Woods units. He 12. A 64-year-old man (BSA 1.9 m2) presented with severe was then challenged with nitroprusside 80 μg/kg/min chest pain and cardiogenic shock. Catheterization and milrinone 0.37 μg, and hemodynamics were: BP revealed severe triple vessel disease. An intra-­aortic 86/58, CVP 8, PA 67/28 (mean 43), PCWP 20, CO 4.1, balloon pump did not stabilize his hemodynamics so CI 1.71, TPG 18, and PVR 5.6. The appropriate man- emergent on-pump CABG x 4 was done (LIMA to agement recommendation is LAD, SVG to PDA, and sequential SVG to OM1 and A. Begin chronic IV milrinone and reassess PVR after OM2). He was weaned from cardiopulmonary bypass 3 months. with four drugs and the balloon pump, but hemody- B. Continuous outpatient management and list for namics deteriorated following chest closure and transplantation (status 2). bypass was reinstated. TEE revealed poor left ventric- C. Emergency cardiac transplantation. 1014 Review Questions

D. Prepare for elective LVAD placement with apical 17. A 59-year-old man experienced increasing angina cannulation. during routine activities. He had two episodes during E. Start IV milrinone, place an ICD and list for trans- hemodialysis last week and was admitted for further plantation (status 1B). workup. A high resolution CT slice is shown. 15. A 68-year-old man had aortic valve replacement with a bileaflet mechanical valve and CABG to the poste- rior descending artery 12 years ago. He now has severe heart failure but is not a transplant candidate due to chronic lymphocytic leukemia that has been in remission. Which of the following statements is true? A. Another cardiac catheter is not necessary as the left will be defunctionalized. B. Destination LVAD is possible, but the mechanical valve must be changed to a bioprosthesis. C. Echo demonstration of severe right ventricular fail- ure mandates BiVAD destination therapy. D. This patient is not a candidate for destination therapy due to the mechanical prosthesis. E. The patient’s aortic valve should be sewn shut or patched closed if an LVAD is placed. 16. A 71-year-old man is undergoing a 4-vessel CABG and mitral valve repair the day after failed percuta- neous angioplasty of the right coronary artery for an The findings are significant for acute myocardial infarction. He cannot be separated A. accelerated coronary atherosclerosis. from bypass despite two high-dose inotropes and an B. associated valvular anomalies. intra-aortic balloon pump. Of the following choices C. exercise-induced coronary spasm. the best strategy to deal with this includes D. increased risk of sudden death. A. BiVAD support with short-term external ventricles, E. right ventricular failure. cannulating the LV apex for LVAD inflow. 18. A 44-year-old man presented with increasing dys- B. BiVAD support with short-term external ventricles, pnoea. Echocardiography revealed severe aortic cannulating the LA for LVAD inflow. valve regurgitation associated with an aortic C. Percutaneous LA and femoral cannulation for left aneurysm. Parts of a CT study are shown. The ventricular support with an external device. patient has no signs of a heritable connective tis- D. Placing a long-term internal mechanical ventricle as sue disorder. During surgery the aortic valve is destination therapy. trileaflet, the left coronary leaflet is prolapsed, E. Withdrawal of support because of patient age and and each of the cusps has fenestrations near the overwhelming mortality of this condition. commissures.

Review Questions 1015

The best operation for this patient is B. continue treatment with beta blockers and repeat A. aortic valve replacement with a mechanical prosthe- echocardiogram in 6–12 months. sis and separate replacement of the tubular portion of C. schedule elective graft replacement of the ascending the ascending aorta with a prosthetic graft. aorta. B. replacement of the aortic root with a composite valve D. schedule elective valve-sparing root replacement. graft and reattachment of with a E. stop the beta blockers, begin oral losartan, and repeat 10-mm prosthetic graft (Cabrol procedure). the echocardiogram in 6–12 months. C. replacement of the aortic root with a cryopreserved 20. The mutation causing most fre- homograft and direct reattachment of the coronary quently involves the gene for buttons to the homograft. A. Elastin D. replacement with a composite mechanical valve B. Fibrillin-1 graft and direct reattachment of the coronary buttons C. Transforming growth-β to the graft (modified Bentall procedure). D. Type I collagen E. valve-sparing aortic root replacement with direct E. Type III collagen reattachment of coronary artery buttons to the pros- 21. A 30-year-old man with no symptoms requested thetic graft (David reimplantation procedure). evaluation of his thoracic aorta. His father died 19. An asymptomatic 22-year-old man with Marfan syn- suddenly at age 41 from an . On drome was referred for evaluation of an ascending physical examination his height and wing span are measuring 5.0 cm in diameter. He has 195 cm and 203 cm, respectively. The arch of his been compliant with his β-blocker regimen but he is palate is remarkably high and striae are noted on concerned because his sister had an aortic dissection his abdominal flanks. His fingers are long and at age 26. An echocardiogram shows normal aortic spindly, and his knee joints are hyperextensible valve function and normal ventricular function. bilaterally. He suffered a dislocated lens in his The most appropriate management is to right eye at age 19. Parts of a CT angiogram study A. continue treatment with beta blockers and repeat the are shown with maximum aortic root dimension of echocardiogram if symptoms develop. 50.0 mm.

Which of the following is the most important man- 22. A 60-year-old woman presented with a 4 month his- agement strategy for this patient? tory of increasing back pain. An echocardiogram A. Aortic root replacement with a mechanical valved revealed normal valve function and a left ventricu- conduit. lar ejection fraction of 0.45. Two CT angiogram B. Losartan therapy and a follow-up CT angiogram in images are shown. There was no evidence of coro- 12 months. nary artery disease on her electrocardiogram or C. A modified remodeling (Yacoub) procedure with myocardial scan. replacement of the non-coronary sinus. D. Valve-sparing aortic root and total aortic arch replacement. E. A Wheat procedure. 1016 Review Questions

The most appropriate plan of treatment is E. Graft replacement of the thoracoabdominal aorta A. Graft replacement of the ascending aorta and trans- during an initial operation, followed by graft verse aortic arch during an initial operation, fol- replacement of the ascending aorta as a subse- lowed by graft replacement of the thoracoabdominal quent operation. aorta as a subsequent operation. 23. A 68-year-old woman has no pain, but she does B. Graft replacement of the ascending aorta during an complain of dyspnea when climbing more than one initial operation, followed by graft replacement of the flight of stairs. She has lower extremity edema on thoracoabdominal aorta as a subsequent operation. examination. An echocardiogram reveals a bicus- C. Graft replacement of the ascending aorta, transverse pid aortic valve with severe regurgitation, a dilated aortic arch, and thoracoabdominal aorta during a ascending aorta, and a left ventricular ejection single operation. fraction of 35%. The diameter at the sinotubular D. Graft replacement of the thoracoabdominal aorta junction is 2.8 cm. Cardiac catheterization con- during an initial operation, followed by graft replace- firms these findings and reveals no significant -cor ment of the ascending aorta and transverse aortic onary artery stenoses. A CT angiogram of the arch as a subsequent operation. aorta is shown.

Review Questions 1017

The best plan to stage surgical treatment in this 24. A 58-year-old woman reported increasing patient is retrosternal pain and dyspnoea over the past 3 A. AVR and graft replacement of the ascending aorta months. Images from her CT reconstruction are and transverse aortic arch with elephant trunk during shown. Other studies revealed moderate-to-severe the first operation, then subsequent thoracoabdomi- aortic valve regurgitation but minimal calcifica- nal repair. tion of the aortic valve and aorta. AVR and graft B. graft replacement of the thoracoabdominal first, replacement of the ascending aorta and lesser cur- then valve-sparing aortic root replacement and vature of the aortic arch (beveled hemi-arch graft replacement of the ascending aorta and repair) is now underway. Cannulation was via the transverse aortic arch later. right axillary artery for arterial inflow, a single C. initial AVR, followed by graft replacement of the right atriocaval cannula for venous drainage, a left thoracoabdominal aorta during a second operation. ventricular vent through the right superior pulmo- D. thoracoabdominal aortic graft replacement first, fol- nary vein, and a retrograde cardioplegia cannula. lowed by AVR and graft replacement of the ascending Systemic cooling was initiated immediately and aorta and transverse arch during the second operation. ventricular fibrillation occurred at a venous return E. valve-sparing aortic root replacement and graft temperature of 31 °C. Soon afterward the left ven- replacement of the ascending aorta and transverse tricle became markedly distended and the perfu- aortic arch initially, then graft replacement of the sionist reported copious return from the left thoracoabdominal aorta later. ventricular vent.

The most appropriate way to manage this situa- C. Place a clamp across the aneurysmal ascending aorta tion is to and deliver antegrade cardioplegia through a needle A. Deliver continuous retrograde cardioplegia until the placed in the ascending aorta. patient has been cooled enough that circulatory D. Place a clamp across the aneurysmal ascending aorta, arrest can be initiated. open the aneurysm, and deliver antegrade cardiople- B. Fill the with iced saline and continue gia through coronary ostial cannulas. rapid cooling until the patient has been cooled E. Place an additional ventricular vent directly through enough that circulatory arrest can be initiated. the left ventricular apex. 1018 Review Questions

25. A 62-year-old with a benign past medical history sis, and moderate regurgitation. At operation, TEE developed an upper respiratory illness. Chest radio- also revealed substantial atheroma within the ascend- graph revealed a prominent upper mediastinum, ing and descending thoracic aorta. Epi-aortic ultra- prompting a subsequent chest CT (see figure). On sound showed atherosclerotic thickening exceeding current examination he is 73 in. tall, weighs 195 lb, 5 mm throughout the distal two-thirds of the ascend- vital signs are normal, and his physical exam is ing aorta and the proximal portion of the aortic arch. unremarkable. The best strategy for arterial cannulation and car- diopulmonary bypass (CPB) for this patient is A. Arch cannulation, CPB with moderate hypothermia, and cardioplegic arrest while the distal ascending aorta is clamped. B. Femoral cannulation, CPB and deep hypothermic circulatory arrest without aortic clamping. C. Femoral cannulation, CPB with moderate hypother- mia, and cardioplegic arrest while the distal ascend- ing aorta is clamped. D. Right axillary cannulation, CPB and deep hypother- mic circulatory arrest without aortic clamping. E. Right axillary cannulation, CPB with moderate hypothermia, and cardioplegic arrest while the distal ascending aorta is clamped. 27. A 38-year-old hypertensive man had a type A aortic Which of the following is the most appropriate dissection 5 years ago and underwent proximal aor- next step in his management? tic repair with ascending aortic and hemiarch A. Aortic root and ascending aortic replacement with replacement. A residual type B aortic dissection was hemiarch reconstruction. subsequently followed. He was lost to follow-up for B. Aortic root replacement. 18 months, but returned with upper back pain C. Ascending aortic replacement. between his spine and left scapula. Despite his anti- D. CT angiogram in 12 months. hypertensive regimen of metoprolol, clonidine, amlo- E. Oral losartan. dipine, doxazosin, and minoxidil, he presents with a 26. A 67-year-old woman was referred for treatment of blood pressure of 180/110 mmHg. He is otherwise symptomatic aortic valve disease. Transthoracic echo healthy and active. Four CT angiogram frames and a showed severe calcification of the aortic valve with reconstruction image are shown. fusion of right and noncoronary cusps, severe steno-

Which of the following is the appropriate next step B. Descending thoracic aortic replacement using a in his management? clamp and sew technique A. Continued surveillance imaging and adding labetalol C. Extent I thoracoabdominal aortic replacement using 300 mg TID to his medical regimen left heart bypass Review Questions 1019

D. Extent II thoracoabdominal aortic replacement using A. Bedrest with strict BP control using esmolol and a temporary axillofemoral shunt nitroprusside E. Thoracic endovascular aortic repair using a B. Endoaortic graft placement for the type B stent-graft dissection 28. Regression analysis of robust natural history data C. Immediate operative repair of the ruptured type B from patients with ascending aortic has dissection been used to determine the diameter at which the D. Immediate surgical repair of his type A dissection probability of rupture or acute dissection increases E. Replacement of the descending aorta for the type B significantly. At this “hinge point” (arrow in the fig- dissection ure), patients become 30% more likely to have a 30. A 68-year-old woman underwent a Crawford extent I complication such as ascending aortic dissection or graft repair of a thoracoabdominal aortic aneurysm rupture during their lifetime. (see figure). Adjuncts used during the repair included

40 left heart bypass, selective visceral perfusion, and cerebrospinal fluid drainage. The patient was extu- bated on the first postoperative day and the cerebro- spinal fluid drainage catheter was taken out 1 day

30 later. Six hours after the catheter was removed the patient complained of severe headache. She is in sinus rhythm with a heart rate of 92 and a blood pressure of 148/82 mmHg. The previous spinal catheter site is dry and without evidence of bleeding or inflammation. 20 White blood cell count = 6.8K/mm3, hemoglo- bin = 10.3 g/dL, platelet count = 122,000/mm3, glu- cose = 98 mg/dL, and creatinine = 1.1 mg/dL.

10 Increased Risk of Complication (%)

0

Aortic Size (cm) -10 The diameter at which this hinge point occurs is A. 4 B. 5 C. 6 D. 7 E. 8 29. A 75-year-old man presented to the ER complaining of tearing pain in his back. His blood pressure was 190/110, HR was 90 and rhythm was normal sinus. A representative frame from his CT angiogram is shown. Which of the following is the most appropri- ate management?

The most appropriate initial management is to A. continue observing the patient and keep her on strict bed rest B. do a lumbar puncture for CSF studies and start IV antibiotics 1020 Review Questions

C. obtain an immediate brain CT scan injury was treated by thoracic aortic endovascular D. place an epidural blood patch repair with a stent graft and her pelvis was repaired E. reinsert the spinal drain and reduce intrathecal pres- by internal fixation. She recovered from her inju- sure to 10–15 mmHg ries, however for the last several months she com- 31. A 45-year-old man underwent repair of a thoracoab- plained of becoming acutely lightheaded while dominal aortic aneurysm 72 h ago. He emerged from exercising. On two occasions in the last month she anesthesia with a normal neurologic exam but had syncopal episodes while using an elliptical required continued ventilator support due to respi- trainer. On physical examination her right and left ratory insufficiency and copious bronchial secre- arm blood pressures are 140/80 mmHg and tions. He was started on a propofol infusion for 120/70 mmHg, respectively. A surveillance CT sedation and the nurse soon noted that he is no lon- angiogram is shown. ger able to move his lower extremities. His blood pressure is 90/40 mmHg, CVP is 16, and his intrathe- cal pressure is 15 mmHg. Which of the following is the most important ini- tial step in caring for this patient? A. Administer a bolus of naloxone and discontinue the propofol B. Consult neurosurgery to place a lumbar cerebrospi- nal fluid (CSF) drain C. Discontinue the propofol infusion and start a nicar- dipine infusion D. Get an MR angiogram of the thoracic aorta to evalu- ate his spinal arterial supply E. Stop the propofol infusion and administer a norepi- nephrine infusion 32. A 63-year-old man with an aneurysmal chronic aortic dissection underwent Crawford extent II graft replacement of the entire thoracoabdominal aorta (see figure). The aortic reconstruction included reimplant- ing the 11th and 12th intercostal arteries. He awoke neurologically intact, was extubated on the first post- operative day, and was transferred out of the ICU on POD# 3. However, the next morning he reported being unable to stand. On exam he is afebrile, his pulse is regular at 86, and BP is 148/78. He is alert, is breath- ing comfortably, and his mental status, sensation, and upper extremity strength are normal. While supine, he is able to lift both legs against gravity but not against resistance. Labs include: WBC 8.6K/mm3, hemoglobin 9.8 g/dL, hematocrit 30.8%, platelet count 138K/mm3, glucose 105 mg/dL, and creatinine 0.9 mg/ dL. The most appropriate immediate action is to A. Administer intravenous heparin B. Begin IV phenylephrine infusion to raise MAP over 90 mmHg C. Insert a lumbar drain and remove cerebrospinal fluid Which of the following is the most appropriate D. Obtain a thoracic aortogram to assess the patency of next step in the management? intercostals arteries A. Carotid ultrasound and Doppler assessment E. Transfuse one unit of packed red cells B. Cerebral angiogram to determine blood flow 33. A 34-year-old woman sustained multiple injuries patterns from a motor vehicle accident 18 months ago C. Coil embolization of the left subclavian artery including a pelvic fracture, pulmonary contusions, D. Left carotid to subclavian bypass and aortic transection at the isthmus. Her aortic E. Left vertebral artery bypass Review Questions 1021

34. A 26-year-old man is brought into the emergency A. Exploratory laparotomy followed by open surgical department following a high-speed motor vehicle repair of the aortic injury. accident. He is awake and alert but complaining of B. Open surgical repair of the aortic injury with an abdominal pain. The chest radiograph reveals no interposition graft by clamp-and-sew technique fol- pneumothorax, but he does have three rib fractures lowed by exploratory laparotomy. on the left. A CT scan of the chest, abdomen, and C. Open surgical repair of the aortic injury with an pelvis reveals a contained partial aortic transection interposition graft utilizing femoral vein-to-femoral in the chest (see figure) and free blood in the abdo- artery cardiopulmonary bypass, followed by explor- men with a suggestion of a splenic laceration. atory laparotomy. D. Stent graft repair of the aortic transaction followed by exploratory laparotomy to repair the intra-abdom- inal injuries. E. Exploratory laparotomy to repair the abdominal injuries and observation of the aortic injury with serial CT scans. 35. A 29-year-old man was ejected during a high-speed motor vehicle collision. He was intubated in the field and remained hemodynamically stable, but he was still unarousable to verbal stimuli upon ER arrival. He has a right femur fracture and he withdraws to painful stimuli with all four extremities. Brain CT shows fron- tal lobe contusions, minimal edema and no frank intra- The most appropriate management of this cranial hemorrhage. Abdominal CT shows no free air patient’s injuries is or solid organ injury. Chest CT frames are shown.

The best management for this patient is arterial lines were placed for blood pressure monitor- A. craniectomy, then descending aortic replacement ing. After sternotomy he was placed on cardiopulmo- using left heart bypass nary bypass via right femoral arterial and right atrial B. endovascular stent grafting of the proximal descend- cannulation. During systemic cooling the right radial ing thoracic aorta arterial pressure acutely fell to a mean pressure of C. intracranial ventriculostomy in the ICU and IV 35 mmHg while the left femoral arterial pressure was nitroprusside for BP control 70 mmHg. The perfusionist noted an increase in the D. thoracotomy and aortic wrapping with a Dacron resistance of the bypass circuit. Which of the follow- patch or prosthesis ing is the correct response to this situation? E. thoracotomy for descending aortic replacement A. Accelerate the cooling process and proceed with under hypothermic circulatory arrest aortic repair 36. A 52-year-old man presented with an acute aortic dis- B. Apply an aortic cross-clamp and administer section originating in the aortic root that extended cardioplegia beyond the aortic bifurcation. He was taken to the C. Cannulate the right subclavian artery for antegrade operating room and right radial and left femoral perfusion 1022 Review Questions

D. Recannulate and perfuse through left femoral artery complaining only of this chest pain. Her blood pres- E. Rewarm the patient sure is 88/40 mmHg. ECG confirms normal sinus 37. A previously healthy 75-year-old woman experienced rhythm with a rate of 88 and no significant ST acute, severe upper chest pain radiating to her neck. changes. Chest radiograph is also unremarkable. CT On initial examination she is neurologically intact angiography frames are shown.

Which of the following is the most appropriate D. Endovascular placement of a thoracic aortic stent graft strategy to manage this patient? E. Total arch replacement with an elephant trunk A. Admission to the intensive care unit, intravenous 38. A 45-year-old, healthy, active man had a near-synco- labetalol, and echocardiogram pal episode when exercising. Further history revealed B. Aortic root and ascending aortic replacement with a repetitive palpitations over the past few years, which mechanical valved conduit his family physician was treating with a beta blocker. C. Aortic valve resuspension, ascending aortic and The patient is currently asymptomatic. A current hemiarch replacement ECG is shown.

Review Questions 1023

The most effective management for this patient is Which of the following is the best management for A. Atrioventricular nodal ablation and dual-chamber this patient? pacemaker insertion A. ECMO, diuresis, antibiotics, AVR after there is evi- B. Dofetilide 500 μg twice daily dence of complete eradication of sepsis C. Percutaneous ablation of the bundle of Kent B. Immediate aortic valve replacement and culture-­ D. Percutaneous left atrial ablation directed postoperative IV antibiotics for 6 weeks E. A thoracoscopic Maze procedure C. IV antibiotics, CT scan of abdomen and brain, emer- 39. A 56-year-old man underwent an orthotopic heart gent AVR if no contraindications are found on head transplant for idiopathic dilated cardiomyopathy. CT scan Baseline creatinine was 1.4. Drug support on arrival D. No cardiac operation due to end-stage presentation in ICU included epinephrine 2 μg/(kg min) and isu- and severe comorbidities (renal failure and CHF) proterenol 0.02 μg/(kg min). He was given tacrolimus E. PICC line, 6 weeks course of intravenous antibiotic 2 mg sublingually, methylprednisolone 125 mg IV q therapy followed by aortic valve replacement 8h to be followed by a prednisone taper, and myco- 42. A 72-year-old man is in the coronary care unit in pul- phenolate mofetil 1 g IV q 12h. He was extubated 6 h monary edema. He was generally healthy until the postop. Hemodynamics remained stable but urine preceding 2 weeks of shortness of breath on exertion, output progressively dropped to 20–30 cc/h 36 h malaise, low grade fevers, and night sweats. On postop and creatinine is now 3.2. Which of the follow- examination he has a loud apical systolic murmur ing medication changes should be made? and scattered bilateral rales. A recent echocardio- A. Hold all immunosuppression gram revealed an anterior leaflet perforation and a B. Hold the methylprednisolone mobile vegetation that measured 1 cm in diameter. C. Hold the mycophenolate mofetil dose once and Lab results include an erythrocyte sedimentation resume at half dosage rate of 81 mm/h and a WBC of 32,000. Multiple blood D. Hold the tacrolimus dose cultures drawn 24 h ago in his primary care physi- E. No changes are needed cian’s office are now positive for Streptococcus bovis. 40. Maintenance immunosuppression following cardiac He is currently receiving appropriate antibiotics and transplantation typically consists of “triple therapy”. aggressive diuretic therapy. Which of the following combination is an established Which of the following most strongly indicates regimen? that surgery should be performed urgently in this A. Cyclophosphamide, steroids and azathioprine patient? B. Cyclosporine, OKT3 and steroids A. the anterior leaflet perforation C. Cyclosporine, prednisolone, azathioprine B. the 1 cm mobile vegetation D. Tacrolimus, cyclosporine and rapamycin C. the pulmonary oedema E. Tacrolimus, mycophenolate mofetil and azathioprine D. WBC = 32,000 41. A 55-year-old man was been treated for fever of E. Streptococcus bovis unknown origin on the medical service for 2 weeks. 43. A 99-year-old man who was reportedly normally His admitting symptoms consisted of progressive functioning presented with acute shortness of breath malaise, fever, chills, and arthralgias. Lab results followed by syncope. After stabilization, his blood included pancytopenia, hyperuricemia, and 2/2 pressure without pharmacologic support is 110/80 blood cultures were positive for Enterococcus faeca- and he is obtunded but responds to pain. His echo- lis. Worsening renal failure required hemodialysis cardiogram demonstrates akinesis of the anterior (present creatinine 6.4, BUN 68). His chest radio- wall, posterior wall and apex (LVEF = 27%) with graph is shown. Cardiac echo showed 4+ aortic aortic stenosis (calculated valve area = 0.4 cm2), three regurgitation and leaflet vegetations. plus mitral regurgitation, and an estimated pulmo- nary artery pressure of 80 mmHg. A CT angiogram suggests significant proximal LAD and RCA steno- ses. BNP on admission was 2000 ng/L. Troponin lev- els are pending. The family wants everything done. Of the following choices the appropriate recommen- dation to the family and primary physician is A. Candid end-of-life discussion and hospice care B. Cardiac cath with angioplasty and stenting of the LAD and RCA C. A milrinone bolus followed by infusion and re-evaluation 1024 Review Questions

D. Minimally invasive aortic valve replacement and to new onset atrial fibrillation. Intraoperative valve double coronary artery bypass analysis confirmed a large valve with excess tissue E. Placement of an intra-aortic balloon pump and and bileaflet prolapse. Operation included triangu- re-evaluation lar P2 resection, posterior commissuroplasty, 44. A 45-year-old woman complained of dyspnea on PTFE neo-chordae to the A2, P2 and P3 segments, exertion (noted after two flights of stairs). Her his- and a size 40 mm semi-rigid complete annuloplasty tory included rheumatic fever during childhood. Her ring. She was separated from cardiopulmonary transthoracic exercise echocardiogram documented bypass easily on a low dose of epinephrine and with mitral valve stenosis (valve area = 1.5 cm2) with normal pulmonary pressures and a CVP of 14. The immobile and calcified leaflets (Wilkins Score = 10), post-­repair TOE shows systolic anterior motion. preserved left ventricular function, an estimated sys- The epinephrine was stopped and CVP was tolic pulmonary artery pressure of 54 mmHg, and a increased to 16 mmHg with volume loading, but the mean valve gradient of 10 mmHg. findings persisted. Cardiopulmonary bypass was The most appropriate management is reinstituted. A. with a mechanical valve Which of the following is the best course of action? B. Mitral valve replacement with a tissue valve A. Anterior leaflet resection C. Open surgical commisurotomy B. Mitral valve replacement D. Percutaneous mitral balloon valvotomy C. Placement of a larger annuloplasty ring E. Repeat clinical assessment in 6–12 months D. Replacement of the ring with an incomplete flexible 45. A 73-year-old woman became acutely short of breath band and coughed up thin sputum. In the emergency E. Sliding posterior leaflet plasty department her heart rate was 120/min and blood 47. An otherwise healthy 50-year-old, asymptomatic pressure was 90/40. She was immediately intubated man has severe mitral valve regurgitation due to and cardiac catheterization showed severe 3-vessel Barlow’s disease and preserved left ventricular func- coronary disease with an occluded distal right coro- tion. Which of the following is the most relevant to nary artery that faintly reconstitutes via collaterals determine whether to proceed with mitral valve sur- from the anterior descending. An echo showed 4+ gery at this time? mitral regurgitation with flail chords. A family mem- A. Accessibility to a center performing a high volume ber indicated that the patient had severe chest pain 5 of mitral valve operations days ago with intermittent mild-to-moderate chest B. Age of the patient pain since. Emergent CABG and mitral repair was C. High probability of valve repair based on the echo done, which included reattaching the papillary mus- findings cle and placing an annuloplasty ring. After weaning D. Potential candidacy for warfarin therapy from cardiopulmonary bypass the transesophageal E. A pulmonary artery pressure of 44/28 mmHg echo showed 3+ mitral regurgitation. 48. Which of the following conditions is a contraindica- Which of the following statements about this case tion to mitral valve surgery via a right mini-­ is most accurate? thoracotomy approach? A. Cardiopulmonary bypass should be resumed and A. A need for concomitant tricuspid valve operation chordal sparing mitral valve replacement performed B. Obstructive peripheral vascular disease B. Concurrent revascularization was unnecessary since C. Pectus carinatum the main cause of her acute deterioration was the D. Previous CABG surgery regurgitation E. Previous right pleurodesis C. The observed mitral regurgitation will improve, 49. A 75-year-old woman had severe hypertension and therefore, no further intervention is necessary moderate mitral regurgitation with minimal annu- D. Since the major cause of cardiogenic shock is mitral lar calcification. You have just completed an regurgitation perioperative mortality is low when uncomplicated coronary artery bypass × 3 and mitral competence is re-established mitral valve repair. As you are removing the retro- E. Temporary mechanical support with an LVAD is grade cannula and preparing to remove the venous indicated to allow the infarction to heal and mitral cannulas you notice a stable but moderate-size hae- regurgitation to resolve matoma in the region of the mid-portion of the 46. A 35-year-old asymptomatic woman had severe coronary sinus. mitral regurgitation due to long standing Barlow’s The most appropriate management of this prob- disease. Mitral valve repair was recommended due lem is to Review Questions 1025

A. administer protamine, decannulate, complete the and no significant pulsus paradoxus. Pressure trac- operation, and avoid manipulation of the involved ings from his heart catheterization are shown. No area. coronary disease was present. B. externally open up and explore the haematoma to patch the anticipated defect. C. externally open up and explore the haematoma, then do a primary suture repair. D. harvest another vein segment, dissect the haema- toma, then ligate and bypass the injured vessel. E. resume CPB and reopen the left for inspec- tion and possible intra-cardiac repair. 50. A 69-year-old man has been treated medically for 3 years for progressive heart failure. Prior history includes CABG 4 years previously. Echocardiography shows an ejection fraction of 20%, LVEDD 7.5 cm, LESVI 160 mL/m2, severe mitral regurgitation due to bileaflet tethering and moderate tricuspid regurgi- tation. Cardiac catheterization shows diffuse coro- nary artery disease bypass grafts to the anterior descending, diagonal, first obtuse marginal and pos- terior descending arteries are all patent. PA pressure is 58/25, PCW = 28, and CVP = 18. Which of the following best predicts that mitral valve repair will NOT provide sustained benefit for this patient? A. Ejection fraction = 20% B. LVEDD = 7.5 cm, LVESVI = 160 mL/m2 C. Mean CVP = 18 A. Constrictive pericarditis D. PA pressure = 58/25 B. Dilated cardiomyopathy E. Patent coronary artery bypass grafts C. Pericardial tamponade 51. Cardiac resynchronization therapy is an appropriate D. Restrictive cardiomyopathy therapy in which of the following patients? E. Tricuspid valve disease A. Fifty-eight-year old woman, non-ischemic cardio- 53. Because of severe heart failure symptoms a 71-year-­ myopathy, class IV heart failure, on chronic dobuta- old man has been taken to the operating room. mine, EF 15%, atrial fibrillation, with a QRS Through a , a complete phrenic-to-­ ­ complex duration 135 ms phrenic is underway for constriction. B. Sixty-three-year old woman, non-ischemic cardio- The left side dissection is complete but it is difficult to myopathy, class III heart failure on tailored therapy, identify a tissue plane over the right side of the heart. EF 30%, third degree heart block with a QRS com- Suddenly dark blood floods the operative field. plex duration 115 ms The most appropriate management is C. Sixty-five-year old man, hypertensive cardiomyopa- A. Application of topical sealants, direct pressure for thy, class III heart failure on tailored therapy, EF 5 min, wide chest tube drainage and closure 40%, sinus rhythm with right bundle branch block B. Bovine pericardial patch repair using pledgeted D. Seventy-year old man, non-ischemic cardiomyopa- sutures followed by chest closure thy, class III heart failure on tailored therapy, EF C. Felt strip repair of the right ventricular rent and com- 15%, atrial fibrillation with left bundle branch block pletion pericardiectomy E. Seventy-two-year old woman, ischemic cardiomy- D. Foley catheter placement through the injury, contin- opathy, class IV heart failure on tailored therapy, EF ued dissection and then primary closure 15%, sinus rhythm with left bundle branch E. Institution of cardiopulmonary bypass, closure of 52. A 62-year-old man with rheumatoid arthritis pre- the defect and completion pericardiectomy sented with shortness of breath and lower extremity 54. An 82-year-old woman underwent AVR with a bovine oedema. Physical exam revealed a positive pericardial tissue valve 3 weeks ago. Her in-hospital Kussmaul’s sign, marked jugular venous distension, postoperative course was uneventful. Three weeks 1026 Review Questions

after surgery she was unable to lay supine and com- The best treatment for this condition is plained of severe shortness of breath with minimal A. Anticoagulation for pulmonary embolus exertion. She presented to the ER, where examina- B. Non-steroidal anti-inflammatory agents tion revealed no murmur, heart rate = 106, and C. Pericardial stripping BP = 92/55. The chest radiograph is shown. D. E. Reoperation and placement of another valve 55. A 54-year-old man with a normal ejection fraction underwent an uneventful 4-vessel CABG. On arrival in the ICU he was on no inotropes or vaso- pressors and cardiac output was 6 L/min. Mediastinal chest tube drainage was 175 mL/h for the first 3 h but diminished significantly thereafter. Six hours after surgery he developed a vasopressor requirement, which has been escalated to 10 μg/ min of norepinephrine (noradrenaline). His car- diac output now is 3.1 L/min, CVP is 21 mmHg and urine output has diminished to 20 cc/h despite giv- ing 1 L of IV saline. A stat ECG is shown.

Which of the following is the most appropriate With respect to the pericardial abnormality dis- therapy? played, which of the following statements is A. 250 cc of 5% albumin correct? B. Cardiac catheterization A. The abnormality typically becomes symptomatic C. Epinephrine (adrenaline) at 2 μg/kg/min with time. D. Intra-aortic balloon pump B. Aggressive HAART therapy will expedite E. Mediastinal re-exploration resorption. 56. An asymptomatic 42-year-old HIV positive woman C. Drainage is indicated to define the aetiology. had a chest CT study (see image). D. The aetiology is most probably fulminant CMV infection. E. The finding marks a more advanced stage of HIV disease. 57. A 75-year-old woman with a history of tobacco abuse presented with increasing shortness of breath and malaise. Contrast CT images are shown. Myocardial was ruled out and pericardio- centesis revealed no malignant cells.

Review Questions 1027

The most appropriate next step in the manage- 59. A 51-year-old attorney with no significant past med- ment is ical history was evaluated for complaints of short- A. CT guided biopsy of the right upper lobe mass and ness of breath, mild chest discomfort and fullness in definitive chemotherapy his head after going for his morning walk. During B. Empirical antibiotic therapy with broad-spectrum stress testing in his cardiologist’s office he had fre- antibiotic for 10 days quent PVCs and chest discomfort. A nitroglycerine C. Median sternotomy and anterior pericardiectomy drip was initiated, which caused hypotension and D. Subxiphoid pericardial window with pericardial he was taken to the hospital. Coronary angiography biopsy showed no evidence of coronary artery disease, and E. VATS pericardial window with drainage into the he was discharged to home 2 days later with a diag- pleural space and pleurodesis nosis of pleurisy. The patient re-presented after a 58. Eighteen months ago a 44-year-old woman com- sudden collapse and hypotension required vaso- pleted 42 Gy of radiation therapy for mediastinal pressor support. Following intubation the chest CT Hodgkin’s lymphoma. She now complains of exer- was obtained. tional dyspnea. Her chest radiographs have been clear, but a transthoracic echocardiogram showed normal ejection fraction, a moderate pericardial effusion and pericardial thickening. Five hundred milliliter of serous fluid was removed by pericardio- centesis but the patient’s symptoms persist. Which of the following should be done next? A. CT scan of the chest B. Diuretic therapy C. Exercise stress testing D. A progressive exercise and conditioning regimen E. Right heart catheterization 1028 Review Questions

Which of the following is the most appropriate extremity oedema and ascites. A pansystolic murmur next step? is heard over the left sternal edge. An echocardio- A. Heparinization and repeat left heart catheterization gram of the tricuspid valve demonstrates fused com- B. Immediate operation for type A dissection missures, thickened and fused chordae tendineae C. Endotracheal intubation and nitric oxide therapy and combined regurgitation and stenosis. D. Lytic therapy for acute myocardial infarction Of the following choices the most appropriate E. The procedure for which heart machine was therapy is invented A. Commissurotomy and repair with rigid ring 60. A 75-year-old woman presented with a shortness of B. Intravenous broad spectrum antibiotics breath. The cardiac auscultatory exam was notable C. Replacement with a bioprosthetic valve for an S3 that increased with inspiration and D. Replacement with a mechanical valve decreased with a Valsalva maneuver. Also heard was E. Valve excision without replacement a parasternal pansystolic murmur that increased 62. A 42-year-old woman with shortness of breath and with inspiration. On physical exam, she had signifi- peripheral edema is scheduled for mitral valve repair cant oedema. Right heart pressure tracings are (flail P2 resection and ring annuloplasty). PA pres- shown (see figure). sure at catheterization was 56/18 and the preopera- tive echo showed moderate tricuspid regurgitation. The best treatment is to A. Leave the tricuspid alone since the patient has only Electrocardiogram mild symptoms B. Repair the tricuspid because the patient has a reduced ejection fraction C. Repair the tricuspid because the patient has pulmo- nary hypertension 40 D. Replace the tricuspid valve because the annulus is dilated E. Wait to decide until the intraoperative transesopha- geal echocardiogram is done RV 63. A 54-year-old woman complains of fatigue and weak- ness. On physical exam, she has a parasternal pan-

Pressure (mmHg) systolic murmur that increases with inspiration.

20 There is jugular venous distention with fused c- and v-waves followed by a prominent y-descent. A chest RA radiograph demonstrates cardiomegaly, increased right atrial and ventricular size and a prominent azygos vein. Echocardiography will most likely demonstrate A. Mitral regurgitation B. Mitral stenosis 0 C. Tricuspid regurgitation D. Tricuspid stenosis and regurgitation Which of the following operations should be E. Tricuspid stenosis offered to her based on this information? 64. A 35-year-old intravenous drug user underwent exci- A. Isolated tricuspid valvuloplasty sion of his tricuspid valve for endocarditis and intra- B. Mitral valve repair alone venous antibiotic treatment of multiple small lung C. Mitral valve repair and tricuspid valve repair abcesses. Informed consent included an understand- D. Mitral valve replacement and tricuspid valve replace- ing that his chance of surviving 15 years was 63% ment with bioprosthetic valves and that most likely reason for his death would be E. Mitral valve replacement and tricuspid valve replace- A. Coronary artery disease ment with mechanical valves B. Liver failure from right heart overload 61. A 59-year-old woman has elevated 5-hydroxytrypta- C. Pulmonary embolism mine (5-HT) serum levels and consequent systemic D. Recurrent endocarditis symptoms. Examination shows significant lower E. Return to drug addiction Review Questions 1029

65. A 45-year-old woman is to undergo mitral valve E. Replace the mitral with a bioprosthesis and place a repair for myxomatous disease. She has 4+ tricuspid mechanical valve in the tricuspid position regurgitation. The treatment that will give her the 67. A 73-year-old, 93 kg woman had severe aortic ste- most sustained benefit for the right-sided pathology nosis. Preoperative left ventricular function was is normal, PA pressure was 50/23 and she had no A. Annuloplasty with a near-complete rigid ring obstructive coronary lesions. At operation the aor- B. Annuloplasty with a partial flexible ring tic annulus was too small to accept a 19 mm stented C. DeVega suture annuloplasty bioprosthesis, and aortic root reconstruction with a D. Mitral valve intervention only 21 mm porcine root was performed. Because of E. Tricuspid valve replacement mild right ventricular dilatation, milrinone (0.5 μg/ 66. A 60-year-old woman has ascites and peripheral (kg min)) was used to wean from bypass. Frequent oedema. In addition to a repairable prolapsing P2 PVCs improved with a lidocaine infusion. TOE mitral scallop, she has multiple chords ruptured on showed good valve function, LV ejection fraction of her tricuspid valve that preclude repair. She is in 55%, and no intracardiac air. Thirty minutes after sinus rhythm. arrival in the ICU, however, she had short runs of Of the following, the best plan is to ventricular tachycardia, hypotension, and cardiac A. Repair the mitral valve and leave the tricuspid valve index dropped from 2.5 to 1.5 L/(min m2). PA pres- as it is sures are unchanged from the OR (40/20) and the B. Repair the mitral valve and place a bioprosthetic milrinone and lidocaine infusions are checked and valve in the tricuspid position found to be running. The ECG is shown. The chest C. Repair the mitral valve and place a mechanical valve film is unremarkable and the arterial blood gas val-

in the tricuspid position ues are: pH 7.35, pCO2 41 mmHg, pO2 245 mmHg,

D. Replace the mitral and tricuspid valve with mechani- HCO3 21 mEq/L. cal prostheses

Which of the following is the most important man- 68. A 57-year-old obese man underwent minimally inva- agement strategy? sive aortic valve replacement via partial upper ster- A. Amiodarone 150 mg IV followed by infusion. notomy. The inferior rim of his left main coronary B. Discontinue milrinone and begin adrenaline at 2 μg/ ostium was noted to be 11 mm above the aortic annu- min. lus. On POD#4, he developed atrial fibrillation and C. Increase the ventilator rate from 14 to 18 breaths per blood pressure was labile. Beta blockers and amioda- minute. rone converted the rhythm to normal sinus but his D. Place an intra-aortic balloon pump at the bed side. blood pressure now continues to be labile. Urine out- E. Return to the OR for aortocoronary bypass put is 20 cc/h and creatinine (normal before surgery) grafting. is 1.7. His wound is shown. Neck are hard to 1030 Review Questions

evaluate, but he has 2+ pretibial edema bilaterally. A. anomalous hepatic venous drainage. He says he feels fine. B. anomalous pulmonary venous drainage into the right Which of the following should be done? . A. Give furosemide to increase urine output. C. coronary sinus septal defect. B. Obtain a chest CT to rule out dissection. D. stenosis. C. Obtain a precordial echocardiogram. E. ventricular septal defect. D. Request an urgent coronary angiogram. 73. During intracardiac repair of a complete atrioven- E. Start a dopamine drip and get a chest radiograph. tricular septal defect there is an excessive amount of 69. An 82-year-old man who was otherwise healthy blood streaming from the left atrium. The most likely underwent aortic valve replacement with a porcine cause of this is prosthesis for critical, symptomatic aortic stenosis. A. anomalous origin of the left coronary artery from the His operation was uncomplicated and the postopera- pulmonary artery. tive echocardiogram demonstrated a well-seated B. azygous continuation of the inferior vena cava. valve without dysfunction. The evening of surgery C. partial anomalous pulmonary venous connection. (8 h after leaving the operating room) the nurse D. a patent ductus arteriosus. reported that he was unable to move the entire right E. a patent left superior vena cava to the coronary sinus. side of his body. He was completely awake and 74. The most likely cause of a poor long-term result fol- responsive. A non-contrast brain CT scan was unre- lowing surgical repair of a complete atrioventricular markable. Optimal management should include septal defect is A. anticoagulation and alpha agonist administration. A. left ventricular outflow tract obstruction. B. bedside duplex of lower extremity veins. B. mitral valve regurgitation. C. with catheter-directed, intra- C. pulmonary vascular obstructive disease. arterial thrombolysis. D. residual atrial septal defect. D. immediate transport to hyperbaric oxygen chamber. E. residual ventricular septal defect. E. intravenous thrombolytic therapy. 75. The most common late complication after an arterial 70. A 3 year old child is noted to have a murmur of aortic switch operation for transposition of the great arter- stenosis. Cardiac catheterization demonstrates a ies is peak-to-peak left ventricular-to-aortic gradient of A. atrial arrhythmias. 50 mmHg, mild aortic regurgitation, and the echocar- B. coronary ostial stenosis. diogram shows a circumferential fibrous ridge in the C. pulmonary artery stenosis. left ventricular outflow tract. Proper management is D. recanalization of the ductus arteriosus. A. aortoventriculoplasty (Konno’ procedure). E. stenosis of the aortic anastomosis. B. continued close follow-up until the gradient increases 76. The arterial switch operation is optimally performed further or symptoms appear. in the first 2 weeks of life for patients with transposi- C. open aortic valvotomy. tion of the great arteries and intact ventricular sep- D. percutaneous balloon dilatation. tum because the E. transaortic resection. A. atrial septal defect created by is 71. The aortogram of a symptomatic 10 year old child likely to close and cause lethal hypoxemia. with Williams syndrome shows supravalvular aortic B. left ventricle is still physiologically prepared to stenosis. Of the following procedures, appropriate function as a systemic ventricle. management is C. older newborn is a poorer surgical candidate. A. composite aortic root replacement. D. right ventricle is physiologically prepared to eject B. Dacron graft replacement of ascending aorta. against elevated neonatal pulmonary vascular C. open valvotomy on bypass. resistance. D. patch enlargement of the aorta. E. use of prostaglandins for longer than 2 weeks is E. percutaneous balloon dilatation of the aortic valve. contraindicated. 72. A 6 year old patient underwent closure of a secun- 77. A 2 month old patient with atresia of the tricuspid dum atrial septal defect. At operation a persistent valve, ventricular septal defect, normally related left superior vena cava was noted which was sepa- great vessels, and no pulmonary stenosis will benefit rately cannulated for bypass. The atrial septal defect most from which of the following surgical was closed with a pericardia! patch. Upon weaning interventions? from bypass, systemic arterial desaturation is noted. A. patch closure of an atrial septal defect A likely cause is unrecognized B. patch closure of a ventricular septal defect Review Questions 1031

C. patch closure of both the atrial and ventricular septal C. veno-arterial ECMO via femoral vein and femoral defects artery D. pulmonary artery banding D. veno-venous ECMO via jugular vein E. systemic-to-pulmonary artery shunt E. ventricular assist device (VAD) without oxygenator 78. The best initial diagnostic studies for a 4 month old via existing right atrial and aortic cannulas infant with stridor since birth are 82. A 16-year-old patient underwent an atrio-pulmonary A. angiography and barium swallow. type for tricuspid atresia at age 2. B. chest X-ray and angiography. She now complains of worsening exercise tolerance. C. chest X-ray and barium swallow. The late complication that is most likely to cause her D. computed tomography and angiography. symptom is: E. magnetic resonance imaging and computed A. left ventricular failure tomography. B. pleural effusion 79. The most important reason for cardiac catheteriza- C. protein-losing enteropathy tion in the preoperative evaluation of a 2 year old D. right ventricular failure child with a ventricular septal defect (VSD) is to E. supraventricular arrhythmia determine 83. A 5-year-old girl had neonatal repair of truncus arte- A. coronary artery anatomy. riosus with a right ventricle-to-pulmonary artery B. if additional VSDs exist. homograft conduit. She developed progressive exer- C. location of the VSD. cise intolerance and echocardiography indicated D. pulmonary vascular resistance. conduit stenosis. She underwent cardiac catheteriza- E. relationship of the VSD to the tricuspid valve. tion and balloon dilation of the conduit valve. Which 80. A 4 year old asymptomatic child has the following of the following findings is an indication for conduit data recorded at cardiac catheterization valve replacement? A. heavily calcified conduit Pressures (mmHg) % Saturation B. pulmonary valve regurgitation SVC 2 55 C. right ventricular end-diastolic pressure = 10 mmHg RA 2 57 IVC 2 60 D. right ventricular hypertrophy RV 50/2 85 E. right ventricular pressure = 80% of systemic PA 52/26 (38 mean) 85 pressure LA 2 100 84. A 17-year-old man is found to have a restrictive ven- LV 125/4 100 tricular septal defect with a sinus of Valsalva aneu- AO 125/82 (98 mean) 100 rysm of the non-coronary sinus. Rupture of this aneurysm would most likely result in an aortic fis- The Qp/Qs (ratio of pulmonary to systemic blood tula to: flow) is A. left atrium A. 0.5 B. left ventricle B. 1.0 C. pericardial sac C. 1.5 D. right atrium D. 2.0 E. right ventricle E. 3.0 85. A 3-year-old boy born with tricuspid atresia was ini- 81. A 1-week-old neonate with transposition of the great tially palliated with a right modified Blalock-Taussig arteries, ventricular septal defect, and coarctation of shunt. At age 9 months he underwent shunt take- the aorta underwent a primary one-stage correction down and a bidirectional Glenn (cavopulmonary through a median sternotomy. Despite maximal inotro- anastomosis) procedure. Which of the following pic support, the baby cannot be weaned from cardio- would significantly increase his risk for a lateral tun- pulmonary bypass. Transesophageal echocardiography nel Fontan procedure? reveals biventricular dysfunction and a technically sat- A. central pulmonary artery stenosis isfactory repair with flow in both coronary arteries. B. interrupted inferior vena cava with azygous Which of the following forms of mechanical support continuation represents the best option for this patient? C. left ventricular end-diastolic pressure = 16 mmHg A. intra-aortic balloon pump via aortic arch D. systolic pulmonary artery pressure = 16 mmHg B. veno-arterial ECMO via existing right atrial and aor- E. transpulmonary gradient = 6 mmHg tic cannulas Answers

1. Answer: C is reasonable for a stable patient, but a neurosurgical In the Arterial Revascularization Trial (ART), patients problem such as an acute bleed is unlikely in this situa- were randomly assigned to receive either bilateral inter- tion. Hyperbaric oxygen is an appropriate treatment to nal thoracic artery grafts or a standard single left internal help clear nitrogen-rich gas bubbles, but chambers are thoracic artery graft during coronary artery bypass graft- seldom available and postoperative cardiac patients are ing (CABG). A prespecified interim analysis at 5 years not easily transported or cared for in such microenviron- showed no significant differences between the two strat- ments. Fortunately, most air embolism complications egies with regard to all-cause mortality or the rate of the are self-limited problems and injury can be minimized composite outcome of death from any cause, myocardial by employing brute force (increasing cardiac output and infarction, or stroke. At 10 years, in intention-to-treat systemic pressure) to purge the obstructing gas bubbles. analyses, there were no significant between-group dif- Induced moderate hypothermia will minimize the meta- ferences in all-cause mortality; in the rate of the com- bolic demands of the heart and brain until the problem is posite outcome of death, myocardial infarction, or resolved. stroke; or in the rate of repeat revascularization. 3. Answer: C 2. Answer: A A bicuspid aortic valve (BAV) is a recognized risk Problems that arise after uneventful operations can be factor for the development aneurysms of the ascending frustrating, yet a thoughtful approach to diagnosis and aorta and subsequent complications such as dissection prompt management can be lifesaving. Emergent car- or rupture. BAV disease seems to follow an autosomal diac catheterization in a postoperative patient with new dominant pattern of inheritance with incomplete pene- onset ventricular arrhythmias may be indicated, but sev- trance. A number of histologic abnormalities have been eral factors make coronary disease unlikely in this case. identified in association with BAV including decreased No native occlusive lesions were identified at preopera- fibrillin 1 concentration, increased metalloproteinase 2 tive catheterization. With the aorta open, return of retro- activity, elastin fragmentation and smooth muscle cell grade cardioplegia from both coronary ostia would have reorientation. Distances between elastic lamellae are been visualized. Unnoticed coronary injury is unlikely, greater than in patients with tricuspid aortic valves. and obstruction by debris would have manifested intra- Subsequently, these patients also have higher rates of operatively with ventricular dysfunction and difficultly dilation of the ascending aorta on long-term echo sur- weaning from cardiopulmonary bypass. In this scenario, veillance than do patients with tricuspid valves. the most likely cause is air embolism from small air A patient with a BAV and ascending aortic diameter bubbles trapped in the pulmonary veins, the left atrium greater than 45 mm is at increased risk of late aortic dis- or the left ventricle after an open operation. This prob- section and sudden death compared to patients with lem can occur despite aggressive de-airing and the use smaller aortic diameters. Reconstruction of the entire of transesophageal echo. In a supine patient the right aortic root and ascending aorta with a composite graft coronary ostium is superior in the ascending aorta and should be considered if the aortic root is also dilated. is, therefore, the most common site of coronary air The increased complexity and operative risk of a Bentall embolism. Air embolism into the carotid circulation can procedure mitigate against its routine use in patients cause early postoperative seizures. Ambient air (80% without coexistent root dilatation. Replacement of the nitrogen) is absorbed into solution much more slowly ascending aorta with a tube graft in addition to valve

than pure oxygen or CO2. For this reason insufflation of replacement or repair is an appropriate procedure in the

the pericardial well during operation with CO2 is rou- absence of root enlargement. Reduction aortoplasty with tinely practiced by many surgeons. Anticonvulsant med- external wrapping of the ascending aorta has been ications are indicated and obtaining a CT scan emergently reported to have acceptable mid-term results, but

© Springer Nature Switzerland AG 2020 1033 S. G. Raja (ed.), , https://doi.org/10.1007/978-3-030-24174-2 1034 Answers

­long-­term follow-up suggests an increased incidence of surveillance plus antiplatelet drugs are recommended progressive dilatation in comparison to tube graft for nearly all such patients. Combined carotid endarter- replacement of the ascending aorta. ectomy and CABG may be appropriate for selected 4. Answer: E patients with high-grade carotid lesions. Recent studies Paravalvular leak occurs in approximately 1–3.4% of offer some insights into which patients can be expected patients after aortic valve replacement. Factors impli- to have accelerated progression of aortic stenosis. Valve cated in the development of periprosthetic leaks include obstruction correlates with valvular calcification. inadequate debridement of annular calcium, endocardi- Therefore, valves with moderate stenosis and heavy cal- tis, using non-pledgeted annular sutures, and anatomic cification of the annulus and leaflets tend to worsen and weakness in the area of the non-coronary sinus. cause symptoms sooner than valves with minimal calci- Indications for intervention include regurgitation worse fication. Additionally, the starting valve area is related to than “mild,” worsening ventricular function, hemolysis, the interval time to symptom development. Studies of and endocarditis. “Vena contracta” refers to the point in aortic stenosis have shown an annual decrease in aortic a fluid stream through an orifice where the diameter of valve area of approximately 0.12 cm2/year and an the stream is the least. In the case of a paravalvular leak increase in mean gradient of 5–10 mmHg/year. Some the abnormal fluid stream can be imaged and measured authors suggest that this progression is more rapid in by echo techniques. The maximum contraction is usu- elderly patients. Regarding the described patient, the ally identified slightly downstream from the orifice. A calcification and moderate gradient indicate that failure regurgitant jet vena contracta of 0.6 cm indicates mod- to replace the aortic valve at this time puts him at high erate to severe regurgitation, which is consistent with risk for a required second operation within a few years. this patient’s postoperative ejection fraction deteriora- The potential role of transcatheter valve insertion in this tion. Percutaneous closure of prosthetic paravalvular setting remains to be defined. leaks is an appealing alternative to reoperation, but 6. Answer: D embolization, hemolysis and lack of successful com- Reoperative bypass surgery is becoming less common plete closure have plagued such attempts. The devices with increased application of percutaneous revascular- used in reported cases have been designed for other pur- ization. Before IMA advantages were generally accepted, poses to resolve lesions such as muscular VSD, ASD, LAD revascularization was routinely done with a saphe- and PDA. Reoperation is the conservative and only reli- nous vein graft. If reoperation is necessary, use of the able approach to resolve the complication described. LIMA would appear to be an attractive option. However, The simplest repair technique, when anatomically fea- its use in this context can result in marginal LAD perfu- sible, is to close the defect with one or more transmural sion prior to full maturation of the arterial graft. When a pledgeted sutures that incorporate the sewing ring and saphenous vein graft was previously placed to the LAD the aortic wall. In the noncoronary sinus sutures can be acceptable reoperative strategies include either replacing placed from outside-­to-inside and up through the sewing the old graft with a new saphenous vein graft (removing ring at the level of the annulus. Leaks in other paravalvu- or ligating the old graft), or placing a LIMA to the LAD lar locations can be more difficult to resolve, but further and leaving the old graft intact. This woman’s old patent dissection and alternate exposures may allow similar LAD vein graft was ligated, so the LIMA inflow is now transmural suture repair in the left and right coronary the sole source of anterior wall perfusion. Noninvasive sinuses. Prosthetic valve re-replacement may be determination of LIMA flow is reliable, and if no techni- required, but this is associated with reduced postopera- cal error was made then going back on bypass to redo the tive survival. LIMA-LAD anastomosis will not resolve the ongoing 5. Answer: D ischemia, which is the underlying problem in the Identification of moderate aortic stenosis at the time described scenario. The measured flows are reasonable of myocardial revascularization presents a clinical values for fresh grafts of the types specified. Augmenting dilemma. Concomitant valve replacement poses and anterior wall flow is the appropriate immediate response. increased operative risk, but the stenosis may progress Placing an intra-aortic balloon pump and adding inotro- and require a reoperation later, when risks are even pic support may temporize the situation, but the underly- higher. Aortic stenosis is a progressive disease process ing cause remains untreated. LVAD placement fails to with a long latent period in which patients may be address the pathophysiology and is premature. If another asymptomatic. Once symptoms do develop, the risks graft fails to resolve the problem, then embolized debris progress exponentially to death unless the obstruction is from old graft manipulation and/or other etiologies must removed by valve replacement. The natural history of be considered, and extraordinary circulatory support may asymptomatic carotid stenosis is relatively benign, and be justified. Answers 1035

7. Answer: D rarily with pericardiocentesis and placement of an intra- Most authorities recommend repair of the mitral or aortic balloon pump. Catheterization is not mandatory. tricuspid valve in any patient with severe regurgitation In the pre-­thrombolytic era this complication of myo- of either valve. The American College of cardial infarction presented within 2 weeks of the initial and American Heart Association (ACC/AHA) guide- infarct (peak at 5 days). However in the current era ven- lines recommend repair for severe ischemic mitral tricular rupture can develop within hours of thromboly- regurgitation or severe tricuspid regurgitation at the time sis or reperfusion. Emergent operation for this problem of concomitant coronary bypass or other surgery. Severe can be technically challenging, but it may be further pulmonary hypertension and poor left ventricular func- complicated by bleeding secondary to the drugs given. tion are each associated with persistent or progressive Like postinfarction ventricular pseudoaneurysm and postoperative tricuspid regurgitation (even after repair), papillary muscle rupture, free wall rupture tends to be so many surgeons advocate an aggressive strategy to the result of acute single-vessel coronary occlusion in correct even lesser degrees of tricuspid regurgitation. the absence of developed collaterals. The size of the The severity of ischemic mitral regurgitation or func- area of infarction is often relatively limited and con- tional tricuspid regurgitation is highly dependent on pre- comitant procedures are seldom necessary. Surgical load, afterload and ventricular function. Fasting and treatment may include: (1) wide pledgeted closure of sedation under general anesthesia may result in substan- the rupture (often difficult because of friable tissue); (2) tial down-grading of mitral or tricuspid regurgitation infarctectomy with patch closure of the defect; (3) pled- when assessed in the operating room. The preoperative geted closure of the rupture with a wide Dacron patch echo assessment under normal loading conditions offers sewn to good tissue and placed over the repair as a sec- the most accurate assessment of atrio-ventricular valve ond layer; and (4) direct application of a patch glued regurgitation. An aggressive plan for CABG plus mital over the infarct, done off-pump if possible. The last of and tricuspid repairs is correct. Excellent results have these options has persuasive advantages. Fibrin glue is been reported for repair of ischemic mitral valve regur- inadequate for reliable adhesion and strength, but gela- gitation with flexible, rigid and semi-rigid rings. The tin resorcin formaldehyde glue has been used effec- best mid-term results of survival and freedom form tively. Various cyanoacrylate synthetic glues have recurrent ischemic mitral regurgitation have been convincing strength and proven efficacy, and the nature reported with semi-rigid annuloplasty rings. Regardless of the patch material seems less critical when these of the type of ring used, after 1 year there appears to be adhesives are used. no difference in annular motion or basal contraction 9. Answer: D after mitral annuloplasty. The posterior annulus is the Some variety of myocardial rupture (free wall, septal, perimeter of the tricuspid valve most prone to dilatation. or papillary muscle) complicates up to 10% of all acute The primary role of a tricuspid annuloplasty ring is to myocardial infarctions (AMI) and is responsible for correct dilatation of the posterior, and to a lesser extent 15% of in-hospital mortality following AMI. Reports the anterior, annulus. Most modern tricuspid “rings” are indicate that early reperfusion for coronary disease not complete rings, but partial with a gap to minimize (either PCI or CABG) minimizes such complications. the risk of damage to conduction system. The atrio-­ Myocardial rupture is more common among patients ventricular node and penetrating bundle of His are close older than 60 years and women as compared to men to the septal leaflet base and the commissure between (1.4:1). Most myocardial ruptures occur between 1 and the septal and anterior leaflets. The anatomy of the valve 5 days following AMI. Echocardiography offers rapid, and the triangle of Koch are seen in the second figure accurate diagnosis of these conditions. In the described (forceps tips are in the coronary sinus). The apex of this case the findings were near-complete papillary muscle triangle corresponds to the area of most concern for rupture, which can manifest as acute pulmonary edema injury from suture placement. with associated tachycardia, tachypnea, rales, respira- 8. Answer: E tory distress, and hypotension. In roughly 2/3 of cases of This woman’s echocardiogram shows a large ante- acute papillary muscle rupture the posteromedial papil- rior effusion with diastolic RV collapse and impaired lary muscle is involved. The posteromedial papillary LV function with posterior akinesis. Postinfarction free muscle usually receives blood supply from a single wall ventricular rupture is most frequently seen in source—either the right coronary artery or the circum- hypertensive women over age 60. Signs and symptoms flex coronary artery in a left dominant system. The of cardiac tamponade, heart failure and shock are most anterolateral papillary muscle receives dual blood often seen. Ventricular rupture represents a surgical ­supply from the left anterior descending and circumflex emergency, although patients may be stabilized tempo- coronary arteries. The presentation of acute MR 1036 Answers

represents only 1–2% of all cases of ischemic MR. Rapid and following guidelines and established protocols for diagnosis is essential to survival. A murmur may be institution of extraordinary mechanical ventricular sup- absent following papillary muscle rupture given a rapid port (beyond inotropes and/or intra-aortic balloon pump) equalization of pressure between the left atrium and improve survival dramatically. Samuels et al. showed ventricle. “Medical” therapy includes afterload reduc- that early (within 3 h) institution of VAD support once tion with vasodilators and/or insertion of an intra-aortic two or more high-dose inotropes are required produced balloon pump. However, often these patients develop improved survival. Several current ventricular support acute, severe cardiogenic shock that is unresponsive to devices are readily available and can be inserted without inotropic and IABP support. Mitral valve replacement is extensive previous experience. The described patient has the best surgical option in most cases. It is associated profound left ventricular failure and ample evidence of with 10–40% mortality depending on patient comorbidi- inadequate systemic perfusion. Correcting the acidosis ties, but without valve replacement this condition is may optimize the response to pharmacologic support, fatal. but the underlying muscle dysfunction is global and 10. Answer: B severe. The man’s pulmonary hypertension is a reflec- Ten-year survival for asymptomatic patients with LV tion of left ventricular failure, and response to nitric aneurysms is 90%. Therefore, surgical intervention oxide is unlikely. Functional grafts are documented, so should not be undertaken based on the presence of an additional bypass conduits are not indicated. Contractility aneurysm alone. Aneurysm resection is not beneficial will not return quickly and aggressive inotropic stimula- for asymptomatic patients with large (>5 cm) aneu- tion will worsen the chance of recovery. Venoarterial rysms. Indications for surgery include documented ECMO is an option, but it is not necessary if mechanical increase in size of the aneurysm, angina, congestive support is instituted early enough to prevent lung dam- heart failure symptoms, arrhythmias, rupture, and age. Isolated LVAD placement will probably suffice for peripheral embolism. Eight-five percent of LV aneu- univentricular failure, although occasionally biventricu- rysms are anterior, as the left anterior descending coro- lar support becomes necessary. Failure to institute nary artery is the most commonly involved vessel. The extended mechanical support early in a patient such as LAD should be bypassed at the time of aneurysm repair. this inevitably leads to morbidity, failure to wean from The most significant factor influencing postoperative VAD support, and ultimate mortality. mortality is poor left ventricular function before opera- 12. Answer: E tion. Other factors that predict post-repair mortality This man had multiple pre-existing problems and include three-vessel disease, preoperative malignant underwent three on-pump procedures within 48 h. He arrhythmias, and advanced age. “Circular” repair (Dor, was intubated, profoundly coagulopathic, in worsening Jatene, others) is generally favored over linear closure, acute renal failure, and grossly volume overloaded. His but the influence on outcome of the method of repair large antero-apical infarction makes the anastomosis of remains somewhat controversial. Ejection fraction will the LVAD inflow cuff difficult. Nonetheless, bleeding improve following repair for almost all patients. from this anastomosis is unlikely because the left ven- However, primary should be con- tricle is well decompressed by the LVAD. The low intra- sidered if the ejection fraction is less than 20%, if symp- ventricular pressure plus protamine and blood toms are NYHA class IV, and if the patient is a good component transfusions control this bleeding. Similarly, candidate for transplantation. The causes of death for with proper technique bleeding from the outflow graft medically managed patient are: arrhythmia (44%), heart anastomosis should not be problematic. The patient’s failure (33%), and recurrent MI (11%). renal dysfunction will likely develop into post-LVAD 11. Answer: E renal failure and require continuous venovenous hemo- Fortunately, the need for post- ventricular dialysis (CVVHD). In the past, renal failure was single assistance in patients with reasonable preoperative func- most powerful predictor of post-LVAD mortality during tion appears to have decreased because of improved device support. Aggressive, early institution of CVVDH myocardial protection strategies. However, the relative with adequate circulation from VAD support will likely number of patients with poor preoperative cardiac func- lead to recovery of renal function. Liver failure often tion has risen and thus the need for unexpected post-­ adds to multi-system organ failure in chronic end-stage cardiotomy support remains essential. The most effective heart failure cases, but once acceptable hemodynamics strategy to deal with this scenario is early institution of are restored, hepatic function will recover. This patient ventricular support prior to end-organ damage. This also appears destined to have worsening right ­ventricular requires considerable judgment, experience with device function. Volume overload is evident from the high CVP options and dogged attention to care details. Creating but PA pressure elevation is modest. Despite the previous Answers 1037

LVAD the patient’s right heart is failing, a secondary tions are less with continuous flow VADs. Bacteremia complication that develops in approximately one third (25%) and pump endocarditis (3%) are equivalent. of LVAD implants. A BiVAD strategy is probably neces- However, driveline, pocket, wound, and local device sary, as RV failure following LVAD alone results in infections and sepsis are all substantially lower with 40–50% mortality before successful bridge to axial flow devices. transplantation. 14. Answer: D 13. Answer: A The patient is an obese young man with blood type O Bleeding is the most common complication associ- who has failed medical management despite several ated with LVAD implantation. Excessive bleeding (>4 admissions and good outpatient compliance. Studies units) occurs in most patients, and returning to the oper- have shown that repeated hospitalizations for fluid over- ating room for control is frequent (up to 60%). load are generally associated with high mortality. The Preoperative comorbidities such as renal insufficiency, recent development of ventricular tachyarrhythmia hepatic congestion, and low flow state contribute to the makes inotropic therapy dangerous, even with an ICD in high incidence of bleeding. Strategies to manage coagu- place. This patient failed his pulmonary vasodilator lopathy include preoperative Vitamin K, preoperative challenge, so a plan to use milrinone in the hopes of low- plasmapheresis, priming the cardiopulmonary bypass ering pulmonary vascular resistance is futile. With his circuit with fresh frozen plasma, and use of antifibrinol- demonstrated irreversible pulmonary hypertension ytics. Some surgeons routinely perform delayed chest (PVR > 4 Woods units, TPG > 15 mmHg) the risk of closure so tamponade and emergency reoperation can be acute right heart failure following a heart transplant is avoided. Planned, early post-pump treatment of coagu- considerable, and is associated with elevated short- and lopathy with FFP, cryoprecipitate and platelet transfu- long-term mortality. sions is routine in many centers. Recombinant factor VII This patient’s weight and blood type each portend a has been used effectively both intra- and postoperatively long waiting time for an appropriate donor heart. Noting in VAD patients, but several reports caution against rou- the recent ventricular arrhythmias in the setting of high tine use of this drug and component therapy because of pulmonary vascular resistance, mechanical support is potential pro-thrombotic consequences. After LVAD needed. Despite the lack of response to nitroprusside placement the reported rates of stroke and thromboem- and milrinone, the refractory high PVR is probably not bolism vary from 10% to 30%. The type of device, anti- absolutely “fixed” in this young adult. Volume and pres- coagulation regimen, and various definitions of sure unloading by an LVAD will improve systemic out- “neurological event” each influence statistical reports of put and should progressively decrease pulmonary these complications. Excessive transfusion of blood vascular resistance, thus reversing a contraindication to products is associated with several postoperative com- transplantation. plications, including right ventricular (RV) failure, 15. Answer: E which is reported to occur in up to 30% of patients who Right heart function must be accurately assessed dur- require an LVAD. RV function in the initial postopera- ing patient evaluation for destination therapy (DT). All tive phase often determines mortality after LVAD. A DT at present involves placing an isolated left-sided typical LVAD patient requires one or two inotropes and mechanical device. Hence, long-term success requires either inhaled nitric oxide or prostacyclin to achieve and adequate right ventricular (RV) function. Although there maintain adequate LVAD flows. All agents usually can are no firm guidelines that assure RV function, current be weaned off slowly over a few days. Despite this antic- workup includes a hemodynamic profile and echocar- ipated outcome, a right-side VAD is required in approxi- diography. Low pulmonary artery pressure, low RV mately 10% of LVAD patients. The incidence of RVAD stroke work index (RVSWI) and several other parame- use has not been higher with the use of newer generation ters have been associated with RV failure after LVAD rotary pumps. Outcome reports indicate similar results placement. A right ventricular “risk score” based on pre- for nearly all bridge LVAD devices, with survival to operative clinical, laboratory, echocardiographic, and transplantation in 60–70% of patients and survival to hemodynamics may be used to predict post-LVAD RV discharge after transplantation of 50–60% of the total failure. Catheterization is mandatory in this patient to population. For comparison, 84% of non-bridged listed gather pressure data and to evaluate the posterior patients receive a transplant (UNOS, 2008; USA patients descending artery graft. If the conduit supplies a large 07/01/2004–12/31/2006). Despite increased immuno- territory of the RV then it must be preserved or regrafted. logic sensitization in bridged patients, contemporary This patient’s mechanical valve is a significant issue. post-transplant survival is similar to non-bridged Diverting flow with an LVAD will lead to stasis and the patients, which is 85–90%. Overall infectious complica- prosthetic valve will clot. If the valve then opens peri- 1038 Answers

odically, the risk of thromboembolization is quite high. as capacitance vessels, primarily filling during the This complication must be prevented, although adding period of diastole (as much as 85% of total flow), and an aortic valve procedure may increase the morbidity intramural pressure and resistance to myocardial perfu- and mortality of LVAD placement. This does not, how- sion progressively increase from the outer to inner layers ever, prohibit destination LVAD placement. Replacement of the heart. Myocardial arterioles have tremendous of the existing mechanical valve with a bioprosthesis vasodilatory reserve capacity and enable high flow with adds to the cross clamp time, and this does not prohibit low resistance in response to exercise. Two mechanisms ejection and the thromboembolic risk. The best strategy of vasodilatation may be available: larger proximal ves- is to close the prosthetic valve with a prosthetic patch or sels by endothelium-derived nitric oxide, and direct immobilizing suture through an aortotomy. The disad- stimulation of smooth muscle cell alpha2-receptors by vantage of this strategy is that if the LVAD fails, there adenosine and other metabolites. will be no cardiac output and mortality is certain. A coronary artery with an oblique origin, an intramu- 16. Answer: A ral (within the wall of the aorta) course, or a position Patients with acute myocardial infarction who have between the great arteries (as seen in this patient’s CT) failed percutaneous angioplasty and undergo emergent may be compressed and its reservoir capacity may be CABG surgery are at high risk for postcardiotomy severely limited. The law of Laplace (tension = pres- shock. Delayed reperfusion can be deleterious and it is sure × radius) dictates that comparable pressure in larger unclear how much myocardium is salvageable. The inci- vessels creates greater wall tension. Thus the aorta and dence of post-cardiotomy shock is 0.2–0.5 % of all main pulmonary artery can compress smaller vessels patients who undergo cardiac surgery. Unfortunately, that course between them or are intramural. Signs and survival to recovery remains poor, reported to be symptoms in older patients can include angina, syncope, 20–40% even in recent studies. Strategies that have arrhythmias, MI, enlarged heart, and sudden death, shown improved survival have generally involved early which is most often associated with activity. Sudden application of mechanical support and transitioning to death is frequently observed in association with anoma- chronic LVAD or BiVAD support if necessary. In the lous origin of the left coronary artery from the right case discussed above the patient is not a candidate for sinus of Valsalva. This pattern is associated with an destination therapy, which is approved only for chronic oblique origin, a slit-like ostium, and an intramural, heart failure. Furthermore, adding another complicated inter-arterial course. Each of these features (oblique ori- operation in this setting almost certainly invites further gin, ostial stenosis, intramural and intra-arterial com- complications. The simplest available option to establish pression) can diminish flow and result in coronary adequate temporary support is often best. While a percu- ischemia. Limited flow and loss of reservoir capacity taneous device with left atrial cannulation achieved cannot meet the increased myocardial oxygen demands through the atrial septum is a possibility for left-side associated with exercise. support, many cases of postcardiotomy shock require 18. Answer: D biventricular support. This patient’s recent MI involved This young patient has an aortic root aneurysm mea- the right coronary, so biventricular failure is almost cer- suring nearly 8 cm in diameter and aortic root replace- tain. ECMO or short-term mechanical ventricles or ment is indicated. Separately replacing the aortic valve blood pumps should be used in this case, with specifics and the tubular portion of the ascending aorta is not ade- determined by availability and expertise of the surgical quate because this would leave aneurysmal sinus tissue team involved. Cannulation schemes for RV support that would be prone to further dilation and eventual rup- should be the right atrial appendage with an outflow ture or dissection. A mechanical composite valve graft graft to the pulmonary artery or by cannulating through will provide a durable root replacement in this patient. the RV outflow tract. Cannulation for the left side most Tension-free reattachment of the coronary arteries is crit- commonly involves LA through the interatrial groove or ical to prevent disruption of the coronary anastomoses the right superior pulmonary vein, with return to the and pseudoaneurysm formation. In most cases this can aorta through direct cannulation or a graft. However, be accomplished by mobilizing the coronary arteries many experienced surgeons advocate direct cannulation along with buttons of surrounding aortic tissue. In of the LV apex to allow for maximal decompression of patients undergoing reoperations, especially those with the left ventricle, fewer cannula complications and extremely large aneurysms, it is often difficult to mobi- higher VAD flows, and improved chance for recovery. lize the coronary arteries enough to enable their direct 17. Answer: D reattachment to the graft without tension. In this situation The pattern of coronary blood flow during the coro- the Cabrol technique may be the best option. Each coro- nary cycle is unique. Epicardial coronary vessels serve nary ostium is sutured to an end of a 10 mm graft, which Answers 1039

is then connected to the aortic graft with a side-to-side regurgitation. In addition to aortic root aneurysms, anastomosis. Tension-free coronary reattachment is then patients with Marfan syndrome are prone to mitral valve achieved without extensive mobilization. Valve-sparing prolapse and to aneurysms and dissections involving the root replacement is an excellent option for young patients aortic arch and thoracoabdominal aorta. This propensity who need root replacement, provided that the annulus is for developing multiple aortic aneurysms mandates that not excessively dilated and the valve leaflets are in satis- periodic aortic imaging be continued for life, even after factory condition. However, the severely dilated root, a patient has undergone successful aortic root replace- cusp prolapse, and fenestrated valve leaflets in the ment. In most patients who meet the clinical diagnostic described patient make valve-­sparing root replacement a criteria for Marfan syndrome (i.e., the Ghent criteria), less satisfactory option. Cryopreserved homografts are the disease is attributed to a mutation in the gene for prone to degeneration, which becomes problematic in fibrillin-1. Fibrillin-1 is a glycoprotein component of younger patients. Homografts are generally reserved for microfibrils, which are key structural components of the cases in which local infection makes it desirable to avoid extracellular matrix within the media of the aortic wall. using a synthetic graft and other prosthetic material. Elastin and collagen are the major components of the 19. Answer: D aortic wall matrix, conferring resilience and strength, All patients with Marfan syndrome require aggres- respectively. Mutations that affect these proteins are sive cardiovascular imaging surveillance including responsible for other cardiovascular conditions. For echocardiograms obtained at regular intervals. Patients example, the vascular form of Ehlers-Danlos syndrome with Marfan syndrome are at high risk of acute aortic is caused by mutations affecting type III collagen, and dissection even when the aorta is only modestly dilated, supravalvular aortic stenosis is caused by mutations and this is particularly true in patients with a family his- affecting elastin. Mutations that affect a receptor for tory of dissection. Therefore, aortic root replacement is transforming growth factor-β have been described in recommended when the ascending aortic diameter patients with clinical features of Marfan syndrome, and exceeds 5.0 cm in patients with Marfan syndrome, in these mutations have been recently shown to cause contrast to the 5.5-cm threshold generally used in non-­ Loeys-Dietz syndrome, a condition associated with par- Marfan patients. When patients with Marfan syndrome ticularly severe aortic and peripheral vascular manifes- undergo isolated graft replacement of the tubular portion tations that require aggressive surgical treatment. of the ascending aorta, progressive disease of the aortic 21. Answer: A root—including annuloaortic ectasia, aortic valve regur- The patient has a strong family history of aortic dis- gitation, and acute dissection—invariably develops, section and enough stigmata to fulfill criteria for a diag- necessitating complex reoperation. Therefore, ascend- nosis of the Marfan syndrome based on the Ghent ing aortic repair should always involve full root replace- nosology. The aortic root has a maximal orthogonal ment using either valve-replacing or valve-sparing diameter of 50 mm. In combination, these findings techniques, even if the sinuses of Valsalva are not yet should prompt aortic root replacement. Given his age, dilated and the aortic valve is competent. two appropriate options include a mechanical valve-­ Although treatment with β-blockers is currently stan- conduit or valve-sparing aortic root replacement. dard for Marfan patients, the effectiveness of this ther- Ascending aortic replacement is appropriate, but total apy has not yet been proved. Recent experiments show aortic arch replacement is unnecessary and unjustified in that losartan, an angiotensin II type 1 receptor antago- this case given the normal caliber of the aortic arch. nist, prevents aortic dilatation in a mouse model of Replacement of only the noncoronary sinus using a Marfan syndrome. This work has generated a current modified remodeling technique is not an adequate pro- randomized clinical trial. Regardless of losartan's effi- cedure for an aortic root aneurysm in a Marfan patient. cacy in preventing aneurysms, however, it will not The tissue of the entire root is involved, even if the dila- replace surgical treatment for patients who have indica- tion is asymmetric. Similarly, a Wheat procedure (aortic tions for aortic replacement. valve replacement combined with supracoronary 20. Answer: B ascending aortic replacement) does not appropriately Marfan syndrome is a heritable connective tissue dis- address the pathology of the aortic root. While losartan order associated with severe orthopedic, ophthalmo- (angiotensin II receptor antagonist) administration may logic, and cardiovascular manifestations. The most be beneficial to reduce the risk of progressive aortic common cardiovascular problem in patients with Marfan ­dilation in Marfan children, this patient’s aortic root has syndrome is aortic root aneurysm caused by progressive already dilated to a critical dimension. Expansion to annuloaortic ectasia. If untreated, such aneurysms ulti- 50 mm portends a significant risk of aortic catastrophe mately rupture, dissect, or cause severe aortic valve and justifies operative intervention at this time. 1040 Answers

22. Answer: D fies the second operation. This patient’s aortic root is not The CT scan reveals an aneurysm that involves the enlarged, so root replacement is not warranted. ascending aorta, transverse aortic arch, descending 24. Answer: D thoracic aorta, and abdominal aorta. Patients with Myocardial protection during prolonged cooling for such large aneurysms involving both the proximal circulatory arrest can be challenging in patients with (ascending and arch) and distal (descending or thora- ascending/arch aneurysms and aortic valve regurgita- coabdominal) aortic segments present a clinical chal- tion. The incompetent valve allows pump flow to distend lenge. Replacement of both segments during a the fibrillating left ventricle and causes a marked single-stage procedure is advocated by some surgeons, increase in sump drainage. If not corrected, ventricular provided that the distal aspect of the aneurysm does distention will cause increased myocardial oxygen not extend into the abdomen. When the distal aspect demand and decreased subendocardial perfusion. The does involve the abdominal aorta, repair is generally resulting inadequate myocardial preservation will lead performed in stages. The dilemma of which aneurysm to severely impaired postoperative cardiac function. to treat first is introduced. In general it is preferable to Adding another ventricular sump may lessen distention replace the proximal segment during the initial opera- but will not correct or reliably control this problem. The tion. This offers the advantage of allowing concomi- best way to protect the myocardium is to induce cardio- tant treatment of coronary artery and valvular disease. plegic arrest as rapidly as possible after fibrillation. The elephant trunk technique can be used in these When anatomically feasible, the aneurysmal ascending cases because it leaves a segment of graft suspended in aorta should be clamped. This will stop the inflow into the descending thoracic aorta, which simplifies the the distended left ventricle, increase systemic flow, and second operation. The distal segment is repaired first allow delivery of cardioplegia. In the described patient, in patients in whom the thoracoabdominal aortic aneu- the ascending aorta is not dissected and atheromata are rysm is disproportionately larger that the ascending not prominent so it can be clamped safely. Placing a aortic aneurysm, or in whom the distal aneurysm is needle in the ascending aorta is unlikely to allow ade- causing the predominant symptoms, as in the patient quate cardioplegia delivery, as the cardioplegia will flow described here. When the distal segment is repaired through the regurgitant valve instead of pressurizing the during the first operation, the reversed elephant trunk root and perfusing the coronary arteries. The best technique–in which an 8 cm segment is invaginated approach is to open the ascending aorta and deliver car- into the upper end of the descending aortic graft—can dioplegia directly into the coronary ostia. When clamp- be used to simplify reconstruction of the arch during ing the aorta is not possible (e.g., in patients with the second operation. enormous aneurysms, see figure) delivering continuous 23. Answer: A retrograde cardioplegia while manually decompressing When a patient presents with aneurysms involving the ventricle and continuing rapid cooling may be the the ascending and thoracoabdominal aortic segments, only option. the decision about which aneurysm to treat first is based 25. Answer: D on three major factors. The presence of symptoms is the This apparently healthy man has a dilated ascending most important consideration, and the lesion causing the aorta with a maximum orthogonal diameter of 44 mm most life-threatening symptoms should be repaired first. that was incidentally identified during evaluation for an Second, the presence of correctable cardiac disease is an upper respiratory infection. He has no family history of issue because clamping the descending thoracic aorta an aortic aneurysm or aortic catastrophe, no stigmata of induces substantial cardiac stress. Addressing correct- a connective tissue disorder and he is not hypertensive. able coronary and valvular disease first reduces the risk Consequently, there is no justification for aortic replace- of cardiac complications during the thoracoabdominal ment at this time as the risks of operation exceed the risk aortic repair. A third consideration relates to assessing of a catastrophe of a 44 mm aorta in a tall 62-year-old the relative sizes of the aneurysms. When one aneurysm man. In asymptomatic patients risk factors that prompt is disproportionately larger than the other, the larger operation include rapid growth (e.g. >5 mm/year) or an aneurysm is treated first. Because this patient has symp- aortic index (see below) greater than 1.5. This corre- toms of aortic valve regurgitation and no symptoms sponds roughly to a diameter of 50–55 mm for a man related to the thoracoabdominal aortic aneurysm, the this size. most appropriate choice is to address the aortic valve, Oral losartan is currently under investigation for use ascending aorta, and transverse aortic arch first. Using to prevent aortic root dilatation in patients, particularly the elephant trunk technique, which leaves a segment of children, with Marfan syndrome. However, for a patient graft suspended in the descending thoracic aorta, simpli- with a dilated ascending aorta but neither hypertension Answers 1041

nor Marfan syndrome, there is inadequate evidence to nulation site in this case because of the risk of retrograde justify its use. This patient requires long-term surveil- embolization of atheromatous debris from the abdomi- lance of his ascending aorta from this point forward. nal and descending thoracic aortic segments. The axil- Annual cross-sectional imaging is appropriate since he lary artery is usually free of atherosclerotic disease and, has no clinically significant aortic valve regurgitation. therefore, represents an excellent alternate cannulation Considering improved safety of large-scale vascular site in this situation. operations and the evolving alternative to open proce- 27. Answer: C dures, there are parallel relaxations of the data-driven This patient’s residual type B aortic dissection has “hinge points” at which relative risks are balanced. In become aneurysmal. The maximal orthogonal diameter Marfan and collagenopathy patients one suggested trig- at the distal arch and proximal descending thoracic aorta ger for intensive surveillance (if not intervention) is an exceeds 65 mm. Additionally, his hypertension is refrac- absolute diameter of 40 mm or a calculated “aortic tory to a five-drug antihypertensive regimen, and he is index” greater than 1.3. A value ≥1.5 is considered a symptomatic with upper thoracic back pain. He is clear operative threshold for these at-risk patients by already on both a beta-blocker (metoprolol) and an some experts. For patients without connective tissue α-blocker (doxazosin), so adding labetalol would likely concerns, these threshold values are somewhat higher. offer little benefit. Continued medical management is The aortic index calculation (maximum measured not appropriate. Stent-grafting a chronic, dissected tho- orthogonal diameter/predicted diameter for the age, sex racoabdominal aortic aneurysm, particularly in a young, and body surface area) was described by Roman et al. in otherwise healthy patient, is appropriate only on an 1989. The formulas, derived from echo measurements of experimental basis at this time. normal patients, for predicted sinus dimensions are: This patient’s aorta is dilated down to the celiac artery, but the infrarenal aorta is of normal caliber. • Children (<18 years): 1.02 + (0.98 × BSA (m2)) Replacing the entire thoracoabdominal aorta is unneces- • Adults (18–40 years): 0.97 + (1.12 × BSA (m2)) sary given the extent of dilatation. It would pose added • Adults (>40 years): 1.92 + (0.74 × BSA (m2)) risk of paraplegia, especially in the setting of dissection. • Regarding the described patient (BSA calculates Descending aortic replacement alone would leave resid- to 2.13 m2): ual dilated aorta near the diaphragm, and using a clamp-­ • aortic index = 4.4/(1.92 + [0.74 × 2.13]) and-­sew technique would add significant risk of = 4.4/(1.92 + 1.58) paraplegia in this setting. Extent I thoracoabdominal = 4.4/(3.5) aortic replacement using left heart bypass is suitable for = 1.26, which rounds up to 1.3, so the this patient. The patient has bovine arch anatomy and it threshold for surveillance (but not for is likely that a safe clamp site could be established operation) has been reached. between the left subclavian and the innominate trunk. 26. Answer: D Using deep hypothermia for spinal cord protection is Intraoperative epi-aortic ultrasound is a sensitive also appropriate, although it adds time to the operation method to identify and determine the severity of aortic time and may increase blood transfusion requirements. atherosclerosis. There are several systems for grading 28. Answer: C the severity of aortic atherosclerotic disease, but all of A critical aspect of surgical decision making about them consider atheroma thickness greater than 5 mm to aortic disease is appreciating the point at which the be severe. Increasing severity of aortic atheromatous potential benefits of an operation, in terms of preventing plaque is associated with an increasing risk of neuro- rupture or acute dissection, outweighs the risks the oper- logic complications. Minimizing aortic manipulation by ation entails. Outcomes after elective repair of ascend- using alternate cannulation sites and avoiding aortic ing aortic aneurysms are excellent, as several series clamping is the cornerstone of reducing the risk of neu- report mortality rates less than 5%. Recent natural his- rologic complications in patients with aortic atheromata. tory studies show that patients with ascending aortic This is a particular challenge for those who require aor- aneurysms are at low risk of catastrophic events until the tic valve replacement because perfusion and myocardial aortic diameter exceeds 6.0 cm (see figure). This finding protection options must be identified. Because this supports elective repair of ascending aortic aneurysms woman’s ascending aortic atheromata extend into the that reach a diameter of 5.5 cm, before they reach the arch, the ascending aorta should not be cannulated or critical size threshold. Lower absolute diameter triggers clamped. The aortic clamp can be eliminated by replac- are applied to patients with heritable connective tissue ing the aortic valve during a period of deep hypothermic disorders, including Marfan syndrome and Loeys-Dietz circulatory arrest. The femoral artery is not an ideal can- syndrome, or a family history of aortic dissection. Other 1042 Answers

recommended indications for repair include symptom- ing by CT or MRI is mandatory every few days, as is atic aneurysms (regardless of diameter) and those repeat echocardiography. observed to increase in diameter 1.0 cm or more within 30. Answer: C a year. Intracranial hemorrhage is an uncommon but poten- 29. Answer: A tially catastrophic complication of cerebrospinal fluid The CT angiogram shown reveals a Type B intramu- drainage. Because this complication may require emer- ral hematoma (IMH) with a penetrating atherosclerotic gent neurosurgical intervention, all patients who have ulcer (PAU). As the name implies, PAU is associated had cerebrospinal fluid drainage (CSF) should undergo with severe atherosclerotic disease and the radiologic immediate computed tomography of the head if postop- appearance is similar to ulceration in the GI tract. The erative headache develops. The most likely cause of the natural history and best treatment of PAU is somewhat headache is a CSF leak. There is usually no external controversial. Recent studies suggest that conservative wetness, because the fluid generally leaks into the tissue treatment is appropriate under most conditions. surrounding the dural opening rather than through the However, close clinical follow-up is mandatory as thin-­ puncture site in the skin. Headaches caused by CSF walled saccular pseudoaneurysms or rupture may occur. leaks are typically positional, i.e., symptoms are much This patient’s ipsilateral pleural effusion heightens con- worse when the patient sits or stands. They are often cern for such complications, and TEE may define the accompanied by sensitivity to light. Most headaches local anatomy more precisely. While endoaortic stent caused by cerebrospinal fluid leaks require treatment grafting might appear to be well suited for a localized with an epidural blood patch, but this should only be PAU, severe atherosclerosis in the aorta and the embolic considered after intracranial bleeding has been ruled risk of manipulation, the risk of paraplegia and access out. In many cases, the patient is unable to get out of bed difficulties moderate these considerations. The current before definitive treatment. Early treatment, therefore, clinical experience with endografting for IMH-PAU is expedites overall postoperative convalescence because it meager. Intramural hematoma was initially thought to enables the patient to resume ambulation. result from rupture of vasa vasora, though there is little It is unlikely that this patient has meningitis, an evidence to support this theory. Others believe that IMH extremely rare complication of lumbar drain insertion, is always associated with some degree of intimal tear given the absence of fever and leukocytosis. Furthermore, that may go undetected. The case of PAU with IMH, as collecting CSF is contraindicated until imaging has is in this patient, appears to contradict that theory. On excluded the presence of an intracranial process. CT or MRI imaging IMH appears as a crescentic or con- Similarly, reinserting a cerebrospinal fluid drain is con- centric wall thickening greater than 0.5–0.7 cm. This traindicated, because this would probably worsen the represents medial degeneration with hematoma within symptoms. the media, though at a location closer to the adventitia 31. Answer: E compared to the pathology of aortic dissection. The nat- This man has developed acute, delayed paraplegia ural history of IMH is more benign than aortic dissec- following thoracoabdominal aortic replacement that is tion, perhaps due to a lower associated incidence of likely secondary to hypotension caused or exacerbated malperfusion syndromes. Aortic dissection, rupture or by starting a propofol infusion for sedation. Delayed late progression to aneurysm may evolve from IMH. Like paraplegia is a well-known phenomenon that occurs aortic dissection, IMH is categorized as type A (involv- after thoracoabdominal aortic surgery and is often due to ing the ascending aorta) or type B (isolated to the hypotension in a watershed zone of devascularized spi- descending aorta). Type B, as in this case, has a less nal cord. If treated immediately it can often be reversed malignant course. Complete resolution of the hematoma by improving spinal perfusion pressure. This is best has been observed in 50–80% of cases with conservative achieved by a combination of hypertensive treatment management. Predictors of progression include the pres- and lumbar CSF drainage. Vasopressors (e.g. norepi- ence of PAU, older age, aortic diameter greater than nephrine, neosynephrine, or vasopressin) can be used to 4.0–4.5 cm, and hematoma thickness greater than 1 cm. initiate hypertension. Mean arterial pressure (MAP) Nonoperative management of type B IMH is widely should be increased stepwise every few minutes until an accepted. Controversy exists regarding the management improvement in the lower extremity neurologic exam is of type A IMH, with some advocating intervention only noted (initial MAP target pressure is above 90 mmHg). in cases of complication (dissection, expansion, rupture) Several factors are involved with most cases of delayed while others propose a more aggressive approach. If onset paraplegia, and its prognosis is much better than conservative management is chosen, patients require paraplegia noted immediately at the end of aneurysm inpatient observation in the ICU with bed rest for 2 repair. In this case, the spinal (or cerebral) perfusion weeks and vigilant blood pressure control. Repeat imag- pressure was only 41 mmHg (MAP-CVP) when neuro- Answers 1043

logic changes were noted. The propofol infusion should arm as evidenced by the minimal blood pressure gradi- be discontinued immediately and a vasopressor infused ent across both arms at rest. With a greater metabolic to reverse the hypotension. There is already a lumbar demand during upper extremity exercise the blood flow CSF drain in place because the intrathecal pressure is in the left vertebral artery may reverse, a possibility best provided, but a new drain is indicated if there is a ques- documented by cerebral angiogram or time-of-flight tion about its function. The drain should be opened to magnetic resonance angiography. Cerebral angiography drop intrathecal pressure to below 10 mmHg to further facilitates better left upper extremity provocative testing improve spinal perfusion pressure. MR angiography than MRA, but either can be helpful in the diagnosis. might help define a region of infarction, but it is an inap- Carotid studies should also be done, but it is not expected propriate first step in management because treatment that abnormal findings will explain this patient’s would be delayed. Nicardipine is an antihypertensive symptoms. drug, which is contraindicated in this setting. Naloxone Once reversal of vertebral arterial blood flow is dem- (Narcan) has been used in the setting of acute paraplegia onstrated, either carotid-to-subclavian bypass or trans- following thoracoabdominal aortic replacement, but its position is appropriate therapy. Direct left vertebral efficacy is not well-established. Its use as an adjunct arterial bypass, although possible, does not specifically would be secondary to the primary therapies (hyperten- address the phenomenon of steal to the left arm, and it is sive therapy and CSF drainage). more cumbersome than simple revascularization of the 32. Answer: C left subclavian artery. The phenomenon of left subcla- Lower extremity paralysis or weakness due to spinal vian steal is now a recognized complication of left sub- cord ischemia remains a major complication of thora- clavian arterial coverage by thoracic aortic stent grafts. coabdominal aortic aneurysm repair, particularly after 34. Answer: A Crawford extent II repairs. Nearly two-thirds of patients Patients with high-speed deceleration injuries are at in whom paraplegia or paraparesis develops have the risk for aortic transection at the level of the ligamentum deficit immediately upon awakening from anesthesia. arteriosum just beyond the left subclavian artery. The The other third of patients are neurologically intact ini- diagnosis is often made on CT scan but may require aor- tially, but the deficit develops at a median of 3 days after tography or transesophageal echo evaluation to confirm operation. Once the deficit occurs, cerebrospinal fluid the injury. Regardless of how the diagnosis is made, drainage should be initiated as soon as possible, because intra-abdominal and intracranial injuries should be lowering intrathecal pressure has been associated with addressed first. Patients with concomitant injuries to the rapid recovery of motor function. Hypotensive episodes thoracic aorta and abdomen should have exploratory lap- are associated with about one-fourth of cases of delayed arotomy prior to repair of the aortic injury. The aortic paraplegia. Causes of hypotension (arrhythmia, sepsis, injury is usually addressed by open repair through a left hypovolemia, pneumothorax, antihypertensive medica- thoracotomy. Similarly, patients with an epidural hema- tions, etc.) must be identified and treated aggressively, toma should have a craniotomy prior to aortic repair. and vasoactive medications should be administered to Stent-graft repair of traumatic aortic transection may also raise the mean arterial blood pressure to 90–100 mmHg. be an option and may become the treatment of choice in It is also prudent to correct anemia in order to optimize the future. Despite the less invasive nature of stent graft- oxygen delivery. The described patient has an adequate ing, however, it is not applicable to all transections and it blood pressure (estimated MAP = 101 mmHg) and only still requires anticoagulation and considerable time. It mild anemia. Therefore, insertion of a cerebrospinal should not supercede laparotomy or craniotomy. Some fluid drain should be the first intervention in this case. scenarios involve delayed identification of aortic transec- Although thrombosis of intercostal arteries has been tion and a nonoperative approach may be appropriate proposed as a potential inciting event in some patients, include: coexisting severe COPD or cardiac disease; a imaging may be difficult and reoperation to re-establish patient with severe head injuries whose prognosis for spinal cord perfusion is seldom feasible. There is no evi- neurologic recovery is uncertain; patients with sepsis fol- dence that anticoagulation has therapeutic benefit in this lowing intra-abdominal injuries (e.g., colon perforation) situation and it could increase the risks associated with to minimize the risk of graft infection; patients with con- inserting a lumbar drain. comitant burns and those with severe right lung contu- 33. Answer: B sions who may not be able to tolerate single lung Based on the symptoms of exercise-induced syncope ventilation. Aggressive beta blockade and invasive moni- and pre-syncope with a history of previously covered toring for blood pressure control is mandatory in all left subclavian artery (see CT angiogram), vertebrobasi- patients with this injury. Serial imaging of the aortic lar insufficiency due to left subclavian steal is suspected. injury may also be helpful in determining the appropriate The patient has excellent collateral blood flow to the left time to intervene surgically. 1044 Answers

35. Answer: B 37. Answer: C This man has sustained a traumatic aortic transection This patient presented with an acute Stanford type A near the isthmus of the aorta, and there is a distal aortic aortic dissection with intramural hematoma. She is dissection. The extent of the aortic abnormalities are not hypotensive and intrapericardial blood clot is obvious on clear from the limited CT slices. Additional injuries the CT study, which indicates probable rupture. She include chest wall trauma, a femur fracture and a cere- should be taken emergently to the operating room for bral contusion. Although he is hemodynamically stable definitive repair. Stent-grafting the ascending aorta is at the time of evaluation his greatest risk for in-hospital inappropriate therapy for an acute type A dissection. death remains exsanguination from aortic rupture into Replacement of the entire aortic root is probably not the left pleural space. In the setting of concomitant neu- required in this patient because gross dilation of the root rologic injury, optimal contemporary management is is not obvious, and the study suggests that only a small endovascular stent-grafting. The proximal descending portion of the non-coronary sinus is dissected. If, indeed, thoracic aorta is quite conducive to endovascular repair. the other two sinuses are spared, then the goal should be The caliber of this patient’s aorta appears large enough to preserve the native valve. Root repair with valve to accept a thoracic aortic stent graft. Open reconstruc- resuspension in this age group is easier and carries an tion with hypothermic circulatory arrest should be acceptably low risk of late failure. Aortic root replace- avoided in the setting of concomitant brain injury espe- ment with a mechanical valve conduit would address the cially when there is intracranial hemorrhage. problem definitively, but this offers no advantage over Craniectomy is not indicated without signs cerebral bioprosthetic root replacement for a patient of this age. edema or bleeding. Open reconstruction utilizing left A mechanical valve conduit requires long-term antico- heart bypass is an acceptable treatment option, and it agulation, and the 15- to 20-year survival after type A would be optimal­ treatment for a small-caliber (<17 mm) dissection repair in this age group does not justify the aorta. However, in the setting of multiple other injuries added challenges of warfarin management. Complete endovascular stent-grafting offers more timely therapy reconstruction of the entire involved aorta is seldom at a lower operative risk to this patient. possible since most ascending dissections propagate and 36. Answer: C have fenestrations distally in the descending aorta. This The described situation is likely due to dynamic patient’s CT study shows no descending dissection, so obstruction of one or more of the brachiocephalic arter- resection of the entire ascending aorta and involved arch ies by the dissection flap. Either cannulation of the false is feasible. Total aortic arch replacement is not neces- lumen in the groin or a re-entry tear between the femoral sary in most acute type A dissections, and this therapy artery and the aortic arch can cause excessive pressure in alone does not address the primary goals of type A dis- the false lumen near the arch. The result can be obstruc- section repair: restoration of aortic valve competency, tion of blood flow into the true lumen of the arch branch resection of the primary tear site, restoration of true vessels. Devastating neurologic injury from inadequate lumen blood flow, and exclusion of the false lumen. flow will follow if hypoperfusion is not immediately 38. Answer: C addressed. Applying a cross-clamp could worsen this Normal atrioventricular (A-V) conduction directs the situation and would certainly not improve it. Rewarming electrical impulse from the SA node though the AV node the patient reverses an important aspect of cerebral pro- to the His bundle and down the Purkinje cells. The A-V tection and further delays definitive therapy for the prox- node creates an electrical delay prior to the impulse imal aortic dissection. Recannulation through the left entering the His bundle. The central fibrous body nor- femoral artery may reconstitute true lumen flow if the mally acts as a functional electrical insulator from problem was due to false lumen cannulations. It will not abnormal A-V conduction. If there is an accessory A-V address obstructive flap dynamics associated with retro- conduction connection (Kent bundle), then more rapid grade perfusion. Cannulation of the right subclavian antegrade conduction down this pathway may occur. artery is most likely to restore cerebral perfusion under This pre-excitation translates to a short P-R interval and the described circumstances. Central aortic cannulation a delta wave noted on the ECG. These features are of the true lumen, if possible, would accomplish the apparent in the augmented figure. This patient has Wolff-­ same goal, but identification of undissected ascending Parkinson-­White (WPW) syndrome. Reciprocating aorta is seldom possible. Attempting to expedite reach- tachycardia can develop if an accessory pathway func- ing deep hypothermia using excessive cooling gradients tions as part of a re-entrant conduction loop. Either the can potentiate cerebral vasoconstriction and non-­ Kent bundle or the normal A-V node can conduct these uniform cooling of the brain, which may actually worsen impulses, but each can function in one direction only. If prospects for cerebral protection. Without assured cere- A-V conduction is normal and the impulse returns via bral flow, this strategy is folly. the accessory bundle it is termed orthodromic. If ante- Answers 1045

grade conduction is via the accessory pathway and re-­ globulin, ATG) agents is used as induction therapy or entry is backward through the A-V node, it is antidromic. salvage therapy for acute rejection episodes, but not for Rapid tachycardia can occur in either case, with the maintenance immunosuppression. upper rate limited only by the refractory periods of the 41. Answer: C pathways. The ventricular response may be fast enough This patient now has strong evidence for endocardi- to be inefficient and cause low cardiac output symptoms tis, and the constellation of symptoms and findings on or sudden death. Development of atrial fibrillation can admission was consistent with a diagnosis of bacterial certainly be dangerous. endocarditis (positive for two to three major criteria WPW and other accessory pathway-mediated tachy- and at least two minor criteria of the Duke Endocarditis cardias were once surgical diseases, but now they are Service). The involvement of the surgeons was late in best treated by percutaneous mapping and ablation of this patient’s course and his prognosis has worsened the accessory pathway. A MAZE procedure or unmapped with renal failure and ongoing congestive heart failure. left atrial ablation will not offer the requisite precise The chest film shows pulmonary edema with a normal identification or destruction of this patient’s accessory heart size, which strongly suggests acute, severe aortic pathway. Since the A-V node is not the abnormality, regurgitation. No matter what complications have node ablation and permanent pacing is not appropriate. developed or how much decompensation has pro- Most patients with WPW syndrome have few symptoms gressed, aortic valve endocarditis with severe regurgi- and chronic drug therapy is undesirable and not reliably tation and heart failure must be repaired or mortality is effective. A primary challenge is to determine when assured. The source(s) of infection must be identified ablation is warranted. and eradicated, and hemodynamics must be corrected 39. Answer: D by replacing the valve. Emergency aortic valve replace- In most cardiac transplant centers immunosuppres- ment (with root debridement) is appropriate in this set- sion after heart transplantation consists of a calcineurin ting, but operation should be preceded by a CT scan of inhibitor (tacrolimus or cyclosporin) and an adjunct the brain and abdomen. Twenty-four to sixty-seven immunosuppressant with or without maintenance corti- percent of patients with endocarditis have systemic costeroids. The nephrotoxicity of calcineurin inhibitors embolization, over half of which involve the brain. An is well established and in the face of declining renal active infarct or abscess requires reconsideration of function this class of drugs should be discontinued and treatment strategies and timing. Five to nineteen per- the renal function followed closely. cent of emboli occur to spleen, and either preliminary Other immunosuppressants can be maintained. or concomitant splenectomy is prudent if a splenic Supportive care should concentrate on maximizing car- abscess is found. Echocardiography is critical to the diac output at modest preload. Atrial pacing and/or isu- diagnosis of endocarditis, and the size of vegetations, proterenol infusion should achieve a heart rate of 110, local extension, and severity of aortic valve regurgita- and diuretics should be used to maintain brisk urine out- tion are factors that influence the likelihood of medical put. Left and right heart afterload can be independently therapy success for native valve infection. Renal fail- managed to maintain systemic perfusion yet minimize ure in patients who have endocarditis is often related to pulmonary pressures. This patient’s tacrolimus can be immunologic phenomena (glomerulonephritis) with restarted when creatinine is returning to baseline. contribution from emboli, sepsis, and hemodynamic 40. Answer: C abnormalities. Standard maintenance triple therapy following car- 42. Answer: C diac transplantation includes one drug from each of the ACC/AHA guidelines are classified both by the following classes: strength of recommendation and the quality of evidence 1. Calcineurin inhibitors: cyclosporine, tacrolimus supporting the recommendation. Class I recommenda- 2. Corticosteroids: prednisone, prednisolone tions are those for which “there is evidence for or gen- 3. Antimetabolites: azathioprine, mycophenolate mofetil eral agreement that the treatment is beneficial, useful, The use of rapamycin (Sirolimus) is being introduced and effective.” Class II recommendations are those for into cardiac transplantation. It is used more routinely in which there is conflicting evidence or a divergence of abdominal organ transplantation. It targets preventing opinion about the efficacy of a treatment. If the weight the actions of a key kinase called target of rapamycin of opinion is in favor of the treatment these recommen- (TOR). There are concerns that wound healing compli- dations are graded Class IIa. Class IIb recommendations cations may increase when rapamycin is given early are those for which treatment efficacy is less well estab- after sternotomy. lished. The internet link supplied outlines current guide- Antibody therapy with monoclonal (OKT3) or poly- lines for care of patients with valvular heart disease. The clonal (antilymphocyte globulin, ALG or antithymocyte timing of surgery is a key part of optimizing surgical 1046 Answers

management of endocarditis. Patients should be taken to 4. establish what/how much the patient and/or family the operating room regardless of the duration of antimi- want to know crobial therapy when there is an indication for surgery. 5. deliver information in a sensitive, straightforward Current ACC/AHA class I indications for surgery in manner, avoiding technical language and native valve endocarditis include: euphemisms 1. any acute valve lesion resulting in heart failure 6. check for understanding and clarify concepts and 2. aortic or mitral regurgitation with hemodynamic evi- terms dence of elevated left ventricular end-diastolic or left 7. respond by using active listening, encouraging atrial pressures or moderate pulmonary expression of emotions hypertension 8. organize an immediate therapeutic plan that addresses 3. structural complications such as annular or aortic the patient’s and family’s concerns and agenda abscess, or other destructive penetrating lesions 9. reassess understanding of the condition and treat- including fistulae and “kissing lesions” ment plan and determine the need for further educa- 4. fungal or other highly resistant organisms tion and follow-up Native valve endocarditis complicated by recurrent 44. Answer: E emboli and persistent vegetations despite appropriate Criteria for catheter intervention for mitral stenosis antibiotic therapy is a class IIa indication for surgery, are based on symptoms, valve characteristics and pul- and the presence of mobile vegetations in excess of monary artery pressures. A symptomatic patient (NYHA 10 mm with or without emboli is classed by the ACC/ class II or worse) must have favorable valve morphology AHA guidelines as a class IIb indication for surgery. In and moderate or severe mitral stenosis and/or PA sys- this patient, heart failure with pulmonary edema is an tolic pressure >50 mmHg at rest (or >60 mmHg with urgent indication for surgery. Other indications exist, exercise to satisfy ACC/AHA Class I indications for per- but they do not justify emergent operation: anterior cutaneous mitral balloon valvotomy/valvuloplasty. leaflet perforation will eventually need surgery; a Favorable valve morphology includes non-calcified, pli- mobile vegetation without embolization is likely to be able mitral valve leaflets. Mitral valve morphology is watched in an elderly patient; Streptococcus bovis is a most important determinant of immediate post-­ sensitive organism which can often be eradicated by valvotomy hemodynamic results and long-term clinical antibiotic therapy; and an elevated WBC at this point outcomes. Patients with valvular calcification, thickened in his treatment does not constitute uncontrolled and fibrotic leaflets with decreased mobility, and subval- sepsis. vular fusion have a higher incidence of acute complica- 43. Answer: A tions and a higher rate of recurrent stenosis on follow-up. Although we have the technology to perform com- Therefore, marked fusion of the leaflets and subvalvular plex operations, one must weigh the expected outcome apparatus and severe commissural calcification are rela- against the risk. Elderly patients (over age 75) may be tive contraindication to percutaneous balloon mitral val- able to tolerate a major operation under ideal conditions, votomy. The Wilkins scoring system grades mitral valve but the addition of major comorbidities makes the morphology using four echocardiographic characteris- chance of success far less likely. Many patients and their tics: leaflet mobility, leaflet thickening, valve calcifica- families anticipate only successful outcomes and do not tion and involvement of the subvalvular apparatus. Each understand the significance of survival with major com- parameter is graded from 0 to 4. A total score of more plications. The patient presented here has an overwhelm- than 8 predicts a low success with percutaneous mitral ing chance for death regardless of the therapeutic valvuloplasty. Other relative contraindications to this intervention. procedure include left atrial thrombus and mitral regur- Breaking bad news to families is not easy for a sur- gitation that is worse than mild. The thresholds for mitral geon. Many hospitals have resources including Ethics valve surgery for stenosis have historically been more Committees available to help in such situations. Nine conservative than for regurgitation. The potential for general points for consideration are suggested: durable repair is much lower, and MVR is generally 1. prepare for discussion by ensuring all information/ reserved for symptomatic patients or those with severe facts/data are available pulmonary hypertension. The described patient has 2. deliver an analysis in person in a private area without NYHA class II symptoms and moderate mitral stenosis. interruptions Scoring contraindicates percutaneous balloon mitral 3. establish patient and/or family knowledge and under- valvotomy, and conventional recommendation is medi- standing of the illness cal management with follow-up in 6 months. Answers 1047

45. Answer: A minimize the risk of SAM. At this point, however, it Mitral regurgitation secondary to myocardial infarc- would be necessary to remove the ring and a longer tion is usually secondary to ventricular dysfunction and period of cross clamping and bypass would be regional wall motion abnormalities. Occasionally the required. Mitral valve replacement is usually not nec- cause is rupture of a papillary muscle. In general, essary and it should be avoided in this young woman. patients who require urgent or emergent operation for Repair is better because valve replacement would mitral regurgitation in the setting of acute myocardial shorten her life expectancy, increase the lifetime risk infarction are at high risk for death and morbidity. The of stroke and potentially complicate childbirth. One risks might be lower with papillary muscle rupture effective solution is to shorten the anterior leaflet. because mechanical dysfunction of the valve contributes This is achieved by making a transverse incision to the regurgitation in addition to the mechanical dys- toward the base of the anterior leaflet and re-suturing function of the ventricle. However, this is not a proven (which will shorten the leaflet by 5–7 mm) or to resect assertion. a 2–3 mm transverse ellipse of the leaflet before re- Concurrent revascularization lowers perioperative suturing (which will shorten the leaflet by 7–10 mm). mortality and all critical abnormalities should be A triangular resection of A2 is another option to addressed at the time of operation. Residual mitral shorten its length. Another option is to use a shorter regurgitation after repair generally will not improve with PTFE chord to lower the margin of the posterior leaf- expectancy or with mechanical support. In the case of let further down into the ventricle, moving the closure complete papillary muscle rupture, many surgeons rec- line more posteriorly and aligning the anterior leaflet ommend immediate chordal sparing valve replacement. away from the outflow tract. Finally, a recently pro- The size of the left ventricle, patient age, ventricular posed technique of posterior leaflet shortening by function, preoperative atrial fibrillation, and the prefer- pledgeted sutures through the leaflet and under the ences of surgeon and patient should all be considered annuloplasty ring has promise as an equivalent to when determining the type of prosthesis used. sliding plasty. 46. Answer: A 47. Answer: C Tools available to treat mitral systolic anterior Mitral valve repair offers superior long-term survival, motion include discontinuation of inotropic agents, freedom from cardiac morbidity, and better quality of additional volume, added alpha agents and, possibly, life in most patients with severe mitral regurgitation β-blockade. When these maneuvers fail to reverse compared to medical management and mitral valve SAM the cause is usually a discrepancy between the replacement. It is the treatment of choice in these residual anterior and posterior leaflet heights and the patients because it is the only treatment that restores septal-lateral diameter of the mitral annulus following normal life expectancy to otherwise healthy patients ring annuloplasty. The type of ring annuloplasty does with advanced degenerative mitral valve disease. not influence the risk of SAM, and this condition has Factors determining timing of surgery for isolated been reported with essentially all types of annulo- mitral regurgitation in the current ACC/AHA guide- plasty rings (flexible, rigid, semi-rigid rings and par- lines include symptoms, left ventricular ejection frac- tial or complete designs). This complication can occur tion (≤60%), left ventricular end-systolic dimension in the absence of an annuloplasty ring, so changing the (≥40 mm), atrial fibrillation, and pulmonary hyperten- ring to an incomplete band is unlikely to resolve sion (PA systolic >50 mmHg at rest or >60 with exer- SAM. Sizing the ring is a more critical issue, but since cise). While there is a consensus that patients in whom a 40 mm ring is the largest commercially available mitral repair is feasible should undergo surgery before size, a larger ring is not an option in the described left ventricular decompensation is documented, the case. Removing the ring could resolve the SAM, but poorer outcomes associated with mitral replacement absence of a ring in degenerative repair can predispose mean that asymptomatic patients with severe mitral to recurrent mitral regurgitation in the long term. The regurgitation but preserved left ventricular function likely cause of SAM in this case is excess residual should not undergo surgery unless a greater than 90% posterior leaflet tissue. Although the posterior leaflet successful repair rate is likely. Mechanical replace- is tall, the repair was undertaken with minimal resec- ment should not be contemplated for the described tion techniques (triangular resection and artificial patient. Successful repair and maintenance of normal chordae). The residual posterior leaflet height is likely sinus rhythm will obviate the need for warfarin, so the displacing the anterior leaflet into the left ventricular acceptability of this drug is not a relevant factor in the outflow tract. A sliding posterior leaflet plasty would timing of surgery. Patients of all ages, including the have been appropriate during the initial operation to elderly, are likely to benefit from mitral valve repair, 1048 Answers

and age is not an important consideration in the utility toma or arterial bleeding from the back of the heart after of surgery in this patient. The key issue, therefore, in mitral valve surgery can represent a highly lethal atrio- deciding to operate on this patient is the likelihood of ventricular dissociation. Patients at risk for this typi- achieving a durable repair. The patient has Barlow’s cally have severe mitral annular calcification that may disease. Echocardiogram in this condition shows a require aggressive debridement prior to mitral valve mitral valve with markedly excess tissue involving replacement. As a general rule, the heart should not be multiple leaflet segments in an otherwise large valve. lifted after mitral valve operations. Minor posterior Leaflet tissue is thickened and redundant with elon- bleeding should be managed expectantly. If there is sig- gated chordae. Regurgitation is due to bi-leaflet, multi- nificant arterial bleeding, however, then repair is man- segment prolapse. Obtaining a competent and durable datory. The problem must be identified and it is likely repair is more challenging in Barlow’s disease, where that a complex repair is required. Bypass must be tissue resection, sometimes of multiple leaflet seg- resumed immediately. For internal repair, following ments, and chordal reconstruction are mainstays of cardioplegia the valve must be inspected, the ring or repair. Although several isolated series report repair prosthesis should be removed and a generous pericar- rates for degenerative disease of 90%, repair rates in dial patch repair may be required to obliterate the annu- developed countries continue to average between 50% lar defect from within the heart. Various external patch and 70%. Echocardiographic assessment is key in and glue techniques have been reported for isolated matching surgical expertise to valve complexity. The cases. Fortunately, this complication follows only 1–2% probability of successful repair of an isolated P2 pro- of MVR procedures and it is rare following mitral valve lapse secondary to chordal rupture due to fibroelastic repair. Individual surgeon or institutional experiences deficiency is high in most centers. However, even an are small. As may be expected when numerous methods experienced mitral repair surgeon may lack sufficient of repair are described, results are generally poor. experience with Barlow's­ disease to offer this patient Attempts at external direct repair on a loaded, beating with complex bi-leaflet prolapse a >90% probability of heart almost always fail. With a prosthesis in place the repair. Analysis of valve repairability based on the spe- exposure necessary to place pledgeted sutures will fur- cific echocardiographic lesions identified is the single ther disrupt the annulus, worsen bleeding, waste time most important factor in deciding whether to proceed and make successful definitive repair even more diffi- with early surgery an asymptomatic patient. cult. The isolated hematoma with modest bleeding 48. Answer: E described suggests a less catastrophic complication, but The right mini-thoracotomy approach is a versatile coronary sinus injuries associated with retrograde car- incision for exposure to the mitral valve. Success is dioplegia techniques can also be difficult to manage. predicated upon free access through the anterior right Risks for coronary sinus injuries include hypertrophied hemithorax and a lateral approach to the inter-atrial in elderly patients with friable tissue. If encoun- groove. Pre-existing obliteration of the pleural space due tered at the start of the operation, the cannula should be to adhesions from prior right chest surgery or pleurode- removed and an alternative myocardial protection plan sis is a relative contraindication. While peripheral vascu- implemented. Repair can be with an onlay patch or by lar disease is a contraindication to retrograde arterial direct suture repair. Direct repair can result in a coro- perfusion, both ascending aorta and axillary artery are nary sinus stricture, which may result in thrombosis and possible perfusion inflow sites with the right anterior a potentially lethal outcome. Following a mitral inter- mini-thoracotomy approach. Pectus excavatum, with the vention these injuries do not lend themselves to isola- attendant leftward shift of the heart can make reaching tion or bypass grafting. If the hematoma is stable the mitral valve exceedingly difficult if not impossible. following protamine administration, then further Pectus carinatum (“pigeon breast”) is not a problem. manipulation of the area should be avoided since the Previous CABG can make a right-side approach prefer- low pressure venous system will typically stabilize and able to repeat sternotomy due to the avoidance of patent heal without intervention. Exploring a stable hematoma grafts and other adherent mediastinal structures. in such a situation can result in uncontrolled bleeding Concomitant tricuspid valve procedures are easily per- and potentially fatal ventricular injuries. formed through this approach. 50. Answer: B 49. Answer: A The presence of patent coronary artery bypass grafts Decisions regarding the management of significant is a potential hazard to reoperative surgery, but inadver- posterior bleeding and/or hematoma after a cardiac tent injury to the grafts can be minimized by careful pre- operation can be difficult. Inappropriate or incorrect operative planning and intra-operative care. Pulmonary decisions can have catastrophic consequences. A hema- hypertension and an elevated CVP are common in Answers 1049

patients with advanced heart failure and do not signifi- this level of end stage heart failure will benefit from cantly impact decision making for this patient. CRT. Conversely, a patient with an EF of 40% likely has Historically, poor ventricular function was associated diastolic heart failure from hypertension and conven- with prohibitive risk for correction of mitral regurgita- tional medical therapy is indicated. Third degree heart tion, as repair or replacement removes the low resistance block is an indication for A-V sequential pacing, but a regurgitant pathway and might worsen heart function. narrow QRS complex is outside indications for biven- However, the safety and potential benefits of mitral tricular pacing—a normal A-V pacemaker is sufficient. valve repair (or replacement with preservation of all 52. Answer: A subvalvular apparatus) in this setting are becoming con- Catheterization hemodynamics in pericardial con- vincing. Increasing emphasis is being placed on ven- striction reveal three key features: (1) elevation and tricular indices other than ejection fraction to predict equalization of diastolic pressures in each of the car- patient outcomes following correction of mitral regurgi- diac chambers; (2) early diastolic “dip-and-plateau” or tation in the setting of advanced heart failure. A recent “square root-sign” configurations in the right and left review of outcomes in patients undergoing combined ventricular tracings; and (3) a prominent y-descent in CABG and mitral valve annuloplasty for ischemic MR the right atrial pressure tracing. The constrictive effect suggests that a preoperative left ventricular end-diastolic of the thickened pericardium results in elevated, equal- diameter (LVEDD) >6.5 cm is associated with reduced ized diastolic pressures all cardiac chambers. Early reverse remodeling and elevated early and late postop- diastolic filling is unimpeded and elevated venous erative mortality. Similarly, a left ventricular end-sys- pressures lead to abnormally rapid filling, while ven- tolic volume index (LVESVI) >150 mL/m2 predicts the tricular pressure decreases during this time. This rapid same bad outcomes. Reconsidering patient candidacy early diastolic filling then abruptly stops and ventricu- for surgery with these threshold values rather than ejec- lar pressure increases abruptly as intracardiac volumes tion fraction can provide symptomatic benefit to many approach the volume limit set by the noncompliant, patients. Additional experience is required to determine thickened pericardium. The dip-and-plateau pattern the optimal combined management plan to improve (a.k.a. square root sign) of the ventricular waveform mortality in this difficult population. reflects rapid filling and arrested volume accumulation 51. Answer: E respectively. Equalized cardiac diastolic pressures and Cardiac resynchronization therapy (CRT or biven- volumes translate to limited stroke volume and cardiac tricular pacing) is an important tool in the management output. Increased sensitivity and specificity of hemo- of systolic congestive heart failure. The concept is that dynamic testing can be augmented by provocative with simultaneous pacing of right and left ventricles, maneuvers. Intracardiac pressure dissociation is a phe- contraction is better coordinated and ejection is nomenon whereby inspiration leads to increased RV improved. The timing of the right and left ventricular inflow and intraventricular septal shifting to the left, contractions can be adjusted to optimize “resynchro- which reduce left ventricular filling. This creates a pre- nized” contraction. Several clinical trials demonstrated load mismatch between the left and right sides of the clinical improvement when selected patients with sys- heart, with elevated preload on the right compared to tolic ventricular dysfunction and heart failure were the left. Simultaneous right- and left-­sided ventricular treated with CRT. Inclusion criteria included cardiomy- pressure tracings become discordant and begin to sepa- opathy (ischemic or nonischemic), NYHA functional rate as a consequence, with LV pressures falling while class III or IV despite maximal medical therapy (but not RV pressures increase. The opposite occurs during on continuous IV therapy), left ventricular ejection frac- expiration. This discordant response does not occur tion ≤35%, QRS duration ≥120 ms (bundle branch with restrictive disorders or congestive heart failure. block), and sinus rhythm. Using these “standard” selec- Examination of the right atrial wave forms shows an tion criteria 60–70% of patients benefited symptomati- exaggerated and deep y-descent compared to the cally from biventricular pacing. Further studies have x-descent, forming a characteristic M or W configura- been completed outside these guidelines for patients in tion. This is in contrast to cardiac tamponade in which atrial fibrillation. Studies on heart failure patients with the y-descent is attenuated or up sloping. Cardiac tam- chronic atrial fibrillation are immature, but functional ponade is the result of generalized cardiac compression benefit from biventricular pacing is less impressive than throughout diastole. As such the diastolic surge that is for patients in sinus rhythm. In the examples given, atrial seen in a constrictive pericardial patient is absent. fibrillation is a relative exclusion without additional Atrial filling occurs during ventricular ejection in early information. Patients on chronic inotropes were not can- and the venous pressure tracing shows an didates for the CRT trials, and there is no evidence that absent or blunted diastolic y-descent. 1050 Answers

53. Answer: E 55. Answer: E Right ventricular wall thickness increases in response This patient’s ECG shows sinus tachycardia, but there to volume and pressure overload, but in normal patients are no acute injury patterns. Cardiac tamponade after sur- the normal range is 2.5 ± 0.5 mm. Normal pericardium gery is a potentially lethal complication if not diagnosed and is ≤2.0 mm thick. Pericardial resection for constrictive treated in a timely manner. The diagnosis is based on com- pericarditis can be fraught with technical difficulties. plete clinical assessment. This patient initially had excellent Dense adhesions between pericardium and epicardium hemodynamic parameters, but initial brisk bleeding was develop, which can obscure identification of proper tis- documented. A significant drop in cardiac output associated sue planes. Common causes of pericarditis include past with a new vasopressor requirement is alarming. surgery, irradiation, past myocardial infarction, and Postoperative fluid resuscitation is commonly required, but infection. Consequently pericardial thickness is this patient’s elevated central venous pressure suggests that increased, the epicardium may not be identifiable, and hypovolemia is not the cause of current low output. Acute the underlying myocardium may also be inflamed and graft closure would result in infarction or ischemia and con- friable. Significant raw-surface bleeding should be sequent congestive heart failure, but the ECG shows only anticipated during the course of dissection, but a sudden sinus tachycardia and nonspecific ST changes. A follow-up rush of dark blood over the right heart suggests right chest film would probably show a widened mediastinal sil- ventricular entry. Attempts at primary repair in a flooded houette, but this finding is nonspecific. A precordial echo- field that is incompletely mobilized may tear and avulse cardiogram would likely show clot or fluid (blood) additional tissue and worsen the injury. In the described collection, but a delay might be introduced and the quality situation initiation of cardiopulmonary bypass is war- of the study might be poor. In this setting the blood pressure ranted to achieve a decompressed, bloodless field in and cardiac output changes are due to cardiac tamponade which to identify optimal dissection planes, repair the until proven otherwise. Clinical and hemodynamic findings laceration, and complete the pericardial resection. are often atypical, the echocardiographic findings are clear Abandoning the procedure may be appropriate if bypass in only about 60% of cases, and the risk of re-exploration is is not available, but only complete pericardiectomy will minimal. This patient should be returned to the operating resolve the patient’s symptoms. room immediately for evacuation of retained blood and clot. 54. Answer: D 56. Answer: E The chest film shows a globular heart shadow with The figure shows a small pericardial effusion posteri- minimal widening of the upper mediastinum. The echo- orly and around the right atrium in addition to lung cardiogram shows a large posteroinferior pericardial effu- parenchymal infiltrates and volume loss in both lower sion with compression of the right-sided (anterior) heart lobes. In routine thoracic imaging, multifocal aneu- structures. The right ventricle is very small and com- rysms, unexplained cardiomyopathy, and venous throm- pressed, which is opposite the expected findings for right bosis in high-risk patients should suggest the possibility heart strain from massive pulmonary embolism. There is of AIDS. The incidence of pericardial effusions in HIV-­ no evidence of pericardial constriction. Although no color infected patients is estimated at about 5%, but most are Doppler flow images of aortic valve function are shown, small and asymptomatic. In this population effusions the large pericardial effusion dominates the findings and can be manifestations of many infectious causes, tamponade explains the clinical presentation. The fluid is although no single pathogen predominates. Malignancy most likely the result of breakdown of pericardial clot and is responsible for some effusions, but most often no osmotic volume expansion rather than pericarditis. Late cause can be identified and progression to symptoms is cardiac tamponade is defined as tamponade occurring uncommon. Several older studies demonstrated short- more than 7 days after cardiac surgery. This complication ened survival of AIDS patients once a pericardial effu- occurs in less than 1% of patients following cardiac oper- sion was diagnosed, suggesting a more advanced stage ations, and this relative infrequency can delay diagnosis of the disease. HAART (highly active anti-retroviral and treatment. It develops more common in women and therapy), which includes protease and nucleoside in patients taking warfarin and/or antiplatelet medica- reverse-transcriptase inhibitors, was introduced in the tions. Patients can present with rapidly worsening dys- mid-1990s. HAART therapy has evolved with variable pnea on exertion and exercise intolerance that can progress composition, but it has no known therapeutic role for to hypotension. The best treatment is immediate admis- pericardial disease. The effects of wide use of these regi- sion and reoperation or catheter drainage to decompress mens on the incidence of effusive pericardial disease are the pericardial space. Opening the inferior aspect of the not known, but effusions remain associated with sternotomy to place a drain will rapidly resolve this advanced HIV infection. In the absence of symptoms condition. pericardial drainage is not indicated. Answers 1051

57. Answer: D 59. Answer: E A CT-guided biopsy of the lung mass can establish a The chest CT shows a “saddle” pulmonary embolus diagnosis of the nodule, but no additional staging infor- with bilateral clot. Obstruction to blood flow and cyto- mation is gained. Documentation of a malignant pericar- kines induce pulmonary vasoconstriction, which leads dial effusion or intrapericardial metastasis confirms to marked acute changes in right heart afterload. The stage IV disease, which defines definitive chemotherapy right heart, a thin walled and relatively weak muscle as indicated treatment. The most common malignancies compared to the left ventricle, is incapable of acute com- that involve the pericardium are lung cancer, breast can- pensation. The RV distends, and secondary dilation of cer, malignant melanoma, lymphoma and leukemia. the tricuspid annulus in the anterior-posterior region Chylous pericardial effusions are rare, but they may be results in tricuspid valve regurgitation (TR). TR exacer- seen with extensive mediastinal disease from lymphoma, bates the right heart’s efficiency and further decreases leukemia or small cell carcinoma. Malignant pericardial forward cardiac output, leaving the left heart underfilled. effusion often presents with tamponade, but true con- Matching of ventilation and perfusion is grossly abnor- striction is rarely caused by metastatic disease. mal, and the aggregate abnormalities result in hypoxia, Pericardiocentesis is an appropriate first therapeutic respiratory distress, and hypotension. This patient likely maneuver and cytology samples may confirm malig- had a smaller PE a few days earlier and when preload nancy. Approximately 40% of malignant effusion taps, was decreased by nitroglycerine he became hypotensive however, are non-diagnostic and the effusion recurrence and was misdiagnosed with cardiac ischemia. rate is high. Pericardial biopsy doubles the diagnostic The treatment of pulmonary embolism is evolving. accuracy and improves drainage. The simplest access is While smaller emboli may be treated with hepariniza- the subxiphoid approach. VATS pericardial window is tion, massive PE as in this case carries a grave progno- an ­excellent option for patients with simultaneous peri- sis. While lytic therapy is an option for this patient, the cardial and pleural effusions, and biopsies of each space risk of severe bleeding is high. Pulmonary can be done either by thoracotomy or VATS. These has been shown to lower this mortality compared to tra- approaches are associated with more pain than catheter ditional therapy. John Gibbon first conceptualized the drainage or subxiphoid window. Percutaneous balloon heart lung machine after watching a patient die from an pericardiotomy can also create a pleural-pericardial attempted pulmonary embolectomy (the Trendelenburg communication. This procedure has been shown to be procedure) by Dr. Edward Churchill at the Massachusetts safe and effective for pericardial drainage, but biopsy is General Hospital. While the first successful operation not possible and the pleural space fluid must then be performed on the heart lung machine was for an atrial addressed. Formal pericardiectomy is prudent only for septal defect (Thomas Jefferson University, 1953), the documented pericardial constrictive disease, which fea- quest for better treatment of massive pulmonary embo- tures minimal pleural fluid and characteristic findings on lism led to the machine’s genesis. CT, echo and right heart catheterization. 60. Answer: D 58. Answer: E The haemodynamic distinction between organic and Asymptomatic pericardial effusion with pericarditis functional tricuspid regurgitation can be difficult. is a common finding during the acute phase of radiation-­ Generally, if the patient with severe tricuspid regurgita- induced pericardial disease. Delayed pericarditis with or tion has a right ventricular systolic pressure above without effusion may present months to years following 60 mmHg, then the tricuspid regurgitation is functional radiation therapy. Effusive-constrictive pericarditis and the result of left-sided heart failure. However, if the should be considered in patients who present with a right ventricular systolic pressure is 40 mmHg, there is a pericardial effusion and thickened pericardium. These substantial organic component. The pressure tracings patients typically present 1–4 years after radiation ther- show mild pulmonary hypertension (PA sys- apy with tamponade-like symptoms. Drainage fails to tolic = 45 mmHg) and a right atrial v-wave spike to relieve symptoms because residual constrictive physiol- 23 mmHg) documents severe tricuspid regurgitation. ogy persists. Right heart catheterization with pressure Organic tricuspid disease is suspected, but this does not tracing analysis is the most sensitive and selective diag- explain the complaint of shortness of breath. Rheumatic nostic test. Conservative or expectant care will provide disease affecting both mitral and tricuspid valves is a no improvement. Diuresis will probably worsen symp- likely explanation. Both pathologies should be toms. A CT scan or exercise testing will confirm abnor- addressed, but mitral repair for rheumatic disease is not malities but no novel therapeutic suggestions are likely predictably durable. Of the choices offered and because to result. When effusive-constrictive disease is identified of her age, replacement of both A-V valves with bio- pericardiectomy should be performed promptly. prostheses is best. 1052 Answers

61. Answer: C when there is pulmonary hypertension or tricuspid Midgut carcinoid tumors may secrete annular dilatation (Level of Evidence C) 5-­hydroxytryptamine (serotonin), tachykinins, and pros- Class III: taglandins. The liver effectively inactivates these com- 1. Tricuspid valve replacement or annuloplasty is not pounds from the portal circulation, but when disease indicated in asymptomatic patients with TR whose advances to include hepatic or distant metastases the pulmonary artery systolic pressure is less than vasoactive substances produced are able to reach the 60 mmHg in the presence of a normal MV. (Level of systemic circulation. Clinically, this is characterized by Evidence C) flushing, diarrhea, and bronchospasm. It is believed that 2. Tricuspid valve replacement or annuloplasty is not these same substances induce fibrous tissue plaques on indicated in patients with mild primary TR (Level of the endocardial surfaces of the heart, predominantly on Evidence C) the right side (15% have left side involvement). Typical The described patient has peripheral edema and tricuspid involvement includes thickened and fused pulmonary hypertension, so a repair is the best option. commissures and chordae tendineae. Similarly, tricuspid The treatment of tricuspid regurgitation is in evolu- and pulmonary valve leaflets are thickened and retracted, tion. Dreyfus and colleagues performed a prospective resulting in combined stenosis and regurgitation. Up to study in over 300 patients to determine if early repair 70% of patients with carcinoid syndrome have evidence for tricuspid dilatation alone would lead to long-term of carcinoid heart disease with peripheral signs of right benefits. Nearly half the patients who did not receive heart failure. a repair had TR increases of two grades or more over When symptoms are severe or if right ventricular a 12 year follow-up period. Only 2% of patients who dysfunction is demonstrated, valve replacement offers underwent repair had similar progression of TR. Other marked improvement and a possible survival benefit. studies have implicated persistent TR as an indepen- Repair of such complex involvement, however, is unreli- dent risk factor for worse mid-term outcomes after able. Previous reports favored use of mechanical pros- cardiac surgery. Despite this and other emerging evi- theses because of potential early failure of bioprostheses dence, the guidelines are limited, as listed above. from carcinoid plaque formation. However, the use of With information from additional clinical studies, warfarin in the face of liver dysfunction poses an ele- repair of the tricuspid likely will be more frequently vated bleeding risk and some patients will undergo sub- indicated. sequent hepatic resection, hepatic artery embolization or 63. Answer: C other surgery. Treating symptoms with somatostatin The signs and symptoms of tricuspid regurgitation analogues (octreotide) seems to minimize plaque forma- and stenosis overlap. However, there are distinct char- tion on tissue valves, and placing a bioprosthesis elimi- acteristics of each. Tricuspid regurgitation (TR) is nates the risk of mechanical valve thrombosis. Although often due to left-sided heart failure and evolves with no large comparative series of different valve types is symptoms of fatigue and weakness that result from available, the use of tissue valves for carcinoid heart dis- reduced cardiac output. Right-sided heart failure leads ease is presently favored. to ascites, congestive hepatosplenomegaly, a pulsatile 62. Answer: C liver, pleural effusions, and peripheral edema. In the Current guidelines for treating tricuspid regurgitation late stages, these patients are wasted with cachexia, include: cyanosis and jaundice. Impressive jugular venous dis- Class I: tention with an s-wave or fused c- and v-waves, fol- Severe TR in patients with MV disease requiring MV lowed by a prominent y-descent, is present. During surgery (level of evidence B) inspiration, this finding is accentuated because of the Class IIa: physiologic increase in venous return. Not surpris- 1. Tricuspid valve replacement or annuloplasty is rea- ingly, atrial fibrillation is common. Auscultation of the sonable for severe primary TR when symptomatic heart is notable for an S3 that increases with inspira- (level of evidence C) tion and decreases with a Valsalva maneuver, an 2. Tricuspid valve replacement is reasonable for severe increased P2 if pulmonary hypertension has devel- TR secondary to diseased/abnormal tricuspid valve oped, and a parasternal pansystolic murmur that leaflets not amenable to annuloplasty or repair. (Level increases with inspiration. The chest radiograph dem- of Evidence C) onstrates cardiomegaly, increased size of the right Class IIb: atrium and ventricle, a prominent azygos vein, possi- Tricuspid annuloplasty may be considered for less ble pleural effusion and upward diaphragmatic dis- than severe TR in patients undergoing MV surgery placement owing to ascites. Tricuspid stenosis (TS) is Answers 1053

most commonly rheumatic. It is rare to have isolated (plication of the posterior leaflet), or placement of flexi- tricuspid stenosis, as some degree of TR is always ble or rigid rings that may be partial, complete or near-­ present and coexisting mitral and aortic disease are complete. With each of these options sutures placed near occasionally seen. The incidence of rheumatic tricus- the AV node and bundle of His should be avoided pid valve disease in underdeveloped countries is still because of the risk of complete heart block. significant. Patients with TS also have presenting A review of 790 patients treated at the Cleveland symptoms of fatigue and weakness related to reduced Clinic assessed durability and risk factors for failure of cardiac output. This lesion is more common in young tricuspid repair. The authors reported that regurgitation women. A diastolic gradient across the valve causes remained stable over time with a rigid ring, increased the right atrium to thicken and dilate, while the right slowly over time with a flexible band and rose rapidly ventricle remains relatively normal in size, but under- over time with a DeVega suture annuloplasty and peri- filled. However, the clinical picture, similar to pure cardial strip reinforcement procedures. Although indica- TR, features jugular venous distention, ascites, pleural tions and techniques for tricuspid repair are in evolution, effusions and peripheral edema. In sinus rhythm, the limited data support repair with a rigid near-complete right atrial tracing and jugular venous pulse have ring. prominent a-waves that accentuate with inspiration. 66. Answer: B The cardiac murmur is mid-diastolic and increases The advantages of mitral valve repair over replace- with inspiration. There may be an opening snap that is ment should dominate decision making about patients best heard along the left sternal border. The chest with mitral regurgitation, and every effort should be radiograph shows cardiomegaly with accentuated made to repair the tricuspid valve when indicated. Since right atrial and pulmonary artery profiles. there is no overwhelming advantage to either mechanical 64. Answer: E or bioprosthetic valves in the tricuspid position, patient Tricuspid valve excision is possible if pulmonary specific factors need to be considered. Some of these pressures are not elevated and the degree of infection is include age, expected survival, arrhythmias, need for extensive. Blood flows passively through the right side right ventricular access (biopsy post-transplant, hemody- of the heart to the . After excision, approximately namic monitors in CHF, need for pacemaker), status of one-half of the patients report ankle edema and mild liver function (for future anticoagulation) and contraindi- exercise intolerance. Early death occurs in 12% of drug cations to anticoagulation. In this case, severe right heart addicts after excision without replacement of the tricus- failure is obvious but tricuspid repair is not possible. A pid valve. Hemodynamic and functional statuses begin mechanical valve would commit the patient to long-term to progressively deteriorate after 5 years. Late survival anticoagulation. A bioprosthetic valve would be a better in such patients is 63% at 15 years, with death due to choice for this patient or even a younger one, especially return to drug addiction and not to recurrent right-sided if the patient is a woman of a child-bearing age. No sur- endocarditis. After eradication of the infection, valve vival benefit (approximately 60% actuarial survival at 5 replacement can be performed months to years later. years) or difference in freedom from reoperation (>90% Although it is preferable to wait until patients have con- at 5 years) has been demonstrated for either mechanical quered their drug dependence prior to valve replace- or bioprosthetic valves in the tricuspid position. Because ment, at times a one-stage procedure is required. Many of the relatively low pressure of the right side, biopros- techniques have been described, including partial exci- thetic valves are more durable in the tricuspid position as sion, prosthetic replacement, and replacement with compared to the mitral. Freedom from structural degen- homograft tissue or a stentless porcine valve, but no eration is nearly 100% at 9 years, but non-structural dys- technique has proven to be superior. The main principal function is high (73%) due to pannus formation on the of treatment is the same for all endocarditis, which is ventricular side of the cusps. This remains a concern that complete elimination of infection. needs further investigation. 65. Answer: A However, prosthetic tricuspid replacement is not Treatment of the left side (mitral) pathology alone complication-free, and mortality approximates 10%. may result in improvement of pulmonary hypertension The risk of complete heart block is approximately 6%. and tricuspid regurgitation, but at least 16% of patients Ten percent of patients require a pacemaker during hos- with 1+ or worse tricuspid regurgitation (TR) progress pitalization for tricuspid valve replacement, and up to and require intervention. Unfortunately, reoperative 25% of patients need a pacemaker within 10 years. mortality approximates 35% in this group. Tricuspid When tricuspid annuloplasty is done, the conduction repair can be done by several methods, including the system should not be at risk and the need for permanent DeVega suture annuloplasty, the Kay “bicuspidization” pacing is less than 3%. Thrombosis of prosthetic tricus- 1054 Answers

pid valves occurs at a rate of 1% per patient-year. A chest radiograph may show mediastinal widening and Thrombosis was more common with past mechanical CT may show an effusion, but the functional informa- prostheses (Smeloff-Cutter, Bjork-Shiley) than with cur- tion from an echo study is a better guide to treatment. In rent bileaflet valves. Bioprosthetic valve thrombosis is this case (see media) the echo shows a modest-size effu- rare. If a tricuspid prosthesis does clot, however, throm- sion in the first clip (subcostal four-chamber view), but bolytic therapy has a reported success rate of over 80%. the right parasternal view is more concerning for Despite the possibility of valve thrombosis, significant impaired atrial filling and moderate tamponade. Optimal thromboembolic events are rare. Pulmonary emboliza- resolution of this patient’s crisis involves decompression tion rates following tricuspid replacement has been by reopening all or part of the sternotomy with con- reported to be less than 1%. trolled anesthesia in the operating room. However, per- 67. Answer: E cutaneous placement of a pericardial drain may be The ECG shows inferior ST depression (II, III, aVF) effective if the effusion is large and accessible. If the consistent with ischaemia. Right coronary artery (RCA) patient is in extremis, then he should be fully skin-­ insufficiency after aortic root replacement occurs in prepped and draped for bedside decompression under 2–3% of cases and has been related to female gender, local anesthesia. This reduces the risk of hemodynamic obesity, and a small LV outflow tract. Because of differ- collapse on anesthetic induction due to vasodilation and ences in porcine and human anatomy, malalignment of loss of preload. A thoracoscopic pericardial window is the RCA can occur and may not be observed until the not appropriate in an unstable patient. RV is filled or distended after emergence from 69. Answer: A CPB. Coronary insufficiency is not unique to root Stroke occurs in 1–5% of patients following car- replacement and can occur after any procedure involv- diac surgery, and it can be a devastating complication. ing the aortic valve. An early decision to perform RCA Most perioperative strokes occur in a delayed fashion grafting in patients with poor RV function, inferior LV after an initial period of normal neurologic function. wall motion abnormalities or significant unexplained This complication can be selectively treated with arrhythmias is usually life-saving. While these condi- catheter-­directed, intra-arterial administration of tions can also be caused by other factors such as coro- thrombolytic drugs within 6 h of the event, and clini- nary air emboli, poor myocardial protection, or reactions cal improvement is possible (38% in one series from to protamine and/or transfusions, these etiologies should the Cleveland Clinic). Treatment with thrombolytics be uncommon, transient, and encountered early while intravenously is associated with a higher risk of still in the operating room. bleeding in postoperative patients and is generally 68. Answer: C contraindicated within 14 days of major surgery. In Coronary ostial obstruction by the supra-annular skirt non-surgical stroke patients, intravenous thrombolyt- of a prosthesis can be catastrophic, but the timing of this ics should be given within 90–180 min of stroke scenario and lack of anginal complaints suggests another detection. An alternate plan is catheter-directed, intra- cause. This patient’s clinical situation and data are arterial thrombolytic infusion, which may be given highly suggestive of “delayed” cardiac tamponade. In within 6 h. Treatment outside these windows can lead the immediate postoperative setting with invasive moni- to hemorrhagic cerebral complications. It is most toring and chest tubes, tamponade is a primary concern, likely that the described patient experienced an intra- but days later it can be difficult to identify. The described operative embolic event. Treatment with thrombolytic patient is in marginally-compensated cardiogenic shock. therapy, whether intravenous or direct intra-arterial, Similar to the physiology of subdural hematoma, clot is not recommended at 8 h, as efficacy is low beyond breakdown and unrecovered blood cause osmotic vol- 6 h, and the risk of bleeding is extreme. Induced mod- ume expansion in the pericardial space with insidious erate hypertension and anticoagulation to arrest a worsening of hemodynamics. Delayed tamponade can propagating site of embolic clot are part of current occur after traditional full sternotomy. When partial ster- guidelines for care. The described scenario is not con- notomy is chosen the pericardium is not widely open sistent with pulmonary emboli or diffuse anoxic and may not communicate with the pleural spaces, so encephalopathy, so venous imaging studies and this complication can develop with a smaller volume of hyperbaric oxygen therapy are not appropriate postoperative blood loss. Tamponade is more common considerations. after valve surgery than CABG operations and it is more 70. Answer: E common in patients treated with anticoagulants. The echocardiogram illustrates discrete subvalvular aor- Aggressive fluid administration is the best immediate tic stenosis. This variety of subaortic stenosis can consist of management, and the added preload will usually improve a fibrous ridge localized to one segment of the left ventricu- blood pressure and cardiac output enough to allow for lar outlet or it can be circumferential. There may be associ- emergent intervention. Diuresis is an incorrect strategy. ated aortic regurgitation due to aortic leaflet thickening; this Answers 1055

is thought to result from the jet of blood produced by ejec- associated with complete atrioventricular septal defect. tion through the stenoric subvalvular segment. Operation Azygos continuation of the inferior vena cava is a condition for discrete subaortic stenosis consists of complete trans- in which the inferior vena cava does not drain normally into aortic resection of the fibrous ridge, often in conjunction the right atrium with the hepatic veins, but rather with the with a septal myomectomy. There are other more severe azygos vein into the superior vena cava. This would not forms of subvalvular stenosis that result in a “tunnel” type cause an excessive amount of blood streaming from the left of left ventricular outlet obstruction, often with associated atrium; rather it would cause an excessive amount of blood hypoplasia of the aortic annulus. These patients may be streaming from the superior vena caval orifice if the tourni- managed by an aortoventriculoplasty (Konno procedure) or quet around the superior vena cava was cephalad to the a modified that enlarges the left ventricular entrance of the azygous vein. The most common partial outlet. The aortic valve itself is usually tricuspid without anomalous pulmonary venous connection is drainage of the leaflet fusion; thus there is no role for aortic valvotomy or right superior pulmonary vein into the right superior vena balloon dilatation. Continued follow-up is appropriate cava. This is evidenced by excessive blood streaming from because the associated aortic valve regurgitation is likely to the superior vena caval orifice and not from the left atrium. progress. A patent ductus arteriosus is commonly associated with 71. Answer: D complete arrioventricular septal defect. In some series this Williams syndrome is the association of supravalvu- association is present in one of every three patients. The lar aortic stenosis, “elfin” facies, and mental retardation. blood from the patent ductus arteriosus fills the pulmonary Supravalvular stenosis is usually localized, often with an artery, traverses the pulmonary bed, and flows back to the associated ridge inside the aorta, although in some cases left atrium, and becomes evident as an excessive amount of it may be more diffuse. True stenosis of the aortic valve blood streaming from the left atrium. This should be the itself is rare in Williams syndrome. Thus, operations that first consideration in the clinical setting described. A patent involve the aortic valve are not generally applicable in left superior vena cava to the coronary sinus would be evi- this condition. Since continued growth of the aorta is denced by blood streaming from the orifice of the coronary desired, operations that involve circumferential graft sinus and not from the left atrium. replacement of the ascending aorta are also not appro- 74. Answer: B priate. Patch enlargement of the aorta is most commonly The three major technical challenges associated with used to repair this type of stenosis. Balloon dilation of total repair of complete atrioventricular (AV) septal the aortic valve will not address the aortic pathology. defects are (1) complete obliteration of intracardiac 72. Answer: C shunts, (2) reconstruction of competent yet non-stenotic Embryologically, the coronary sinus is an extension of left and right AV valves, and (3) preservation of an intact the left superior vena cava. It lies in the atrioventricular conduction system. Large residual intracardiac shunts at groove behind the left atrium. When there is a defect in its either atrial or ventricular level are rare following repair. roof, coronary sinus blood can enter the left atrium Small residual intracardiac shunts are usually well toler- directly. Thus, a coronary sinus septal defect (“unroofed ated. Poor long-term results should not occur because of coronary sinus”) in association with a persistent left supe- elevated pulmonary vascular resistance if patients are rior vena cava can result in significant flow of systemic appropriately selected preoperatively. Left ventricular venous blood to the left atrium; this may produce sys- outflow tract obstruction is a known complication of an temic arterial desaturation when an atrial septal defect is incorrectly placed patch used to obliterate the interven- closed. Anomalous hepatic venous drainage can cause a tricular communication. Obstruction may also occur in small right-to-left shunt into the left atrium; this is unlikely some patients for anatomic reasons. Left ventricular out- to cause significant systemic arterial desaturation. flow tract obstruction, however, causes hemodynamic Unrecognized anomalous pulmonary venous drainage deterioration early after repair. This complication is not results in a persistent left-to-right shunt which does not well tolerated and it is thus not a cause of long-term prob- produce systemic arterial desaturation. Pulmonary valve lems. Creation of a competent yet non-stenotic left AV stenosis, even when severe, will not produce a right-to- valve (“mitral” valve) can pose a real challenge in the left shunt when the atrial septum has been rendered intact. small infant. Mitral valve regurgitation must be suspected Ventricular septal defects produce left-to-right shunts in any patient with persistent hemodynamic deterioration except in cases of advanced pulmonary vascular disease after repair of complete AV septal defect, especially in the (Eisenmenger syndrome) where increased pulmonary presence of normal sinus rhythm and absence of residual vascular resistance can produce a right-to-left shunt. left-to-right shunts. 73. Answer: D 75. Answer: C Anomalous origin of the left coronary artery from the Atrial arrhythmias are uncommon following an arte- pulmonary artery does create a left-to-right shunt which rial switch operation. Stenosis of the coronary artery ori- empties into the pulmonary artery. However, this shunt gins has been extremely uncommon. The most common flow is usually small and in addition, this anomaly is rarely late complication after an arterial switch operation for 1056 Answers

transposition of the great arteries (TGA) is supravalvu- that the evaluation proceed in a step-wise fashion and lar pulmonary artery stenosis. Recanalization of the duc- that redundant testing not be performed once the diag- tus arteriosus cannot occur since most arterial switch nosis has been established. Angiography is rarely indi- operations involve the Lecompte maneuver which cated for the diagnosis of a vascular ring. It is indicated requires that the ductus arteriosus be divided in order to if associated congenital heart lesions are to be simulta- transfer the bifurcation of the pulmonary artery to a neously evaluated. The best initial diagnostic study is position anterior to the ascending aorta. Stenosis at the an antero-posterior and lateral chest X-ray. A barium aortic anastomosis is infrequent. swallow should be the next study. Deep, persistent, 76. Answer: B extrinsic indentation of the posterior aspect of the Atrial septal defects created by balloon septostomy are esophagus indicates either a double aortic arch or right unlikely to close and it is, therefore, unlikely that the aortic arch with retroesophageal left subclavian artery patients would develop lethal hypoxemia. The major rea- and left ligamentum. Persistent bilateral compressions son for performing the arterial switch operation within the of the esophagus on the antero-posterior view are usu- first 2 weeks of life for patients with transposition of the ally present with a double aortic arch. Computed great arteries (TGA) and intact ventricular septum is to tomography and magnetic resonance imaging will in take advantage of the fact that the left ventricle in utero many cases demonstrate the precise morphology of a has increased wall thickness because the ductus arteriosus vascular ring. However, they are not good initial diag- is open and the left ventricle pumps against systemic nostic studies because of the cost and low yield when resistance. Left ventricular wall thickness decreases rap- applied to a large number of patients with stridor and idly within the first weeks of life and several series have no other evidence for vascular ring. They are typically identified age older than 2 weeks at the time of operation used only if enough information is not obtained from as a risk factor for the arterial switch operation for TGA the barium swallow to recommend vascular ring divi- with intact ventricular septum. It is true that the newborn sion through a left thoracotomy. is well prepared physiologically to withstand the stress of 79. Answer: D an operation, in general, but this is not the dominant rea- The diagnosis of ventricular septal defect (VSD) is son for performing an arterial switch operation in the first usually made on the basis of clinical examination and 2 weeks of life. The pulmonary vascular resistance is history. Further diagnostic evaluation is aimed primarily unlikely to be significantly elevated after an uncompli- at determining size, location, and number of septal cated arterial switch operation for TGA, as most patients defects. Associated complications of the defect includ- have essentially normal pulmonary artery pressures in the ing valve function, magnitude and direction of shunt, early postoperative period. Prolonged administration of and pulmonary vascular resistance are also identified by prostaglandins may have deleterious effects, but some additional studies. Echocardiography has replaced car- patients have been safely maintained on prostaglandins diac catheterization and cineangiography as a reliable for many weeks. and noninvasive method of obtaining most of the infor- 77. Answer: D mation required prior to surgical therapy. Two-­ In a patient with tricuspid atresia and a ventricular septal dimensional echocardiography and color Doppler flow defect a Fontan-type operation is the future long-­term goal. studies can reliably demonstrate size, number, and loca- Closure of an atrial septal defect would be fatal because the tion of the septal defects relative to other intracardiac only pathway for systemic venous return to the systemic structures including the cardiac valves. Additional infor- ventricle would be removed. Closure of the ventricular sep- mation regarding atrioventricular valve function and tal defect would induce severe cyanosis by completely prolapse of aortic valve leaflets can also be demon- obstructing the main source of pulmonary blood flow. strated. Coronary artery anatomy, particularly of the Since a Fontan-type operation requires the pulmonary vas- proximal branches, can often be obtained by echocar- cular resistance to be low, a secure and properly placed pul- diography but cineangiography is definitive. Preoperative monary artery band will protect the pulmonary vascular knowledge of the course of the coronaries is not rou- bed while controlling congestive heart failure symptoms. tinely required for surgical repair of isolated VSDs since The augmentation of pulmonary blood flow by a systemic- a ventriculotomy is rarely necessary. Pulmonary artery to-pulmonary shunt (Blalock-Taussig shunt) would be pressure and pulmonary vascular resistance can be esti- unnecessary and harmful as it would further risk the devel- mated by echocardiographic measurements of the peak opment of both congestive heart failure and pulmonary vas- flow velocity across the tricuspid valve if regurgitation is cular disease. present. However, in cases where no tricuspid insuffi- 78. Answer: C ciency is present or when elevated pulmonary vascular A vascular ring should be suspected in infants with resistance is suspected, cardiac catheterization remains stridor, “wheezing”, recurrent respiratory tract infec- the most accurate method of assessment. In a 2 year old tions, and the classic “seal bark” cough. It is important child with a VSD, cardiac catheterization is useful to Answers 1057

determine pulmonary vascular resistance and response artery (PA) there is a left to right shunt present, the

to therapy such as oxygen or nitric oxide. chamber at which the O2 step-up first appears disclosing 80. Answer: E the level of the defect. If there is an oxygen stepdown At cardiac catheterization oxygen saturation and between the left atrium and aorta, a right to left shunt is pressures are recorded in each great vessel and cardiac present. The magnitude of a left-to-right shunt can be chamber. When there is an increase in oxygen saturation calculated from oxygen saturation data. Using the Fick from the superior vena cava (SVC) to the pulmonary principle the pulmonary blood flow OUptake Qp = 2 Pulmonary vein Ocontent - pulmonary arteryOcontent 22

and the systemic blood flow, function and pulmonary function. Nevertheless, in cases of isolated left ventricular dysfunction, highly selective Oconsumption Qs = 2 left ventricular assist devices have been utilized with AorticOcontentS- VC Ocontent 22 eventual survival of 40–70%. Use of a VAD without an oxygenator via existing right atrial and aortic cannulas is

O2 content is directly proportional to O2 saturation, not appropriate because deoxygenated blood would be thus the latter can be substituted in these equations. Also, pumped into the aorta. ECMO utilizes veno-arterial

O2 uptake equals O2 consumption. Thus, when the first bypass with a membrane oxygenator, which allows both equation is divided by the second, hemodynamic and pulmonary support. ECMO results have been excellent as treatment for neonatal respiratory Qp Aortic saturation -SVCsaturation ()100 - 55 = = = 30. failure. Reported survival exceeds 80% for some indica- Qs PVsaturation - PAsaturatiion 100 -85 () tions, including neonates with meconium aspiration, persistent fetal circulation, and primary pulmonary This relationship, which can be estimated from easily hypertension. ECMO has been utilized with increased measured data, is extremely useful in managing patients frequency to provide postcardiotomy support for chil- with congenital defects such as in determining the need dren with severe cardiopulmonary dysfunction after sur- for operation and in determining the balance of pulmo- gery for congenital heart disease. Myocardial recovery nary and systemic blood flows in patients following such can be expected in some patients in 24–48 h, and most operations as the . patients who are going to recover sufficiently to be 81. Answer: B weaned from ECMO will do so in 5–7 days. Survival for After surgery for congenital heart disease, severe low postcardiotomy application of ECMO after biventricular cardiac output state may be refractory to maximal medi- repair is approximately 40%, but it is less following sin- cal management. Mechanical circulatory support may gle ventricle palliation. ECMO provides total cardiopul- allow salvage of many of these challenging patients. monary support and may allow the heart and lungs to Three forms of mechanical support are currently avail- heal. Although this may be accomplished with venous able to infants and children: (1) the intra-aortic balloon drainage from the right atrium and inflow into the aorta, pump (IABP); (2) the ventricular assist device (VAD); many centers feel it is important to decompress the left and (3) extracorporeal membrane oxygenation (ECMO). side of the heart as well with an additional drainage Each technique has advantages and disadvantages. This catheter in the left atrium. Although adult ECMO can be child has biventricular dysfunction, probably from conducted via femoral vein and femoral artery, it is not stunned myocardium following a prolonged crossclamp possible to use the femoral vessels in a neonate because and bypass time, and requires biventricular extracorpo- of their small sizes. Veno-venous ECMO is a useful real support. The criteria for extracorporeal support technique for isolated respiratory dysfunction such as include significant ventricular dysfunction despite maxi- meconium aspiration, but it is not useful in patients with mal inotropic support with no residual correctable severe cardiac dysfunction. defects and reversible ventricular dysfunction. The 82. Answer: E IABP is available in small sizes for use in neonates, but Supraventricular arrhythmia is the most common late the aortic arch surgery performed in this patient repre- problem for patients after a Fontan procedure. This is sents a contraindication to IABP use. Three types of particularly true for patients that have had an atrio-­ VAD may be utilized: left, right, and biventricular. pulmonary type Fontan. These patients can develop Children with left ventricular dysfunction are even less massive right atrial dilation and their repairs often likely than adults to have preserved right ventricular involve multiple atrial suture lines. These two factors 1058 Answers

contribute significantly to late atrial tachycardias. coronary sinus. The incidence of fistulas from the vari- Patients with atrial arrhythmias and dilation of the atrial ous sinuses and the percent distribution to adjacent car- portion of the atrio-pulmonary connection benefit diac structures are shown below: greatly from conversion to an extracardiac Fontan and Anatomy of Aneurysms or Fistulas of The Sinus of right atrial maze procedure. Protein-losing enteropathy Valsalva (n = 149) in 129 Patients in Fontan patients is less common than supraventricular tachycardia. Most of the difficulties with pleural effu- Right coronary Left coronary Noncoronary Site of rupture sinus sinus sinus Total sions occur in the early postoperative period and gener- Nonruptured 31 19 29 79 ally resolve with time. The incidence of pleural effusions Ruptured seems to be reduced when the Fontan pathway is fenes- Right ventricle 35 1 6 42 trated. Congestive heart failure due to failure of the sin- Right atrium 9 0 11 20 gle ventricle is relatively uncommon. Left atrium 1 1 2 4 83. Answer: E Left ventricle 2 0 1 3 Inserting a right ventricle-to-pulmonary artery con- Pericardium 0 0 1 1 Total 78 21 50 149 duit is necessary in the repair of many congenital heart defects to establish unobstructed blood flow between the From: Takach TJ, Reul OJ, Duncan JM, et al. Sinus of valsalva aneuyrsm or fistula: management and outcome. Ann Thorac Surg 1999; right ventricle and the pulmonary circulation. This is 68: 1573–1577 often referred to as a “Rastelli” procedure. Whether reconstruction is achieved with a bioprosthetic valve-­ Rupture of a sinus of Valsalva aneurysm from the conduit or a homograft conduit, progressive stenosis and non-­coronary sinus results in a fistula to the right atrium restriction to pulmonary blood flow often occurs after in most patients. Likewise, a ruptured sinus of Valsalva several years. Calcification plays a major role in the aneurysm from the right coronary sinus fistulates into development of this stenosis, but calcification in and of the right ventricle most frequently. Sinus of Valsalva itself is not an indication for conduit change. Although aneurysms of the left coronary sinus are rare and may most of these patients develop pulmonary stenosis as result in a fistula to the left atrium or the pericardial their chief problem, some also develop pulmonary space. Presenting symptoms of rupture of a sinus of regurgitation. Indications for conduit replacement Valsalva aneurysm are those of aortic valve regurgita- because of pulmonary regurgitation include exercise tion and congestive heart failure. Patients diagnosed intolerance, atrial and/or ventricular arrhythmias and with a sinus of Valsalva aneurysm should undergo surgi- progressive ventricular dilation. Occasionally such cal repair to resect the aneurysm tissue and to close patients can initially be treated with balloon valvulo- (with patches) the sinus defect and any associated ven- plasty. More frequently, however, because of a combina- tricular septal defect. If aortic regurgitation has devel- tion of somatic growth and calcification of the valve oped secondary to prolapse or elongation of the involved leaflets the conduit requires replacement. Cath findings aortic leaflet, repair of the aortic valve may also be indi- of a peak gradient greater than 50 mmHg or an echocar- cated. The approach is through a median sternotomy diographic mean gradient over 50 mmHg, near-systemic using cardiopulmonary bypass and cardioplegic arrest. right ventricular pressures or severe tricuspid regurgita- The lesions can usually be exposed and repaired through tion are all indications for surgical intervention. The the aortic root, although exposure may be improved by development of right ventricular hypertrophy alone does opening the involved cardiac chamber. In the illustration not justify conduit replacement. a noncoronary sinus of Valsalva aneurysm is visualized. 84. Answer: D Its “windsock” is inverted and excised to create an ellip- Sinus of Valsalva aneurysms are uncommon lesions tical opening. The defect is closed transversely. Excellent involving the aortic root sinuses. These lesions are char- surgical results are currently obtained in most patients acterized by thin-walled outpouchings of tissue arising with these lesions. from media-deficient regions of the aortic sinuses and 85. Answer: C extending to surrounding cardiac structures. Several factors may increase the risk of the Fontan Approximately 50% have an associated subarterial ven- operation in patients with single ventricle physiology. tricular septal defect. They may remain asymptomatic The ideal candidate has: (1) low pulmonary artery pres- for many years, but are prone to rupture with consequent sures (systolic PA <15–18 mmHg); (2) pulmonary vas- fistulization to adjacent cardiac chambers or the pericar- cular resistance (PVR) <2 Woods units; (3) a low dial space. While they may occur in any of the three aor- transpulmonary gradient <8 mmHg; (4) a competent tic sinuses, the right (67%) and non-coronary (25%) atrioventricular valve; and (5) good systolic and dia- sinuses are affected much more frequently than the left stolic ventricular function (ejection fraction >50%, Answers 1059

LVEDP <10 mmHg). Pulmonary blood flow is passive cations to achieve the Fontan reconstruction, but add no and driven only by the difference between the central significant operative risk. Pulmonary artery stenosis, if venous pressure and the pulmonary venous pressure. surgically correctable, does not pose an increased risk Therefore, obstruction due either to anatomic (narrowed for Fontan. Decreased ventricular systolic or diastolic pulmonary arteries or veins) or physiologic factors function increases the risk of the Fontan procedure. An (PVR) will make a successful Fontan procedure less ejection fraction <40% or an LVEDP >12 mmHg are likely. In multivariate risk analyses, pulmonary artery considered relative contraindications to the procedure. pressure and pulmonary vascular resistance are highly Likewise, pulmonary artery systolic pressure linked variables. Anatomic abnormalities and variations >20 mmHg, PVR >4 Wood units, or a transpulmonary such as bilateral superior vena cava or interrupted infe- gradient over 10 mmHg are each associated with rior vena cava may require additional technical modifi- increased risk for the Fontan procedure. Index

A levosimendan, 602 Acidosis, 764, 836 milrinone, 602 Activated coagulation time (ACT), 764, 801 African-American Heart failure Trial (A-HeFT), 601 Acute aortic syndrome (AAS), 49, 531 Aggressive anti-impulse therapy, 503 Acute cellular rejection, 660–661, 1005 Akinetic hibernating myocardium, 57 Acute coronary syndrome (ACS), 270 Albumin and glutaraldehyde, 123, 124 Acute heart failure (AHF) Alfieri edge-to-edge repair, 455 decompensation, 602 Allograft rejection diuretics, 603 acute cellular rejection, 1005 inotropes, 603 antibody-mediated rejection (AMR), 1005, 1006 modified Forrester classification, 602 outcomes, 1006 nitrates, 603 treatment, 1006 Acute ischaemia-reperfusion injury, 666 Amiloride, 601 Acute kidney injury (AKI), definition, 755 Amplatzer® device, 232 Acute kidney injury network (AKIN), 755 Anaemia, 64 Acute kidney injury requiring dialysis (AKI-D), 755 AndraStent, 451 Acute myocardial infarction (AMI), 229 Angiotensin converting enzyme (ACE) inhibitors, 73 Acute rejection, 642, 649, 650 Angiotensin receptor blockers (ARBs), 598, 601, 602 Acute respiratory distress syndrome (ARDS), 623, 625–627 Angiotensin receptor-neprilysin inhibitor (ARNI), 598 Acute type A aortic dissection (ATAAD) Anomalous aortic origin of the left coronary artery classification, 478–480 (AAOLCA), 824, 825 clinical presentation, 476 Anomalous aortic origin of the right coronary artery (AAORCA), 824 complications and surveillance, 483–485 Anomalous left coronary artery from pulmonary artery (ALCAPA), contemporary reports, 484 823, 824 diagnostic testing, 476, 478 Anomalous right coronary artery from the pulmonary artery iatrogenic risk factors, 476 (ARCAPA), 823, 824 incidence, 475 Antegrade crystalloid cardioplegia, 431 mortality rate, 475, 476 Anthracycline-induced cardiomyopathy, 1001 open repair, 476 Antibody-mediated rejection (AMR), 661, 1005, 1006 prognosis, 475 Anti-CD52 antibodies, 657 risk factors, 476 Anticoagulation, 783, 784 RTAD, 478 Anti-hypertensive therapy, 669 surgical technique Antiplatelet agents, 763 antegrade cerebral perfusion (ACP), 480 Antithymocyte globulin, 1005 antegrade TEVAR, 483 Aorta aortic root repair options, 482 blunt aortic injury, 700 arch repair options, 480, 482 penetrating aortic injury, 700 cardiopulmonary bypass (CPB), 480 Aortic arch aneurysms common repair strategies, 480, 481 anatomy, 530, 531 elephant trunk (ET), 482 aortic arch replacement, 537 frozen elephant trunk (FET), 482, 483 cerebral metabolic rate, 536 hypothermic circulatory arrest (HCA), 480 clinical presentation, 531 myocardial protection, 480 CT scan, 530 open descending thoracic/thoracoabdominal aortic repair, 482 definition, 529 transesophageal echocardiography, 480 follow-up, 530 symptoms, 476 growth rate, 531 Adeno-associated viruses (AAVs), 272 imaging, 531–534 Adenovirus, 272 isolated arch aneurysms, 529 Adult cellular rhabdomyoma, 677 pathology, 531 Advanced heart failure perfusion and cerebral protection, 537 dobutamine, 602 Saccular arch aneurysm, 529, 530

© Springer Nature Switzerland AG 2020 1061 S. G. Raja (ed.), Cardiac Surgery, https://doi.org/10.1007/978-3-030-24174-2 1062 Index

Aortic arch aneurysms (cont.) sizing techniques, 320 surgical intervention, 533, 534 straight Dacron tube graft, 319 surgical repair, 538–542 teflon pledgeds, 320 treatment upper hemi-sternotomy, 319 DHCA, 534 water sealing test, 321, 322 perfusion and cerebral protection, 534 deciding factors, 318 RCP, 538 external aortic ring annuloplasty, 318 Aortic diseases indication, 318 CT, 49, 50 intraoperative measurement, 319 MRI, 49, 50 natural history, 315 , 861 pathophysiology, 315, 317 Aortic prosthetic valve infective endocarditis, 409 prophylactic aortic root/ascending aorta surgery, 318 Aortic regurgitation (AR), 33, 362, 994 technical errors, 318 Aortic root anatomy, 326–328 Yacoub technique, 315 Aortic stenosis, 30 Aortopathy, 362, 363 criteria for assessing, 31 Aortopulmonary collaterals, 647 stress echo, 37 (APW) types of, 32 associated conditions, 870 Aortic translocation, 913, 914 classifications, 870 Aortic uncrossing procedure, 990 clinical manifestations, 870 diagnosis, 870, 871 annuloplasty, 333 embryology, 869 arch/axillary cannulation, 330 management ascending aorta cannulation, 330 interventional therapy, 872 cusp repair techniques medical therapy, 871 free margin plication, 333, 334 post operative management, 873 free margin resuspension, 333, 334 surgical, 872, 873 follow-up data, 325 natural history, 871 functional aortic annulus restoration, 330, 331 outcomes, 873 indications, 325, 328 pathophysiology, 870 outcomes, 325 Apical myectomy, 736 repair-oriented functional classification, 328, 329 APOLLO trial, 241 for root aneurysms, 325 Aprotinin, 65 standard midline sternotomy, 330 Arch advancement technique, 935 subcommissural annuloplasty (SCA), 331 Arginine vasopressin (AVP), 75, 76 valve exposure and assessment, 330 Arterial cannulation, 784 valve-sparing root replacement Arterial graft patency rates, 213 built-in neoaortic sinuses graft, 332 Arterial line filter (ALF), 786 coronary button preparation, 331 Arterial Revascularization Trial (ART), 203, 214 Dacron graft selection, 331, 332 Arterial switch operation, 901, 913 external aortic root dissection, 331 , 152 graft preparation, 331 Arto System, 459 leaflet assessment and repair, 332 Asymmetric hypertrophy of left ventricle, 735 long-term durability, 331 Atresia, 908 prosthesis preparation, 332, 333 Atrial fibrillation (AF), 738 proximal suture line, 332 medical management, 727 sinus of valsalva resection, 331 myocardial sleeve transition zones, 727 valve re-implantation, 331 pathology, 727 venous cannulation, 330 peri-atrial fat pad embedding, 727 Aortic valve replacement (AVR), 421, 834 preoperative assessment, 728 Aortic valve-sparing root replacement (AVSRR), 338, 340 surgical or hybrid procedure, 728 aortic cusp prolapse, 318 surgical procedures, 728 calibrated expansible aortic ring annuloplasty, 319 surgical rhythm control David operation, 315, 316 Cox-Maze (CM) procedure, 728, 729 antegrade blood cardioplegia, 319 hybrid approach, 730 aortic sinuses excision, 319 left atrial appendage exclusion, 730, 731 arterial cannulation, 319 less extensive lesion sets, 729 Dacron graft, 320, 321 Atrial septal defects (ASDs) David V operation, 322 anatomy, 839 distal graft anastomosis, 322 cardiac catheterization, 841 Hegar dilator, 321 clinical features, 840 multiple horinzontal matress sutures, 320 early complications, 845 optimal cusp orientation and coaptation, 321 imaging, 840, 841 Pledgeted 4-0 polypropylene sutures, 319 indications and timing, 841 polypropylene sutures, 321 intracardiac echocardiogram, 840, 841 sinotubular diameter sizing, 320 late complication, 846 Index 1063

MICS versus catheter based techniques, 845 Balloon-expandable IntraStent, 451 postoperative complication, 845 Baseline 2D echocardiography, 56 primum, 840 Bentall procedure secundum, 839 aortic root exposure, 310, 311 sinus venosus, 840 cardiopulmonary bypass, 309, 310 surgical and interventional therapy recommendations, 843–844 closure, 313 surgical technique and results coronary button re-implantation, 312 minimally invasive cardiac surgery, 844 distal anastomosis, 312 right axillary incision approach, 844 evolutions, 307, 308 secundum ASD, 844 French Cuff technique, 307, 311 sinus venous, 844 hemi-arch replacement, 312, 313 traditional surgical closure, 844 indications, 307 surgical vs. transcatheter management, 841–845 mini-Bentall procedure, 307, 308 transcatheter vs. surgical closure, 845 mini-sternotomy, 308–310 transesophageal echocardiogram, 840 outcomes, 313, 314 transthoracic echocardiogram, 840 surgical management, 307 Atrial switch operation, 900 Bernoulli equation, 30 Atriclip LAA exclusion system, 730, 731 Berry syndrome, 870 Atrioventricular (AVV) dysfunction, 959 Beta-blockers, 598, 601 Atrioventricular disruption Bicuspid aortic valve (BAV) external repair, 779 ACC/AHA guidelines, 364 internal repair, 778 anatomy, 359–361 management, 778, 779 aortic arch, 368 pathogenesis, 778 aortic replacement, 366–368 risk factors, 778 aortic valve dysfunction, 366, 367 types, 777 aortic valve stenosis, 365 Atrioventricular septal defect (AVSD) aortopathy, 362, 363 anatomic features and nomenclature, 949–951 cardiovascular event, 360 diagnosis, 951 clinical presentation, 361 indications and timing, 951, 952 complications, 359 LVOTO, 959 diagnosis, 361, 362 Nunn repair etiopathogenesis, 361 annuloplasty effect, 955 incidences, 337 atrial septation, 956, 957 indications, 364 autologous pericardium/CardioCel® membrane, 956 management, 363 AV valve competence, 957 medical events, 360 AV valve leaflet coaptation, 954 outcomes, 336 AV valve replacement, 958 postoperative surveillance, 368 competence testing, 956 prevalence, 359, 361 ligation, 954 prosthetic valve, 365, 366 line of septation, 954 randomized clinical trials, 336 LVOT obstruction, 958 regurgitation, 364 mechanism of incompetence, 956 Ross procedure, 366 modified DeVega annuloplasty, 957 shaving technique, 336 modified ultrafiltration, 954 subcommissural annuloplasty, 364, 365 post cardiopulmonary bypass intraoperative transesophageal surgical events, 360 imaging, 957, 958 TAVR, 365 Rastelli A leaflet arrangement, 955 valve-related complications, 337 small interatrial communication, 957 valvular complications, 362 superior suture placement, 955 Bicuspid valve repair suture placement en face, 955 cusp repair, 336, 337 ventricular portion closure, 955 free margin plication/resuspension, 335 zone of apposition, 956 outcomes, 340 outcomes of surgery, 958, 959 shaving technique, 335 surgical repair, 952–954 survival, 340, 341 Atrioventricular valves, 906 type 0, 335 Australian repair, 952 type 1, 335 Autosomal dominant familial disease, 996 Bidirectional Glenn shunt, 816 Awake extra corporeal membrane oxygenation, 626 Bilateral internal mammary artery Axillary artery insertion, 616 anastomoses, 202 Azathioprine, 658 circumflex marginal branches, 200 clinical outcomes, 202–203 DSWI, 204 B endothoracic fascia, 201 Balloon angioplasty (PTCA), 257 history, 199, 200 Balloon (BAS), 817, 900, 944 in-situ right IMA, 202 1064 Index

Bilateral internal mammary artery (cont.) anterograde cerebral perfusion (ACP), 565 pericardium anterior, 201 drugs, 565 proximal anastomosis, 202 hematocrit, 565 rationale, 200 hypothermic cardiac arrest (HCA), 565 retrosternal crossover in-situ right IMA, 202 pH management, 565 side-to-side perpendicular anastomoses, 202 retrograde cerebral perfusion (RCP), 565 T or Y graft configuration, 202 topical head cooling, 565 Bilateral internal thoracic arteries (BITA), 168, 208 Brain stem death Bilateral pulmonary arteries, 872 in catecholamine surge, 640 Bilateral pulmonary plethora, 853 diagnosis, 639 Bileaflet valves, 291, 375 Branch pulmonary arteries, 814 BIMA grafting, 201 Bretschneider, 104 Bioglue®, 124 Brockenbrough-Braunwald-Morrow sign, 741 BioGlue® Surgical Adhesive-impregnated Teflon, 778 Bronchiolitis obliterans syndrome (BOS) phenotype, 651 Bioprosthetic mitral valves, 375 B-type natriuretic peptide (BNP), 640 BiPella configuration, 634 Biplane main pulmonary artery angiography, 449 Bipolar (RFA), 728 C BiVAD CentriMags™, 636 Cabrol procedure, 307 Bivalirudin, 115 Caisson system, 459 Biventricular aortic valve stenosis, 832 Calcific constrictive pericarditis, 707 Biventricular hypertrophy, 997 Calcified mural thrombus (pseudotumor), 675 Biventricular pacing, see Cardiac resynchronization therapy (CRT) Calcifying amorphous pseudotumor, 675 Bjork-Shiley tilting disk valve, 291 Calcineurin inhibitor, 1005 Bladder reservoirs, 801 Calcium blockers, 602 Blalock-Hanlon operation, 817 Calculated panel reactive antibody (cPRA) algorithm, 655 Bleeding Candy-plug technique, 501, 502 management, 763 Carbomedics bileaflet valve, 375 risk factors, 763–764 CarboMedics mechanical valve, 295, 375 Blood cardioplegia, 96, 792 Carcinoid heart disease Blood conservation strategies anterior mitral valve leaflet, 464 logistic regression analysis, 64 cardiac valve reintervention, 471 minimize blood loss chest radiographs, 470 anaesthetic technique, 65 clinical findings, 466 dual antiplatelet therapy, 65 echocardiographic studies, 466 haematological irregularities, 66, 67 functional outcomes, 471 hypothermic circulatory arrest, 65 gross surgical pathologic specimens, 464 lysine analogue antifibrinolytics, 65 microscopic examination, 464 postoperative, 66 New York Heart Association (NYHA) class I-II functional status, preoperative assessment, 65 467 retrograde autologous priming, 65 NYHA class, 468 serine protease inhibitor aprotinin, 65 operative mortality, 471 patient blood management, 64, 67 pathologic specimen, 465 PBM, 63 pathology and pathophysiology, 465, 466 red cell mass and erythropoiesis, optimization of, 64 perioperative management, 467 risk of, 64 right ventricular dilation, 468 transfusion requirements, 66 surgical considerations Blood-exposed non-intimal surface (BENIS), 223 degree of stenosis, 469 Blood products, 763 left-sided heart disease, 470 Blood pump functions, 798 postoperative surveillance, 470 Blunt aortic injuries, 700 prosthetic valves, 469–470 Blunt cardiac injury (BCI), 699, 700 survival of patients, 463, 465 Blunt carotid injury, 701 symptoms, 466 Blunt innominate artery injuries, 700 transesophageal echocardiogram, 466, 467 Body’s hemostatic ability, 764 tricuspid and pulmonary valves, 463 Bolton Relay Thoracic Stent-Graft with Plus Delivery System, 553 valve replacement, 463, 467, 468 Bovine serum albumin (BSA), 123 perioperative mortality, 470 Bovine serum albumin-glutaraldehyde (BSAG), 124 pulmonary valve replacement, 469 Bradycardia, 84 survival and functional outcomes, 471 Brain death, associated with myocardial injury, 640 tricuspid valve replacement, 469 Brain injury Cardiac allograft vasculopathy (CAV), 642, 651, 652, 661, 665, 669 incidences, 564 Cardiac arrhythmias, 965 independent predictors, 564 Cardiac catheterization, 871 monitoring, 565, 566 findings, 711 pathophysiology, 564 history of, 3 protective strategies invasive diagnostic (see Invasive coronary angiography) Index 1065

right heart catheterization subcutaneous ICDs, 87 complications, 11 surgical lead implantation, 91 indications, 9, 10 threshold, 84 PCWP, 11, 12 troubleshooting temporary epicardial systems, 89 procedure, 11 types of, 84, 85 pulmonary artery, 11 WiSE CRT system, 87 right atrium, 11 Cardiac resynchronisation therapy defibrillator (CRT-D), 609 right ventricle, 11 Cardiac resynchronisation therapy pacemaker (CRT-P), 609 Cardiac computerized tomography (CT) Cardiac resynchronization therapy (CRT), 84, 238, 640 aortic diseases, 49, 50 CARE-HF trial, 608 CAD, 43, 45 COMPANION trial, 608 cardiac tumours, 49 complications, 611 cardiomyopathies, 46, 47 coronary sinus cannulation, 610 clinical applications, 44 coronary sinus venogram, 610, 611 congenital heart disease, 48 definition, 607 heart failure, 46, 47 device programming, 611 iodinated low-osmolar agents, 41 future research, 611 ionising radiation, 42 hemodynamic benefits, 608 myocardial ischaemia, 45, 46 indications, 609 myocardial viability, 46 LV lead position, 610 pericardial diseases, 48 MADIT-CRT trial, 608 post-surgical patient, 51 mechanisms of action, 608 valvular heart disease, 47, 48 MIRACLE trial, 608 x-ray tube, 41 MUSTIC trial, 608 Cardiac conduction disease, 84 patient selection, 609 Cardiac decompression, 779 post implantation, 611 Cardiac fibroma, 678, 679 RAFT trial, 608 Cardiac Insufficiency Bisoprolol Study II (CIBIS II), 598 REVERSE trial, 608 Cardiac lipoma, 678 venous access, 610 Cardiac magnetic resonance imaging (CMR), 42, 56, 58, 60 Cardiac surgery Cardiac myxoma, 681 albumin and glutaraldehyde, 123, 124 Carney complex, 680 clinical use in, 121 CNC1 susceptibility gene (PRKAR1A), 681 cost analysis, 125 constitutional symptoms, 679 cost of blood products, 125 cytogenetic studies, 681 cyanoacrylates, 124 diagnosis, 680 diminished pain, 120 differential diagnosis, 681, 682 expenses and units, 126 embolic phenomenon, 679 fibrin sealants, 123 endocardial attachment and mean age, 679 hemodynamic instability, 119 histogenesis and development, 681 hemostatic agents, 119 histologic diagnosis, 680 minimal tissue trauma, 120 immunohistochemical profile, 682 PEG, 123 initial symptoms, 680 perioperative and postoperative complications, 119 molecular basis, 679 ramifications, 119 overlapping functions and signaling pathways, 681 RCT, 122 Cardiac pacing surgical scenarios in, 122 anatomy and physiology, 81 thrombin gelatin matrix, 124, 125 complications and problems, 86 tissue adhesives, 121 dual chamber pacemaker, 85 tissue sealants, 121 epicardial leads, 89 topical hemostatic agents, 120, 121 guideline recommendations for, 84 Cardiac surgery associated acute kidney injury (CSA-AKI) His bundle, 86 biomarkers, 760 implantation techniques, 85 incidence, 755 indications for, 82–84 intraoperative strategies lead extraction, 91 anemia correction, 757 leadless pacemaker, 86, 87 balanced crystalloid solutions, 758 MRI-induced cardiac lead heating and movement, 88 blood glucose control, 757 occlusion balloon, 91 blood management program, 758 pacemaker sensitivity, 86 dexmedetomidine, 759 percutaneous extraction, 91 levosimendan, 758 peri-operative and post-operative factors, 90 mini-CPB and zero-balanced ultrafiltration, 758 peri-operative and post-operative pacing, 88, 89 natriuretic peptide, 758 perioperative management, 87 prophylactic fenoldopam, 758 permanent pacemaker implantation, 89, 91 remote ischemic preconditioning, 758 postoperative bradycardia, causes of, 90 sodium bicarbonate, 758 sensitivity, 84 surgical strategies, 758 1066 Index

Cardiac surgery associated acute kidney injury (CSA-AKI) (cont.) circuit, 94 vasopressin, 759 coagulation and proinflammatory cascades, 101 volatile anesthesia, 758 conduct of KDIGO staging, 755, 756 anticoagulation, 783, 784 limitation, 755 arterial cannulation, 784 pathophysiological features, 756, 757 blood flow, 786 postoperative strategies femoral cannulation, 784–785 early use of RRT, 759 goal-directed-perfusion, 787, 788 extended daily dialysis, 759 initiation, 785, 786 KDIGO bundle, 759 pump flow/oxygen delivery, 786 prediction, 756 pump suckers, 788 preoperative strategies venous cannulation, 784 aspirin, 756 vents sucker, 788 preoperative erythropoietin (EPO) plus iron, 757 weaning, 788 renal function optimization, 756, 757 deep hypothermic circulatory arrest, 788 RIFLE and AKIN criteria, 755 diverting venous blood, 783 risk factors, 756 drained venous blood, 95 Cardiac Surgical Trials Network (CTSN), 374 key disposable components, 783 Cardiac tumors kinetic energy, 95 benign neoplasms microporous polypropelene hollow fiber membrane adult cellular rhabdomyoma, 677 oxygenators, 95 cardiac fibroma, 678 modified ultrafiltration, 790 cardiac lipoma, 678 monitoring, 95 cardiac myxoma, 679 myocardial injury, 101, 102 lipomatous hypertrophy of atrial septum, 678 pathologic oxidative balance, 101 papillary fibroelastoma, 675–676 pathophysiology of, 96, 97 rhabdomyoma, 676, 677 pediatric operations, 783 classification, 673 pediatric vs. adult, 789, 790 clinical symptoms, 673 roller pumps, 95 CT, 49 temperature and pH regulation, 96 incidence, 673 Cardio-pulmonary exercise testing, 640 MRI, 49 Cardio-Thoracic Surgery Network (CTSN) trial, 238 primary cardiac lymphomas, 685–687 CARE-HF trial, 608 primary cardiac sarcomas Carillon Mitral Contour System, 459 angiosarcoma, 682, 683 Carney complex (CNC), 680 leiomyosarcomas, 684 Carotid and coronary artery disease rhabdomyosarcomas, 685 ACT I trial, 278 undifferentiated pleomorphic sarcomas, 684 acute coronary syndrome, 278 reactive cardiac masses and pseudotumors asymptomatic disease, 278 calcifying amorphous pseudotumor, 675 chronic stable angina, 278 ectopias, 675 CREST trial, 278 germ cell tumors of the heart, 675 management of, 278–279 mesothelial pseudotumors, 675 myocardial reserve, 278 mural thrombi, 674 practical approach, 279 CardiAQ system, 459 procedural selection, 279 Cardioband technique, 458 routine carotid artery screening, 278 Cardiogenic shock, 615, 621, 631, 632, 634, 635, 637 synchronous and staged approaches, 278 Cardiomegaly, 853 traditional staged approach, 278 Cardiomyopathy, 60 Carotid artery anthracycline-induced cardiomyopathy, 1001 blunt carotid injury, 701 CT, 46, 47 penetrating injury, 700, 701 dilated cardiomyopathy, 1001 Carotid artery stenting (CAS), 278 hypertrophic cardiomyopathy, 1002 Carotid ultrasound, 277 MRI, 46, 47 Cascular compression syndrome, 990 restrictive cardiomyopathy, 1002 Catheter-based balloon (BAV), 437 Cardioplegia, 96 Catheter based therapies, 867 Cardioplegia administration techniques, 793–794 Central shunt, 816 Cardioplegic arrest, 641, 791–793 Central venous pressure (CVP), 10 Cardioprotection, 187 Centrally located squamous carcinoma, 691 Cardiopulmonary bypass (CPB), 70, 109, 149, 178, 713, 718 Centrifugal pumps, 625, 798, 799 aortic cross clamping, 101 CentriMag™ devices, 635, 636 arterial line filters, 95 Chavan-Haverich endograft, 546 cannulation strategies, 93–95, 376 Chordal sparing technique, 377 cardioplegic solutions, 96 Chronic lung allograft dysfunction (CLAD), 651 centrifugal pumps, 95 Chronic obstructive pulmonary disease (COPD), 626 cerebral autoregulation, 95 Chronic primary regurgitation, 373 Index 1067

Chronic relapsing/recurrent pericarditis, 704, 706 complications, 882 Chronic thromboembolic pulmonary hypertension (CTEPH), deep hypothermia, 882 717, 724 disadvantage, 882 Chronic type B aortic dissection (CTBAD) heart and great vessels, 882 aggressive anti-impulse therapy, 503 selective antegrade cerebral perfusion, 882 Candy-plug technique, 501, 502 VSD closure, 882 DISSECT classification, 499 morphology, 875 European Society for Vascular Surgeons (ESVS) guidelines, 498, in older children and adults, 882, 883 499 outcomes, 883 false lumen deployment, 502 pathophysiology, 875 false lumen occlusion techniques, 500, 501 Cold blood cardioplegia, 105, 310 false lumen thrombosis, 502, 504 Color flow Doppler, 24 fenestrated and/or branched stent-grafts, 500 Commissurotomy, 391 imaging, 497, 498 COMPANION trial, 608 incidence, 497 Complete heart block (CHB), 84, 965 indications, 499 Complex long-segment septal hypertrophy, 743 Knickerbocker technique, 502, 503 Computer tomogram (CT) angiography, 168 level of exercise, 504 Concomitant coronary bypass grafting, 239–240 open surgical repair, 499 Conduction system, 81, 906–907 quality of life (QOL), 504, 505 Congenital aortic valve stenosis stent graft induced new entry (SINE), 502–503 anatomy, 830 TEVAR, 499, 500 clinical presentation, 830 Chylothorax, 984 critical aortic stenosis Cinch® II implant system, 303 catheter intervention, 832 Cinch® implant system, 302 hybrid procedure, 832 Circulatory arrest, 719 long-term outcomes, 832 Circumflex aorta, 988 medical management, 831–832 Classic Blalock-Taussig shunt, 815 surgical intervention, 832 CLEAN-TAVI trial, 443 timing of intervention, 832 Cleveland Clinic Score, 756 diagnosis CNC1 susceptibility gene (PRKAR1A), 681 cardiac catheterization, 831 Coagulase-negative staphylococci (CoNS), 771 cardiac magnetic resonance, 831 Coagulation factor deficiency/dysfunction, 764 cardiopulmonary exercise testing, 831 Coarctation of the aorta chest radiography, 831 clinical presentation echocardiogram, 831 childhood/adolescent/adult, 876 non-critical aortic stenosis fetal, 875 catheter intervention, 833 infant, 876 long-term outcomes, 834 neonatal, 875–876 medical management, 833 definition, 875 surgical intervention, 834 definitive treatment, 879 timing of intervention, 833 diagnosis pathophysiology, 830 angiography, 878 Congenital heart anomalies catheterisation, 878 branch pulmonary arteries, 814 computed tomography scanning, 877–878 to enhance mixing echocardiography, 876 balloon atrial septostomy, 817 magnetic resonance imaging, 878 Blalock-Hanlon operation, 817 initial management open atrial septectomy, 817 child/adolescent/adult presentation, 879 palliative atrial switch operation, 817 fetal presentation, 879 hybrid palliation, 817, 818 infant presentation, 879 to increase pulmonary blood flow neonatal presentation, 879 bidirectional Glenn shunt, 816 left thoracotomy without CBP central shunt, 816 advantage, 880 classic Blalock-Taussig shunt, 815 alternative treatment, 880 ductal stenting, 817 complications, 880 modified Blalock-Taussig shunt, 815 disadvantage, 880 Potts shunt, 815 patch aortoplasty, 880 right ventricle-to-pulmonary artery shunt (), 816 resection and end-to-end anastomosis, 880 right ventricular outflow tract stenting, 817 resection and extended end-to-end anastomosis, 880 systemic-pulmonary shunt, 814 subclavian flap aortoplasty, 880 Waterston shunt, 815 surgical technique, 880 mechanisms, 813 long-term follow-up, 883 physiologic repairs, 813 median sternotomy with CBP pulmonary artery banding advantage, 882 complications, 814 alternative transcatheter approach, 882 reduction of pulmonary overflow, 813 1068 Index

Congenital heart anomalies (cont.) with isolated left ventricular outflow tract obstruction, 908–909 technique, 814 with isolated ventricular septal defect, 911–913 tetraining of left ventricle, 814 with pulmonary stenosis/atresia and ventricular septal defect, Congenital heart disease, 866 909–910 cardiac tumor, 1003 with tricuspid valvar insufficiency, 910–911 CT, 48 Conotruncal malformations, 869 indications, 1003 Constrictive pericarditis, 707, 711, 712 MRI, 48 Continuous blood gas monitor, 801 single ventricle pathway, 1002 Continuous in-line oxygen saturation monitors, 801 two-ventricle pathway, 1002 Continuous renal replacement therapy (CRRT), 759, 803, 805, 806 Congenital left ventricular outflow tract obstruction (LVOTO) Continuous veno-venous hemodialysis (CVVH), 636 definition, 993 Continuous wave Doppler, 24, 25 subvalvar aortic stenosis Conventional coronary artery bypass grafting (CABG) anatomy, 994 antiplatelet therapy, 154 aortoventriculoplasty procedures, 995–996 arteriotomy, 152 clinical presentation, 994 cannulation, 150 diagnosis, 994 conduit preparation, 152 four-stage aetiology, 993, 994 conduits used in, 151 indications, 994 de-airing and cross clamp removal, 153 Konno operation, 996 distal anastomoses, 152 modified Konno Operation, 995 graft configurations, 152–153 outcomes, 996 history of, 149 resection of fibrous shelf, 995 intracardiac and vascular cannulae, 151 Ross-Konno procedure, 996 indications for, 150 supravalvar aortic stenosis outcomes, 153 anatomy, 996, 997 patient selection algorithm for, 150 clinical presentation, 997 pericardium, 150 diagnosis, 997 proximal anastomoses, 153 diffuse supravalvar aortic stenosis, 998 retrograde and antegrade cannulae, 151 indications for surgery, 997 sternal closure, 153 outcomes, 998 sternotomy, 150 pathophysiology, 997 target identification and preparation, 151–152 single-patch (McGoon) technique, 998 Conventional ventilation or ECMO for Severe Adult Respiratory sliding aortoplasty, 998 Failure (CESAR) trial, 625 three-patch (Brom) technique, 998 Cooley technique, 248–249 two-patch repair (inverted Y-shaped) Doty technique, 998 Cooperative North Scandinavian Enalapril Survival Study COngenital Multicenter trial of Pulmonic vAlve regurgitation (CONSENSUS), 598 Studying the SAPIEN interventIONal (COMPASSION) CoreKnot®, 424 trial, 453 CoreValve implantation, 584 Congenitally corrected transposition of the great arteries (ccTGA) Coronary allograft vasculopathy (CAV), 1006 surgical pathology Coronary angiography, 8 atrioventricular valves, 906 Coronary arteries, 906, 907 conduction system, 906–907 Coronary arteriovenous fistulas (CAVF),see Coronary artery fistulas left ventricular outflow tract, 906 (CAF) ventricular septal defect, 906 Coronary artery aneurysms surgical treatment antiplatelet therapy, 283 alternative procedures, 913–914 classification, 281, 282 with isolated left ventricular outflow tract obstruction, 908–909 clinical presentation, 282 with isolated ventricular septal defect, 911–913 coronary artery ectasia, 282 with pulmonary stenosis/atresia and ventricular septal defect, definition, 281 909–910 diagnosis, 282–284 with tricuspid valvar insufficiency, 910–911 iatrogenic causes, 282 Congenitally corrected transposition of the great arteries (ccTGA) and obstructive coronary artery disease, 282 clinical presentation, 907 optimal medical therapy, 282 diagnosis, 908 pathogenesis, 282 isolated tricuspid insufficiency, 908 percutaneous therapy, 283 outcomes, 914 prevalence, 281 pulmonary stenosis, 908 risk factors, 282 surgical pathology surgical revascularization technique, 283 atrioventricular valves, 906 Coronary artery anomalies conduction system, 906–907 anatomical variations, 824 coronary arteries, 906, 907 anomalous left coronary artery from pulmonary artery, 823, 824 left ventricular outflow tract, 906 anomalous right coronary artery from pulmonary artery, 823, 824 ventricular septal defect, 906 characteristics, 821, 822 surgical treatment coronary artery fistula, 826, 827 alternative procedures, 913–914 Kawasaki disease, 821, 822 Index 1069

left coronary artery, anomalous aortic origin of, 824, 825 classification, 284 right coronary artery, anomalous aortic origin of, 824 clinical presentation, 285 sudden death, 824, 825 diagnosis, 285, 286 Coronary artery bypass grafting (CABG), 15, 43, 103, 207, 231, 377 epidemiology, 284, 285 adverse neurologic outcomes, 219 natural history, 285 aorta-vein graft anastomosis, 224 pathophysiology, 285 decisional algorithm for, 135 treatment, 286 distal anastomotic devices, C-Port Distal Anastomosis System, Coronary artery injury, 699 224–226 Coronary endarterectomy (CE) ELANA, 226 anticoagulation protocol, 255–256 grafting strategy and conduit choice, 134 double vessel endarterectomy, 257 Heartstring III proximal seal system, 220, 221 history of, 254 LITA, 131, 132 indications, 255 minimal-access and non-sternotomy revascularization, 220 LAD endarterectomy, 253 nitinol U-Clips, 226 off-pump coronary artery bypass grafting, 257 PAS-Port proximal anastomotic device, 223 on-lay patch, 257 perioperative stroke, 219 PTCA, 258 proximal anastomotic devices PTCRA, 257, 258 Heartstring proximal seal system, 220–222 safety of, 256–257 port proximal anastomosis system, 222–224 single endarterectomy, 257 radial artery, 132, 133 technique, 254, 255 right gastroepiploic artery, 133 Coronary ostia engagement, 6 saphenous vein, 133, 134 Coronary sinus cannulation, 610 surgical myocardial revascularization, 131 Coronary sinus venogram, 610, 611 traditional on-pump CABG, 219 Cox regression analysis, 959 Coronary artery disease (CAD), 43, 60, 72, 149 Cox-Maze (CM) procedure, 728, 729 ACS, 270 C-Port Distal Anastomosis System, 224–226 adhesions, 692 C-Port Flex-A (flexible) distal anastomotic device, 225 biologic deficiencies, 270 C-Port xA (rigid shaft) distal anastomotic device, 224, 225 cardiac subtypes, 270 Crohn’s disease, 64 cellular reprogramming, 270 Crown PRT™ valve, 299–301 choice of incision for access, 694 , 728 concomitant procedure, 692 CryoLife, 294, 295 CT, 43, 45 Crystalloid cardioplegia, 792 extent of pulmonary resection, 694 Crystalloid resuscitation, 764 extracorporeal circulation, 692, 693 CT cardiac angiography (CTCA), 43, 45 gene therapy Culture-negative infective endocarditis, 406 AAVs, 272 Custodiol™, 105 angiogenesis, 273 Custodial cardioplegia, 310 angiogenic clinical trial design, 271 Cyanoacrylates, 124 angiogenic factors, 271 Cyclic adenosine monophosphate (cAMP), 602 angiogenic gene therapy, 270 Cystic tumor of the atrioventricular (AV) node, 675 antifibrotic genes and downregulation, 271 Cytomegalovirus (CMV) infections, 668 cardiac cellular reprogramming, 273–274 challenges in, 274 emerging evidence, 271 D in situ cardiac cellular reprogramming, 271 Dacron vascular prosthesis, 546 initial gene therapy trials, 271 DeBakey classification system, 479, 488, 489 miRNA, 272 Deep hypothermic circulatory arrest (DHCA), 96, 521, 534, 788 plasmids, 271 Deep sternal wound infections (DSWIs), 204, 769–770 routes of administration, 272, 273 incidence, 771 stem cell applications, 270 risk factors, 772 VEGF, 270 treatment, 772 viruses, 272 Defibrillators, 84 MCS procedures, 269 DEFLECT trial, 443 MRI, 43, 45 Dehiscent mitral prosthesis, 28 off-pump coronary artery bypass grafting, 693 Del Nido cardioplegic solution, 792 patient work-up, 691 Delayed gadolinium enhancement (DGE), 47 prevalence, 691 Depressed ventricular function, 642 risk factors, 691 Descending thoracic aortic aneurysms (DTAA) staged procedure, 692 cardiopulmonary bypass stress echo, 37 cannulation techniques, 520, 521 therapeutic approach, 270 deep hypothermic circulatory arrest (DHCA), 521, 522 Coronary artery ectasia, 282 full cardiopulmonary bypass, 521 Coronary artery fistulas (CAF), 826, 827 partial cardiopulmonary bypass, 520, 521 2008 ACC/AHA Guidelines, 285 partial left heart bypass, 521 1070 Index

Descending thoracic aortic aneurysms (DTAA) (cont.) Dual chamber pacemaker, 85 cerebrospinal fluid drainage, 519 Ductal stenting, 817 classification, 516, 518 Ductus arteriosus, 892 diagnosis, 516, 517 Duke Activity Status Index, 263 endovascular treatment, 517 Dyspnea, 245 epidemiology, 516 etiology, 515, 516 incision, 519, 520 E medical management, 516 EACTA/ASE protocol, 38 morbidity and mortality, 515 Ebstein’s anomaly (EA), 415 open surgical treatment, 517 anatomical , 971 preoperative planning and set-up, 519, 520 anatomy, 972 preoperative testing, 519 associated atrial level arrhythmias, 971 prognosis, 516 associated cardiac defects, 971 repair technique, outcomes, 524, 527 clinical presentation, 971 risk factors, 516 diagnostic evaluation Desensitization strategy, 656 cardiac catheterization, 974 Dexmedetomidine, 759 chest X-ray, 972, 973 Diagnostic coronary angiography, 5 computed tomography, 974 Diagonal pumps, 803 echocardiography, 973, 974 DiGeorge syndrome, 893, 934 , 973 Digoxin, 601, 602 magnetic resonance imaging, 974 Dilated cardiomyopathy (DCM), 46, 1001 functional pulmonary atresia, 971 Discontinuing heparin, 764 outcomes, 979 Diuretics, 601, 602 patent foramen ovale/atrial septal defect, 971 Dobutamine, 75, 602 pathophysiology, 972 Dobutamine stress echocardiography (DSE), 17, 57 prevalence, 971 Dominant left arch, 983 surgical indications, 974–975 Donor after circulatory death (DCD) setting, 641 surgical procedures Door to Unloading with IMPELLA CP System in Acute Myocardial bicaval cannulation, 976 Infarction (DTU) study, 632–633 biventricular repair, 976–979 Dopamine, 75 cone reconstruction, 976 , 23 Monocusp repair, 976–978 Dor technique, 249, 250, 749, 750 neonatal EA, 975 DOSE trial, 603 principles, 976 Double antiplatelet therapy (DAPT), 255 Sebening stitch, 975 Double aortic arch, 981, 983, 984 single ventricle palliation, 976 Double outlet right ventricle (DORV) Starne’s procedure, 976 classification, 963 symptomatic neonate with EA, 974, 975 clinical presentation, 964 ECG-gating, 42 definition, 961, 962 Echocardiography (Echo), 56 diagnosis, 964 contrast echocardiography, 37 embryology, 962 infective endocarditis, 34, 35 genetic/chromosomal disorder, 962 masses, 36 history, 961 modalities of, 24 outcomes, 967, 968 color flow Doppler, 24 pathophysiology M-mode, 24 centricular septal defect and great artery relation, 963 PW and CW Doppler echocardiography, 24 patent ductus arteriosus, 963 strain imaging, 24, 25 pulmonary vascular resistance, 963 TDI, 24 physical signs, 964 3D echo, 25 postoperative management myocardial function cardiac arrhythmias, 965 diastolic function, 29, 30 low cardiac output syndrome, 965 left ventricular systolic function, 26–29 postoperative bleeding, 966 right ventricular systolic function, 29 pulmonary hypertension, 965 non-invasive diagnostic imaging test, 23 residual lesions, 965 pericardial disease, 35–36 preoperative management, 964–965 stress echo, 37 surgical management, 966, 967 surgically-relevant information, 30 ventricular septal defect TEE, 38, 39 doubly committed VSD, 963 ultrasound theory, 23 non-committed VSD, 962 valve pathology assessment subaortic VSD, 962 aortic regurgitation, 31 subpulmonic VSD, 962 aortic stenosis, 30 Doubly committed ventricular septal defect, 860 mitral regurgitation, 33, 34 Dual antiplatelet therapy, 65 mitral stenosis, 31, 33 Index 1071

pulmonary valve, 34 ARDS, 625 tricuspid valve assessment, 34 cardiac support, 626 Echo-CRT trial, 609 centrifugal pumps, 625 Ectopias, 675 circuit configuration, 623, 624 ™ EDWARDS INTUITY valve system, 426, 427 CO2 removal, 625 Edwards SAPIEN transcatheter , 448–452 complications, 627 Effusive pericarditis, 706, 707 contraindications, 627

Effusive-constrictive pericarditis, 707 ECCO2R, 626 Ehlers-Danlos syndromes, 31, 318 ECPR, 626, 627 Eisenmenger syndrome, 646, 851 gas exchange, 624, 625 Electrical dyssynchrony, 608 hypercarbic respiratory failure, 626 Electrocardiogram (ECG), 56 hypoxemic respiratory failure, 625, 626 Electrocardiogram (ECG)-synchronized pulsatile flow, 803 life-limiting comorbidities, 627 Electron-beam CT (EBCT), 246 long term outcomes, 627 Electrophysiologic mapping, 728 LTVV, 625 Electrophysiological evaluation, 738 in pediatric population, 623 Elephant trunk procedure (ET), 539, 540 primary graft dysfunction after lung transplantation, 626 anesthetic management, 574 Extracorporeal Life Support Organization (ELSO) registry, 623 cardiopulmonary perfusion and brain protection, 574 Extracorporeal membrane oxygenation (ECMO), 93 classification, 574 cannulation evolution of, 573, 574 access sites, 801 first stage, 575, 576 insertion technique, 801 frozen elephant trunk, 577, 578 circuit hybrid aortic arch repair, 545 bladder reservoir, 801 outcomes, 578 blood pump, 798 postoperative surveillance, 577 cannulae, 798, 800 preoperative evaluation, 574 continuous blood gas monitor, 801 second stage, 576, 577 continuous in-line oxygen saturation monitors, 801 Emboli Detection and Classification (EDAC) system, 805 oxygenator, 798, 799 Embolic phenomenon, 679 tubing, 799, 800 Embryonic branchial arch regression, 865 clinical outcomes, 802 ENABLE™ valve system, 426, 427 complications, 802 Enalapril, 598 contraindications, 798 (EMB), 660, 1005 definition, 797 Endoscopic saphenous vein indications, 798

CO2 insufflation, 139 management complications, 139 anticoagulation, 801 evidence, 139–141 blood flow, 801 patient selection, 141 daily management, 801 surgical technique, 141 priming solution, 801 endoscopic radial artery harvesting, 141–144 weaning off, 802 endoscopic saphenous vein harvesting, 144–147 pediatric MCS Endoscopic vein harvesting (EVH), 162, 208 adverse events, 811 Epicardial left atrial appendage exclusion devices, 731 complications, 808 Epicardium, 703 devices and patients, 808 Epinephrine (adrenaline), 74 Jostra Rotaflow centrifugal pump, 807 Eplerenone in Mild Patients Hospitalization and Survival Study in long term, 808 Heart Failure (EMPHASIS-HF) trial, 601 outcomes, 807 Eplerenone Post-Acute Myocardial Infarction Heart Failure Efficacy Pedimacs, 808 and Survival Study (EPHESUS), 601 short-term, 807 Epstein-Barr virus (EBV), 1006 survival post-VAD implantation, 808 Erbium:YAG, 262 Tandem Heart, 807 European and American CRT guidelines, 609 Thoratec PediMag, 807 European Society for (ESVS), 488 translational research EuroSCORE, 438, 615 continuous renal replacement therapy, 803, 805, 806 EuroSCORE II, 438 hemoconcentrators, 803 E-vita Jotec hybrid prosthesis, 548–550 pulsatile ECMO, 803, 804 Excimer coronary laser (ECLA), 258 venoarterial, 798 Excimer laser-assisted non-occlusive anastomosis (ELANA), 226 venovenous, 798 Extended daily dialysis (EDD), 759 Extended myectomy technique, 742 Extra corporeal membrane oxygenation (ECMO), 231 F

Extracorporeal carbon dioxide removal (ECCO2R), 625, 626 False aneurysm, 244 Extracorporeal cardiopulmonary resuscitation (ECPR), 626, 627 Femoral artery insertion, 616 Extracorporeal circulation (ECC), 692, 693 Femoral cannulation, 784–785 Extracorporeal life support (ECLS) Fibrin sealants, 123 1072 Index

Fibroblast growth factors (FGFs), 273 HFpEF (see Heart failure with preserved ejection fraction Fick’s Law, 786 (HFpEF)) Finger-thumb technique, 540 HFrEF (see Heart failure with reduced ejection fraction (HFrEF)) Floseal®, 125 MRI, 46, 47 Fluorescent in situ hybridisation (FISH), 934 Heart failure with preserved ejection fraction (HFpEF) Fontan/total cavo-pulmonary connection (TCPC), 929 ACEI and ARBs, 602 Fractional flow reserve (FFR), 215 beta blockers, 601 COMPARE ACUTE trial, 19 calcium blockers, 602 cut-off value, 17 digoxin, 602 DEFER trial, 19 diuretics, 602 definition, 16, 17 effective therapeutic approach, 601 FAME 2 trial, 19 Heart failure with reduced ejection fraction (HFrEF) FAME trial, 19 ACEIs, 598–600 iFR, 16, 20 beta-blockers, 598–600 IVUS, 18 cause of, 597 LMS stenosis, 18 H-ISDN, 601 non-invasive diagnostic techniques, 17 ivabradine, 601 adenosine stress computed tomography myocardial perfusion mineralocorticoid receptor antagonists, 601 imaging (CTP), 17 pathophysiology of, 597 dipyridamole technetium-99m sestambi, 17 RAAS inhibitors, 598 MRMPI, 17 randomised controlled clinical trials, 598–600 myocardial perfusion imaging, 17 Heart Team concept, 438 pre coronary artery bypass grafting, 18–19 Heart transplantation (HTx) pressure wire measurements, 18 brain stem death, 639 Fractional shortening, 26–27 causes of death, 666, 667 Frank-Starling Law, 27 clinical criteria, 639 Free right internal thoracic artery (FRITA), 208 complications FREEDOM trial, 214 acute rejection, 642 FreeFlo design, 499 cardiac allograft vasculopathy, 642 Freestyle bioprosthesis, 347 malignancy, 642 Fremes’ solution, 105 primary graft dysfunction, 642 French Cuff technique, 307, 311 donor criteria, 640 Frozen elephant trunk (FET) technique, 545 early complications Functional aortic annulus (FAA), 326–328 AKI, 668 Functional pulmonary atresia, 971 cytomegalovirus (CMV) infections, 668 hyperacute rejection, 668 non-cytomegalovirus (CMV) infections, 668 G PGD/PGF, 666 Gamma amino butyric acid (GABA), 66 pulmonary hypertension, 667 Ganzoni equation, 64 RV function, 667 Gerbode’s defect, 851 surgical complications, 668 Germ cell tumors of the heart, 675 ex-situ perfusion, 642 Gianturco stainless steel Z-stent segments, 499 history, 639 Goal-directed-perfusion (GDP), 787, 788 incidence, 639, 667 Goretex® conduit, 929 ISHLT registry data, 666 Graft vascular disease, 665 late complications Graft-versus-host (GVHD), 651 anti-hypertensive therapy, 669 Great Ormond Street Echocardiogram (GOSE) Score, 974 cardiac allograft vasculopathy, 669 malignancy, 669 renal dysfunction, 668 H steroid therapy, 669 Harken Soroff Valve, 291 long-term side effects, 665 Harpoon Medical device, 457 organ allocation systems, 640, 641 Heart pre-operative selection and early donor management, 639 blunt cardiac injury, 699, 700 procedure and technical considerations coronary injuries, 699 haemodynamic and clinical outcomes, 641, 642 penetrating cardiac injury, 699 organ implantation, 641 surgical approach, 699 organ retrieval, 641 Heart failure (HF) recipient criteria, 639, 640 causes, 607 rejection, 665 classification of, 597 risk factors and contraindications, 640 CT, 46, 47 triple immunosuppressive therapy, 665 definition, 597, 607 Heart-lung transplantation (HLTx) digoxin, 601 complications diuretics, 601 acute rejection, 649, 650 epidemiology, 607 airway complications, 651 Index 1073

cardiac allograft vasculopathy, 651, 652 indications, 547 chronic lung allograft dysfunction, 651 OPEN PLUS® Hybrid stent-graft prosthesis, 547 graft-versus-host disease, 651 preprocedural examination, 547 history, 645, 646 TEVAR procedure, 548 indications Vascutek Thoraflex® hybrid stent-graft, 547 combined cardiac and pulmonary disease, 646 zones 0, 548 Eisenmenger syndrome, 646 zones 1, 548 uncorrectable congenital heart disease with atresia/hypoplasia, 646 zones 2, 548 postoperative management, 649 zones 3, 548 selection criteria Hybrid coronary revascularization (HCR), 181 aortopulmonary collaterals, 647 LITA grafting, 193 previous median sternotomy/thoracotomies, 647 LITA-to-LAD graft, 193 recipients age, 647 medical therapy, 193 survival, 652, 653 MIDCAB, 194, 198 technique patient positioning, 195 donor procedure, 647–648 patient selection, 194 recipient procedure, 648, 649 PCI, 193 Heartstring III proximal seal system, 220, 221 rationale for, 194 Heartstring proximal seal system, 220–222 robotic approaches, 194 Hemi-Mustard modification, 911 technique, 194, 195 Hemoconcentrators, 803 timing of interventions, 197 Hemodynamic cardiac catheterization, 712 Hybrid Norwood approach, 927 , 803 Hybrid palliation, 817, 818 Hemolung Respiratory Assist System (Hemolung RAS), 626 Hydralazine and isosorbide dinitrate (H-ISDN), 601 Heparin-induced thrombocytopenia (HIT) Hyperacute rejection, 668 alternative non-heparin anticoagulants, 114 Hyperkalemia, 794 bivalirudin, 115 Hyperkalemic crystalloid cardioplegia, 102 in children, 116 Hypertrophic cardiomyopathy (HCM), 37, 46, 1002 clinical picture, 111 apical myectomy, 736 clinical prediction rule, 112 dynamic obstruction, 735 clinico-pathological syndrome, 110 impact, 735 CPB, 109 long-term results, 745–747 diagnosis, 111, 112 LVOT obstruction, 735 extracorporeal membrane oxygenation, 109 with nonobstructive HCM, 736 frequency of, 111 operative methods hemoperfusion, 109 apical hypertrophy/midventricular obstruction, 743 left heart bypass, 109 complex long-segment septal hypertrophy, 743 management of, 114–115 induction of anesthesia, 741 pathogenesis, 110 myectomy, 742–744 PF4, 110 myocardial bridging, 744 plasmapheresis, 115, 116 patient evaluation, 741 platelet inhibitors, 115 peak-to-peak left ventricular outflow tract gradient, 741, 742 scoring system, 112 SAM-related mitral regurgitation, 743 timing of, 110, 112 perioperative results, 745 treatment, 113 postbypass management, 744 Hepatocyte growth factor (HGH), 273 postoperative management, 745 Hibernating myocardium, 55, 56 preoperative evaluation High-dose glucocorticoids, 658 age thresholds, 738, 741 His bundle pacing, 86 atrial fibrillation, 738 Histidine-tryptophan-ketoglutarate (HTK), 104 cardiac magnetic resonance, 738 Ho:YAG lasers, 262 complete heart block, 738 Hufnagel valve, 291 electrophysiological evaluation, 738 Human leucocyte antigens (HLA), 641 exertional chest pain or pressure, 738 Hybrid aortic arch repair flowchart, 736, 739 Chavan-Haverich endograft, 546 late gadolinium enhancement, 738 clinical investigation, 547 latent obstruction, 736 clinical outcomes, 550 mitral valve procedures, 736 Criado classification, 548 severe centrally directed mitral regurgitation, 736, 737 CT scan, 548 subaortic septal thickness, 736, 738, 740 Dacron vascular prosthesis, 546 sudden cardiac death, 738 elephant trunk technique (ET), 545 transesophageal echocardiogram, 736 E-vita Jotec hybrid prosthesis, 548–550 transthoracic echocardiography, 736 four-branched hybrid prosthesis, 545 unroofing, 738, 741 frozen elephant trunk (FET) technique, 545–547 prevalence, 735 hemiarch/full arch replacement, 545 subaortic myectomy, 736 hybrid stented graft prosthesis, 545 transaortic , 735 1074 Index

Hypertrophic cardiomyopathy-related death, 746 diagnosis, 398 Hypocalcemia, 794 incidence, 397 Hypoperfusion, 74 indications, 398 Hypoplasia, 832 intravenous antibacterial therapy, 398 Hypoplastic aortic arch, 938 medical therapy, 398 Hypoplastic left heart syndrome (HLHS), 924 mitral repair, 400 anatomy, 924 of mitral valve, 397 clinical implications, 924 mortality rate, 397 future aspects, 930 para-annular extension, 398 late outcomes, 930 surgical principles, 399 morphology, 923, 924 systemic embolization, 399 neonatal management Inferior vena cava (IVC), 34 degree of cyanosis, 925 Inhaled nitric oxide (iNO), 76 diagnosis, 925 Innominate artery differential diagnosis, 925 blunt innominate artery injury, 700 ductal patency, 924 penetrating injury, 700 hybrid procedure, 927 Inotropes, vasopressors and vasodilators Norwood operation, 925–926 active pharmacological agents, 74 outcomes, 927 acute on chronic heart failure, 77 prostaglandin E2 infusion and supportive measures, 924 circulatory volume status, 70 pulmonary artery conduit vs. Blalock-Taussig shunt, 926–928 coronary artery bypass graft surgery, 77 Norwood procedure, 923 inotropic support, 70 second stage procedure, 928, 929 LV hypertrophy/severe diastolic dysfunction, 77 third stage procedure, 929, 930 orthotopic cardiac transplantation, 77 transplantation, 930 phosphodiesterase-type III inhibitors, 75 Hypothermia, 764, 793 practical dilemmas, 70 Hypothermic cardioplegia, 102 pulmonary circulation Hypothermic circulatory arrest (HCA), 480 iNO, 76 Hypoxia-induced factor alpha (HIF-α), 273 phosphodiesterase-type V inhibitors, 76 prostacyclin pathway, 76 RV dysfunction, 76 I RV function, 76 IABP-SHOCK II trial, 615 right ventricular dysfunction, 77 Iloprost, 76, 77 sympathomimetic drugs Immunosuppression, 649 AVP, 75, 76 cardiac allograft vasculopathy, 661 dobutamine, 75 corticosteroid withdrawal, 659 dopamine, 75 desensitization strategy, 656 epinephrine (adrenaline), 74 drug-drug interactions, 660 levosimendan, 75 eculizumab, 661 norepinephrine (noradrenaline), 74 goal of, 655 treatment strategies and inotropes, 71 imlifidase, 661 valvular surgery, 77 induction therpay, 656, 657 vascular resistances and vasoplegia, 71, 72 maintenance therapy, 657, 1005 vasoactive support, principles of, 72–74 medications ventricular dysfunction, 70 azathioprine, 658 ventricular preload, 70, 71 calcineurin inhibitors, 659 In-situ right internal mammary artery, 202 glucocorticoids, 658 Instantaneous wave-free pressure ratio (iFR), 16, 20 mycophenolic acid, 659 INSTEAD trial, 490 proliferation signal inhibitors, 659 Integrative viruses, 272 pre-transplant, 1005 Intermittent warm blood cardioplegia, 105, 106 rejection Internal mammary artery (IMA), 199 acute cellular rejection, 660–661 Internal thoracic artery, 131, 132 EMBx concordance rate, 660 International Collaboration on Endocarditis–Prospective Cohort Study renal sparing, 660 (ICE-PCS), 407 standard triple therapy, 659 International Registry of Acute Aortic Dissection (IRAD), 487, 497 Impella® devices, 632–634 Interrupted aortic arch (IAA), 893 Impella-EUROSHOCK-Registry, 632 anatomy, 933, 934 Implantable cardioverter defibrillator (ICD), 81, 87, 640, 746 clinical features, 934 Induction therpay, 649, 656, 657, 1005 diagnosis, 934 Infective endocarditis (IE), 34, 35 hypoplastic aortic arch, 938 anatomy, 397, 398 LVOT obstruction, 938 of aortic valve (AV), 397 outcomes, 936–938 aortic valve reconstruction, 397, 400, 401 postoperative care, 936 aortomitral curtain (AMC) reconstruction, 400–402 preoperative management, 934 congestive heart failure (CHF), 398 prostaglandin E1 (PGE1) treatment, 933 Index 1075

surgical management Ischemic mitral valve regurgitation (IMR) arch advancement technique, 935 autologous stem-cell based treatments, 241 arch reconstruction, 936 chronic ischemic mitral regurgitation, 238 arterial cannulation site, 935 concomitant coronary bypass grafting, 239–240 bicaval cannulation, 935 diagnosis and assessment, 237, 238 bypass techniques, 935 epidemiology, 237 complete deep hypothermic circulatory arrest, 935 indications for, 238–239 hybrid palliation, 935 medical therapy for, 238 left carotid artery turndown bridges, 935 mitral valve repair, 239 ligation, 935 mitral valve replacement, 239 logical and systematic fashion, 936 pathophysiology of, 237, 238 LVOT obstruction, 936 percutaneous treatments, 240, 241 selective cerebral perfusion, 935 randomized controlled trials, 240 systemic ventricle or ejection pathway, 936 Ishimaru landing zone classification, 541 VSD closure, 936 Isolated ventricular septal defect (VSD) Intra-aortic balloon pump (IABP) aortic insufficiency, 861 axillary artery insertion, 616, 617 clinical features, 851–852 complications, 617 closure, 856 console problems management, 620 components, 850 contraindications for, 615, 616 epidemiology, 849 femoral artery insertion, 616 Gerbode’s defect, 851 haemodynamic function, 614 hybrid approach, 861 hemodynamic effects, 615 investigations indications for use, 615 chest X-ray, 852, 853 limb ischemia, 620 echocardiography, 852–855 poorly synchronised balloon inflation 12-lead electrocardiography, 852 diastolic augmentation, 619 juxta-arterial/doubly committed subarterial defect, 851 early balloon deflation, 618, 619 left and right ventricles, 850 early balloon inflation, 617 medical management, 855 late balloon deflation, 618, 619 muscular, 850–851 late balloon inflation, 617, 618 non-surgical intervention supply and demand, 614 closure of isolated VSD, late effects, 862 systemic anticoagulation, 617 muscular VSDs, percutaneous device closure, 861 weaning, 620, 621 pacemaker implantation, 862 Windkessel effect, 613 pathophysiological effects, 851 Intracardiac echocardiogram (ICE), 840, 841 perimembranous, 850 Intracellular-type cardioplegia, 792 pulmonary artery banding, 861 Intramural hematoma (IMH) residual or additional VSD, 861 clinical features, 509 surgical intervention diagnosis, 510 continuous vs. interrupted sutures, 859 endovascular repair, 511, 512 doubly committed VSD, 860 epidemiology, 508, 509 indicaions, 856 medical management, 510 muscular, 860 open surgical repair, 511 perimembranous, 856–860 pathophysiology, 507, 508 TV apparatus detachment, 860 (IVUS), 18 Isovolumic contraction time (ICVT), 28 Intrinsic MV disease, 736 Isovolumic relaxation time (IVRT), 28 Invasive catheter angiogram (ICA), 43 Ivabradine, 601 Invasive coronary angiography indications, 4 post-procedure care J complications, 8, 9 Jacky and Sarah (Terumo®), 5 coronary angiogram analysis, 8 Jatene technique, 249, 250 sheath removal and closure devices, 8 Jostra Rotaflow centrifugal pump, 807 pre-procedure preparation, 4 Jugular bulb oximetry, 95 technical aspects Junctional ectopic tachycardia (JET), 965 access, 4 Juxta-arterial, or doubly committed subarterial defect, 851 angiographic views, 6 catheter selection and manipulation, 5 pharmacology, 5 K Investigation of Stent Grafts in Aortic Dissection (INSTEAD) Trial, Kaplan-Meier curves, 338 555 Kaplan-Meier survival analysis, 526 Iron-deficiency anemia, 64 Kawasaki disease, 821, 822 Ischaemia–reperfusion injury, 757 Kidney Disease Improving Global Outcomes (KDIGO) criteria, 755 Ischemic cardiomyopathy, 60, 61 Kimny (Boston Scientific®), 5 Ischemic heart disease (IHD), 74, 84 Knickerbocker technique, 502, 503 1076 Index

Kommerell diverticulum, 983 Loeys-Dietz syndrome (LDS), 318, 476 Konno-Rastan techniques, 349 AHA 2010 guidelines, 587 Kussmaul sign, 710 aortic dissection, 591 arterial tortuousity, 592 cardiomyopathy, 592 L initial reports, 589 Labile outflow tract obstruction, 736 mitral valve disease, 591, 592 Laplace’s law, 749 neuroanatomic abnormalities, 592 Lariat suture delivery system, 730, 731 non-syndromic presentations, 587 Late gadolinium enhancement (LGE), 738 prophylactic surgery, 589 Late sternal SSI, 771 TGFBR2 mutation, 589 Late sternal wound infections, 773 TGFβ receptor mutations, 589 Latent obstruction, 736 valve-sparing root replacement, 589 Lateral mitral annulus, 26 Loop-sampling systems, 66 12-lead electrocardiography, 852 Low cardiac output syndrome (LCOS), 965 Leadless pacing systems, 86 Low-molecular-weight heparin (LMWH), 110, 764 Leaman score, 8 LV hypertrophy/severe diastolic dysfunction, 77 Lecompte maneuver, 901 Lyme disease, 84 Left anterior descending (LAD) artery, 10, 193, 199–200 Left anterior oblique (LAO), 6 Left aortic arch, 984 M Left atrial appendage exclusion, 730, 731 Magna Ease™ valve, 301–302 Left bundle branch block (LBBB), 608 Magnetic resonance imaging (MRI) Left circumflex artery (LCx), 8 CAD, 43, 45 Left coronary angiography, 6 cardiac tumours, 49 Left coronary artery system, 7 cardiomyopathies, 46, 47 Left internal mammary artery (LIMA), 157 clinical applications, 44 Left internal thoracic artery (LITA), 131, 168, 196, 207–209, 212 cochlear implantsintra-ocular foreign bodies, 43 Left main stem (LMS) disease, 10 congenital heart disease, 48 Left main stem (LMS) stenosis, 18 different radiofrequency waves, 42 Left pulmonary artery, 721 gadolinium molecule, 43 Left ventricle (LV), 850 heart failure, 46, 47 Left ventricle to pulmonary artery (LV-PA) valved conduit, 909 insulin pumps, 43 Left ventricular (LV) cineangiogram, 246 myocardial ischaemia, 45, 46 Left ventricular aneurysm, 243 myocardial viability, 46 Left ventricular angiography, 6 neurostimulators, 43 Left ventricular assist device (LVAD), 74 pericardial diseases, 48 Left ventricular ejection fraction (LVEF), 26, 608, 609 post-surgical patient, 51 Left ventricular outflow tract, 906 signal intensity, 42 Left ventricular outflow tract obstruction (LVOTO), 899, 958, 959 T1 and T2 relaxation times, 42 Left ventricular pseudoaneurysm, 243 valvular heart disease, 47, 48 causes of, 244 Maintenance immunosuppression, 1005 Left ventricular systolic dysfunction, 55 Maintenance therapy, 657 Left ventricular systolic function Major adverse cardiovascular events (MACE), 168, 224, 632 regional/segmental contractile abnormalities, 29 Malignant fibrous histiocytoma, 684 systolic strain, 28 Manougian technique, 408 volumes/dimensions changes, 26–28 Marfan syndrome (MFS), 317, 476, 493 Left ventricular volume reduction (LVVR) acute aortic dissection (AAD), 590 aneurysmal portion, 749 AHA guidelines, 589 Dor technique, 750 cardiomyopathy, 592 linear septoexclusion, 750 clinical criteria and mutation analysis, 589 outcomes, 753 differential diagnosis, 587 overcoat technique, 750 elective root replacement, 589 secondary mitral regurgitation, 752–753 ESC 2014 guidelines, 587 STICH trial, 753 interventions on distal aorta, 591 volume-related or shape-related, 750, 752 mitral valve disease, 591, 592 volume-related technique, 749 prophylactic surgery, 589 Left ventriculography, 246 TGFBR2 mutation, 589 Leiomyosarcomas, 684 valve-sparing root replacement, 589 Lentiviruses, 272 VSRR failure, 589 Less extensive lesion sets, 729 Massive transfusion protocols (MTP), 765 Levosimendan, 75, 602 McCarthy technique, 250 Limb ischemia, 111 Mean arterial pressure (MAP), 70 Linear septoexclusion, 750 Mechanical circulatory support (MCS), 269, 616, 641 Linx™ anticalcification technology, 301 Mechanical synchrony, 608 Lipomatous hypertrophy of the atrial septum, 678 Mechanical valve replacement, 834 Index 1077

Median sternotomy, 158, 713 history, 429, 430 Mediastinal pleura, 984 patient approval, 429 Medtronic, 295 patient positioning, 431 Medtronic CoreValve Evolut R THV, 439 patient selection, 430 Medtronic MELODY transcatheter pulmonary valve, 448–450 periareolar approach, 434 Mehta Score, 756 preoperative screening, 430, 431 Melody valve, 448 repair technique Membranous and muscular atresia, 942 advantages and disadvantages, 434 Mesothelial/monocytic incidental cardiac excrescence, 675 Alfieri’s stitch, 432, 433 Mesothelial pseudotumors, 675 annuloplasty, 432 Metoprolol CR/XL Randomised Intervention Trial in Congestive complications, 434 Heart Failure (MERIT-HF), 598 neochordae resection, 432, 433 Micra (Medtronic), 86 quadrangular resection, 432 Microporous polypropelene hollow fiber membrane oxygenators, 95 short- and long-term mortality, 433 MicroRNA (miRNA), 272 triangular resection, 432 Microsample point-of-care tests, 66 surgical intervention, 429 Midventricular obstruction, 736 surgical setup, 431, 432 Mildly dysplastic valve, 919 Ministernotomy Mild-moderate pulmonary stenosis, 919 advantages, 424 Milrinone, 602 central aortic cannula removal, 425 Mineralocorticoid receptor antagonists, 601 central aortic cannulation, 424 Miniaturised cardiopulmonary bypass, 97 CPB cannulation, 424 Mini-Bentall procedure, 308 CT scan rules and selection, 426 Minicardioplegia method, 793 cumulative sum (CUSUM) analysis, 426 Minimally invasive aortic valve replacement (MIAVR), see J sternotomy, 424 Ministernotomy myocardial protection, 424 Minimally invasive aortic valve surgery (MIAVS) neurological complications, 426 advantage, 422 postoperative complications, 426 AHA definition, 422 prospective randomized trial, 426 CT scan, 422, 423 retrograde cardioplegia, 424, 425 preoperative anaesthetic considerations, 423, 424 right anterior minithoracotomy approach, 424–426 preoperative assessment, 422 single surgeon’s learning curve (M.G.), 426 preoperative planning, 422 surgical failure, 426 rehabilitation resources, 422 with sutureless valves, 426, 427 right anterior minithoracotomy approach, 422, 423 T or V-shaped ministernotomy, 424 routine preoperative evaluation tests, 422 transverse aortotomy, 424 Sealant devices, 423 venous cannulation, 424, 425 STS database, 421 Mini-sternotomy, 308–310 Minimally invasive conventional coronary artery bypass grafting MISTRAL-C, 443 absolute and relative contraindications for, 170 Mitochondrial permeability transition pore (MPTP), 102 angiographic patency, 168 MitraClip device, 240, 241, 456 beating heart, 181 MitraFlex, 457 BITA, 168 Mitral and tricuspid prosthetic valve endocarditis, 411 CTA, 168 Mitral regurgitation (MR), 33, 34 HCR, 181 Accucinch, 459 Kaplan–Meier curves, 168 Amend device, 459 left anterior descending artery, 172 anatomical changes, 455 literature on, 169 Arto System, 459 long-term follow-up, 169 Carillon Mitral Contour System, 459 median time, 168 chordal approach, 457 myocardial protection, 178 direct annuloplasty, 458 OPCAB grafting, 168 indirect annuloplasty, 459 PCI, 167 leaflet repair post-operative morbidity, 168 Alfieri edge-to-edge repair, 455 proximal anastomosis, 171 CLASP trial, 456 redo-minimally invasive CABG, 168 clinical development, 456 small left thoracotomy, 168 EVEREST-I randomized trials, 456 starfish, 172 EVEREST-II randomized trials, 456 sternotomy, 168 24-French (Fr) delivery catheter, 456 technique, 170–172 MitraClip®, 456 Minimally invasive cardiac surgery (MICS), 844 Percu-Pro device, 457 Minimally invasive direct coronary artery bypass grafting (MIDCAB), Millipede, 459 194 Mitralign, 459 Minimally invasive mitral valve (MV) percutaneous leaflet repair contraindications, 430 clinical development, 457 definition, 430 MitraClip®, 456 1078 Index

Mitral regurgitation (MR) (cont.) physiologic fixation, 303 percutaneous MV repair, 455 Mosaic Ultra™ valve, 301 prevalence, 455 AOA® tissue treatment, 302 stress echo, 37 durability, 302 TMVI (see Transcatheter MV implantation (TMVI)) hemodynamic performance, 302 TMVr annuloplasty, 457 Physiologic Fixation process, 302 transcatheter valve interventions, 455 Motion-mode (M-mode), 24 VenTouch system, 459 Multicenter Automatic Defibrillator Implantation Trial-Cardiac Mitral stenosis, 31, 33 Resynchronization Therapy (MADIT-CRT) trial, 608 Mitral valve disease, 10 Multicentre InSync Randomised Clinical Evaluation (MIRACLE) AHA/ACC guidelines, 374, 375, 381 trial, 608 chronic primary regurgitation, 373 Multidetector-row CT (MDCT), 246 echocardiographic evidence, 373 Multi-dose cold cardioplegia, 793 echocardiographic measurements, 374 Multi-dose warm cardioplegia, 793 etiology, 373 Multi-planar reformats (MPR), 41 minimally invasive mitral repair, 381 Multiple arterial bypass grafting (MABG), 207, 214 postoperative management, 378 Multisite Stimulation in Cardiomyopathy (MUSTIC) trial, 608 prevalence, 374 Multi-testing sequential Bayesian approach, 17 repair technique Mural thrombi, 674 annuloplasty, 384–386 Muscle flap, 772 assessment, 386 Muscular ventricular septal defects, 850–851, 860 indications, 386 Mycophenolate mofetil (MMF, Cellcept®), 659 long-term survival, 386 Mycophenolic acid, 659 mini-thoracotomy, 387 Myocardial bridging, 744 polytetrafluoroethylene (PTFE) neochordae placement, 383 Myocardial contrast echocardiography, 60 quadrangular resection, 382, 384 Myocardial deterioration, 101 triangular resection, 382, 383 Myocardial dysfunction, 640 repair vs. replacement, 374–375 Myocardial ischaemia rheumatic mitral valve repair (see Rheumatic mitral valve repair) CT, 45, 46 Sondergaard’s groove, 376 MRI, 45, 46 surgical technique, 376 Myocardial protection anterolateral right mini-thoracotomy, 382 cardioplegia administration techniques, 793–794 cardiopulmonary bypass cannulation, 376 cardioplegic arrest, 791–793 chordal sparing technique, 377 cardioplegic solutions coronary artery bypass grafting (CABG), 377 adjuncts in, 106 everting suture technique, 377 antegrade cardioplegia cannula, 103 mechanical valve prosthesis fixation, 376 bretschneider, 104 minimally invasive (MIS) approaches, 377 cardioplegia perfusion, 105 native valve endocarditis (NVE), 377 circumflex artery grafting, 104 non-everting suture technique, 377 cold blood cardioplegia, 105 prophylactic intravenous antibiotic therapy, 376 coronary sinus pressure, 104 prosthetic valve endocarditis (PVE), 377 crystalloid/blood based solutions, 104 robotic approach, 382 crystalloid cardioplegic solution, 104 Sondergaard’s groove, 376 HTK, 104 sternotomy, 382 intermittent warm blood cardioplegia, 105, 106 transesophageal echocardiogram, 377 ischemia/reperfusion, 102 transcatheter interventions, 378 principal aims of, 102 transesophageal echocardiography (TEE), 382 retrograde cannulas, 103 transthoracic echocardiography (TTE), 382 retrograde infusions, 104 valve selection, 375, 376 routes of, 103 valve-in-valve replacement, 378 STH-1, 104 Mitral valve procedures, 583 warm blood intermittent cardioplegia, 105 Mitral valve replacement (MVR), 777 hypothermia, 793 M-mode echo, 27 hypothermic cardioplegia, 102 Moderate pulmonary valve stenosis or dysplasia, 919–921 normothermic methods, 103 Modified Blalock-Taussig shunt, 815, 944 on-pump beating heart technique, 792 Modified Forrester classification, 602 in pediatric myocardium, 794–795 Modified one patch technique, 952 pediatric vs. adult hearts, 791–792 Modified ultrafiltration (MUF), 790 TECAB, 178 Monoclonal antibodies, 657 Myocardial viability Morphologic right septal approach, 909 baseline 2D echocardiography, 56 Mosaic Mitral valve, 301 CMR’s, 58, 60 Cinch® II implant system, 303 dobutamine stress echocardiography, 57 durability, 303, 304 ECG, 56 hemodynamic performance, 303 imaging modalities for, 56 Index 1079

ischemic cardiomyopathy, 60, 61 contraindications of, 161 LV dysfunction, 60 controversies, 163 myocardial contrast echocardiography, 60 embolization and stroke risk, 157 PET imaging, 57 evolution, 157, 158 SPECT imaging, 57 exposure and grafting, setup for, 159 graft failure, 162 graft patency and long-term survival, 157 N incomplete revascularization, 157 Native valve endocarditis (NVE), 377 indications, 160 Natriuretic peptides, 598, 758 intraoperative heparin, 163 Near infrared spectroscopy (NIRS), 95 outcomes, 161, 162 Negative-pressure wound therapy (NPWT), 767, 772 prevent hemodynamic instability, 160 NeoChord DS1000, 457 ROOBY trial, 162 Neodymium:YAG, 262 technique, 158–160 Neonatal intensive care unit (NICU), 866 vascular bulldog clamp, 160 Neurological complications, 725 On-pump and off-pump redo coronary artery bypass grafting, 189 Nicks technique, 348 On-pump beating heart technique, 792 Nipple-cut, 434 Open atrial septectomy, 817 Non-cytomegalovirus (CMV) infections, 668 Open Pivot Mitral Valve, 375 No peek technique, 621 OPEN PLUS® Hybrid stent-graft prosthesis, 547 Non ST elevation acute coronary syndromes (NSTEACS), 4 Open surgical duct ligation, 867 Non-viable myocardium, 55, 59 OPTIME-CHF trial, 602 Nordic Aortic Valve Intervention Trial (NOTION), 441 Optitorque Tiger, 5 Nordic aprotinin patient registry (NAPaR), 66 Organ allocation systems, 640, 641 Norepinephrine (noradrenaline), 74 Organ Care System (OCS™ HEART), 642 Normal physiologic postnatal closure, 865–866 Orthotopic cardiac transplantation, 77 Normothermia, 103 Outflow tract obstruction, 901 Normothermic blood cardioplegia, 793 Oxygen saturation, 95 Nunez technique, 348 Nunn and classical two-patch techniques, 959 Nunn repair P annuloplasty effect, 955 Palliative aorto-pulmonary shunts, 918 atrial septation, 956, 957 Palliative atrial switch operation, 817 autologous pericardium/CardioCel® membrane, 956 Palmaz XL , 451 AV valve competence, 957 PaPa (Medtronic®), 5 AV valve leaflet coaptation, 954 Papillary fibroelastoma, 675–676 AV valve replacement, 958 Papillary myxomas, 681 competence testing, 956 Paraplegia, 880 ligation, 954 Parietal pericardium, 703 line of septation, 954 PAS-Port proximal anastomotic connector, 222, 223 LVOT obstruction, 958 Patch aortoplasty, 880 mechanism of incompetence, 956 Patent ductus arteriosus (PDA), 856, 963 modified DeVega annuloplasty, 957 anatomy, 865 modified ultrafiltration, 954 closure post cardiopulmonary bypass intraoperative transesophageal blunt and sharp dissection, 867 imaging, 957, 958 catheter based therapies, 867 Rastelli A leaflet arrangement, 955 ligation, 867 small interatrial communication, 957 long-term results, 867 superior suture placement, 955 narrow aortic isthmus, 867 suture placement en face, 955 NICU, 866 ventricular portion closure, 955 right angle type dissection, 867 zone of apposition, 956 VATS approach, 867 embryology, 865 normal physiologic postnatal closure, 865–866 O patency with pulmonary overcirculation, 866 Obstructive coronary artery disease, 282 patency without overcirculation, 866 Obstructive hypertrophic cardiomyopathy, 746 Patent foramen ovale (PFO), 898 Off-pump closed valvotomy, 888 Patient blood management (PBM), 63, 64, 67 Off-pump coronary artery bypass grafting (OPCAB) grafting, 168, Pediatric heart transplantation 219, 693 allograft rejection aggressive antiplatelet therapy, 163 acute cellular rejection, 1005 anastomoses and suboptimal graft patency, 162 antibody-mediated rejection (AMR), 1005, 1006 anesthesia, 160 outcomes, 1006 atherosclerotic ascending aorta, 157 treatment, 1006 compare with on-pump CABG, 162 anatomic abnormalities, 1004 1080 Index

Pediatric heart transplantation (cont.) constrictive pericarditis, 707, 711, 712 cardiomyopathy CT, 48 anthracycline-induced cardiomyopathy, 1001 etiology, 704 dilated cardiomyopathy, 1001 intrathoracic and intracardiac pressures, 711 hypertrophic cardiomyopathy, 1002 mechanical function, 703 restrictive cardiomyopathy, 1002 MRI, 48 congenital heart disease parietal pericardium, 703 cardiac tumors, 1003 pericardial effusion, 706, 707, 711 indications, 1003 pericardiectomy single ventricle pathway, 1002 cardiopulmonary bypass, 713 two-ventricle pathway, 1002 complete pericardiectomy, 713 coronary allograft vasculopathy, 1006 early and late outcomes, 714 future aspects, 1007 indications, 713 historical roots, 1001 lateral approach, 713 immunosuppression left anterior thoracotomy, 713 maintenance, 1005 median sternotomy, 713 pre-transplant, 1005 with radiation induced constrictive pericarditis, 714 implantation process, 1005 restrictive cardiomyopathy, 714 infection, 1006 technique, 713 malignancy, 1006 tricuspid valve regurgitation, 714 pediatric heart donation, 1004 Waffle procedure, 713 recipient diagnosis by age, 1001, 1002 risk factors, 707 recipient evaluation syndromes, 704 anatomic consideration, 1004 visceral pericardium, 703 contraindications, 1003 Pericardial effusion, 706, 707, 711 protocol, 1003 Pericardiectomy pulmonary vascular resistance, 1004 cardiopulmonary bypass, 713 retransplantation, 1007 complete pericardiectomy, 713 sensitized patients, 1004 definition, 705 survival, 1007 early and late outcomes, 714 Pediatric Interagency Registry for Mechanical Circulatory Support indications, 713 (Pedimacs), 808–811 lateral approach, 713 Pediatric mechanical circulatory assist devices (MCS) left anterior thoracotomy, 713 adverse events, 811 median sternotomy, 713 complications, 808 with radiation induced constrictive pericarditis, 714 devices and patients, 808 restrictive cardiomyopathy, 714 Jostra Rotaflow centrifugal pump, 807 step-by-step, 708–710 long term, 808 technique, 713 outcomes, 807, 808 tricuspid valve regurgitation, 714 Pedimacs, 808 Waffle procedure, 713 short-term, 807 Perimembranous ventricular septal defects, 850 survival post-VAD implantation, 808 PERIMOUNT Plus Mitral™ valve, 301–303 Tandem Heart, 807 Peripheral pulmonary stenosis, 886 Thoratec PediMag, 807 PETTICOAT technique, 499 Penetrating aortic injury, 700 Phospholipid Reduction Treatment (PRT), 301 Penetrating aortic ulcer (PAU) Placement of Aortic Transcatheter Valves (PARTNER) trial, 581 clinical features, 509 Plasmids, 271 diagnosis, 510 Platelet factor 4 (PF4), 110 endovascular repair, 511, 512 Plication, 248 epidemiology, 508, 509 Pneumothorax, 86 medical management, 510 Polyclonal antibodies, 657 open surgical repair, 511 Polyethylene glycol polymers (PEG), 123 pathophysiology, 507, 508 Pooled red blood cells (PRBCs), 66 Penetrating cardiac injury, 699 Porcelain ascending aorta (PA) PERCEVAL S™ valve system, 426, 427 atheromatous aortic disease, 582 Percu-Pro device, 457 and cardiac surgery, 583, 584 Percutaneous coronary intervention (PCI), 4, 15, 167, 181, 185, 193, and cardiovascular risk, 583 219, 261, 269 classification, 581 Percutaneous device closure, 861 definition, 579, 580 Percutaneous myocardial laser revascularization (PMR), 265 diagnosis, 580, 581 Percutaneous transluminal coronary rotational atherectomy (PTCRA), morbidity and mortality, 579 257 non-atheromatous aortic disease, 582 Periareolar incision, 434 pathophysiological mechanisms, 582 Pericardial constriction, 707 patient selection, 581 Pericardial diseases, 35–36 prevalence, 579, 583 cardiac catheterization findings, 712 TAVR procedures, 581, 584 chronic relapsing or recurrent pericarditis, 704, 706 Porcine valves, 375 Index 1081

Port proximal anastomosis system, 222–224 Pre-implant immunotherapy, 1005 Positive end-expiratory pressure (PEEP), 171, 764 Pre-transplant pulmonary hypertension, 667 Positron emission tomography (PET), 56, 57 PREVENT IV trial, 140 Post-infarction ventricular aneurysms Primary cardiac lymphomas (PCL), 685–687 clinical features, 245 Primary cardiac sarcomas false aneurysms, 244 angiosarcoma, 682, 683 incidence, 244 leiomyosarcomas, 684 investigations rhabdomyosarcomas, 685 chest X-ray finding, 245 undifferentiated pleomorphic sarcomas, 684 CT scanning, 246, 247 Primary graft dysfunction (PGD), 626, 642, 666 electrocardiogram, 245 Primary graft failure (PGF), 666–667 left ventriculography, 246 Primembranous ventricular septal defect, 856–860 MRI, 247 Progressive hypertrophy, 830 TTE, 245, 246 Proliferation signal inhibitors (PSI), 659 management Prophylactic fenoldopam, 758 pseudoaneurysm, 247 Prophylactic intravenous antibiotic therapy, 376 transcatheter device closure, 247 Prophylactic subcutaneous heparin, 764 pathophysiology, 244–245 Prospective Comparison of ARNI with ACEI to Determine Impact on pseudoaneurysm, 244 Global Mortality and Morbidity in Heart Failure true aneurysm, 244 (PARADIGM-HF) trial, 598 Cooley technique, 248–249 Prostaglandin E1 (PGE1), 900, 933 Dor technique, 249, 250 Prosthetic valve endocarditis (PVE), 377 Jatene technique, 249 annual incidence, 405 McCarthy technique, 250 antibiotic management, 407 outcomes of, 250 of aortic graft, 412 plication, 248 aortic perivalvular abscess/fistula, 408, 410, 411 Stoney technique, 249 aortic prosthetic valve infective endocarditis, 409 surgical indications, 248 aortic root abscess, 411 Post-infarction ventricular septal defect (VSD) aortic valve removal, 408 AMI, 229 bovine pericardial bioprosthesis, 405, 406 GUSTO-I trial findings, 229 culture and pathological examination, 408 indications of heart transplantation, 231 definition, 405 postoperative results and prognosis, 233 early PVE, 405 risk factors and operative mortality, 230 etiologic agents, 405 surgical techniques homograft vs. conventional mechanical/biological prostheses, apical septal defect, 232 408 concomitant procedures, 233 imaging, 406 double caval venous drainage, 231 in-hospital mortality, 407 medium/large septal defects, 233 late PVE, 405 necrotic myocardial tissue, 231 Manougian technique, 408 small ventricular septal defect, 232 mitral and tricuspid prosthetic valve endocarditis, 411 therapeutic strategies and options, 231 operative mortality, 407 TTE, 229 prophylactic antibiotics, 406 Postoperative bleeding redo sternotomy, 407 intraoperative approaches, 766, 767 reoperative aortic valve replacement (AVR), 408 medical management, 764 surgical management, 406, 407 acidosis, 764 Kaplan-Meier Curves, 407 antifibrinolytic agents, 765 surgical mortality, 407 coagulation factor deficiency/dysfunction, 764 surgical/transcatheter approaches, 411 crystalloid resuscitation, 764 transcatheter aortic valve replacement (TAVR), 411 desmopressin, 764 Protek Duo cannula, 635 hypothermia, 764 Prothrombin complex concentrate (PCC), 765 massive transfusion protocols, 765 Pseudoaneurysms, 244, 245 positive end-expiratory pressure, 764 PTE, see Pulmonary thromboendarterectomy prothrombin complex concentrate, 765 Pulmonary arterial hypertension (PAH), 869 recombinant factor VII (rFVIIa), 765 Pulmonary arteries (PAs), 892 transfusion, 765 Pulmonary artery (PA) sling restrictive hemoglobin transfusion, 765 associated intracardiac anomalies, 987 thrombocytopenia/suspected/known platelet dysfunction, 764 clinical presentation, 987 thromboelastography, 764 follow-up, 986 TXA and epsilon aminocaproic acid, 765 history, 986 unbalanced red cell transfusions, 765 operative techniques re-exploration, 766 left pulmonary artery, 987, 988 reoperation versus transfusion, 766 no early deaths/complications, 988 surgical methods, 765 pre- and postoperative anatomic findings, 987 Post-transplant lymphoproliferative disorder (PTLD), 642, 669 slide tracheoplasty, 988, 989 Potts shunt, 815 with tracheal resection, 987 1082 Index

Pulmonary artery banding (PAB), 861 supravalvar, 885 complications, 814 valvar, 885 reduction of pulmonary overflow, 813 diagnosis, 886, 887 retraining of left ventricle, 814 indications, 887 technique, 814 management, 887 Pulmonary artery catheters (PACs), 9 natural history, 886 Pulmonary artery systolic pressure (PASP), 34 pathophysiology, 886 Pulmonary atresia with intact ventricular septum (PAIVS) postoperative management, 888 aetiology, 941 presentation and clinical manifestations, 886 anatomy surgical treatment coronary abnormalities, 942–943 off-pump closed valvotomy, 888 imperforate pulmonary valve membrane, 941 valvotomy/valvectomy, 887–888 membranous and muscular atresia, 942 Pulmonary vascular resistance (PVR), 640, 851, 870, 893, 963, 1004 right ventricle and tricuspid valve, 941 Pulmonary vasculature, 723 clinical presentation, 943 Pulmonary vein isolation (PVI), 729 outcomes, 946, 947 Pulsatile Glenn shunt, 909 pathophysiology, 943 Pulsed-wave (PW) Doppler echocardiography, 24, 25 stages of surgery Pulsus paradoxus, 710 mild right ventricular hypoplasia, 944, 945 Pump-assisted cardioplegia method, 793 moderate right ventricular hypoplasia, 945–946 Pump flow/oxygen delivery, 786 neonatal management, 943–944 Pumpless extracorporeal lung assist (pECLA), 798 neonatal period, 944 Pump suckers, 788 severe right ventricular hypoplasia, 946, 947 Purkinje fibres, 81 surgical management, 943 Pulmonary bifurcation, 894 Pulmonary capillary wedge pressure (PCWP), 10–12 Q Pulmonary endarterectomy (PEA), see Pulmonary Quattro valve, 375 thromboendarterectomy Pulmonary flap technique, 872 Pulmonary occlusive disease, 722 R Pulmonary regurgitation (PR), 922 RAAS inhibitors, 598 Pulmonary stenosis, 908 Rabbit anti-thymocyte globulin (rATG), 642 Pulmonary thromboendarterectomy (PTE) Radial artery (RA), 132, 133, 210 alveolo-respiratory, 718 Radial artery harvesting

cardiopulmonary bypass, 718 CO2 insufflation, 139 circulatory arrest, 719 complications, 139 clinical results evidence, 139–141 complications, 725 patient selection, 141 mortality rate, 725 surgical technique neurological complications, 725 endoscopic radial artery harvesting, 141–144 post-operative complications, 725 endoscopic saphenous vein harvesting, 144–147 pulmonary pressures and resistance, 724 surgical equipment, 141 hemodynamic goal, 718 Randomized Aldactone Evaluation Study (RALES), 601 indications, 718 Randomized On/Off Bypass (ROOBY) trial, 163 IVC filter, 718 Rapid-deployment sutureless valves, 347 left pulmonary artery, 721 Rare vascular ring operation principles, 718 circumflex aorta, 988 postoperative care, 722 vascular compression syndrome, 990 prophylactic goal, 718 Rastelli A leaflet arrangement, 955 PVR level, 718 Reactive cardiac masses and pseudotumors right pulmonary artery calcifying amorphous pseudotumor, 675 cardiopulmonary bypass, 719 ectopias, 675 closure, 721 germ cell tumors of the heart, 675 dissection, 720 mesothelial pseudotumors, 675 exposure, 719 mural thrombi, 674 right pulmonary artery (PA), 720 Recombinant factor VII (rFVIIa), 765 site of action, 724 RECOVER RIGHT study, 633 surgical instruments, 722, 723 Redo coronary artery bypass grafting (CABG) surgical specimen, 722 aggressive PCI, 185 true endarterectomy, 719 atheromatous disease, 185 UCSD CTEPH treatment algorithm, 723, 724 cardioprotection, 187 UCSD intra-operative classification, 722, 723 challenges in, 185 Pulmonary valve stenosis grafting pattern and selection of conduits, 188 anatomy and spectrum of lesions heart, ascending aorta and grafts, 187 isolated subvalvar, 885 indications, 186 peripheral, 886 indications for, 186 Index 1083

median sternotomy, 186, 187 commissural leaflets, 390 off-pump vs. on-pump redo CABG, 190 commissurotomy, 391, 394 PCI vs CABG, 189, 190 exposure approach, 392, 393 posterolateral thoracotomy approach, 187 intraoperative transesophageal echocardiography, 391 status of, 186 leaflet augmentation and reconstruction, 394, 395 surgical outcomes, 188, 189 leaflet peeling, 394 transabdominal approach, 187 leaflet problems, 391 Redo-minimally invasive CABG, 168 mitral annuloplasty, 395 REDUCE FMR study, 459 mitral annulus, 390 Rejection operation and surgical techniques, 395 antibody mediated rejection, 661 papillotomy, 394 EMBx concordance rate, 660 pathological changes, 389 non-invasive methods, 660 physiologic function, 391 Relay thoracic stent-graft, 500 postoperative echocardiography, 395 Remodeling process, 598 restrictive/obstructive chords resection, 394 Remote ischemic preconditioning, 758 subvalvular apparatus, 390 Renal protective strategies systematic valve analysis, 393, 394 intraoperative strategies treatment, 390 anemia correction, 757 tricuspid regurgitation, 392 balanced crystalloid solutions, 758 valvular atrial fibrillation, 392 blood glucose control, 757 Right aortic arch, 981, 984–986 blood management program, 758 Right axillary incision approach, 844 dexmedetomidine, 759 Right coronary artery (RCA), 6, 7 levosimendan, 758 Right coronary artery (RCA) lesion, 9 mini-CPB and zero-balanced ultrafiltration, 758 Right gastroepiploic artery (RGEA), 131, 133, 210 natriuretic peptide, 758 Right gastro-omental artery (RGEA), 133 prophylactic fenoldopam, 758 Right heart catheterization (RHC), 9 remote ischemic preconditioning, 758 Right internal thoracic artery (RITA), 131, 209, 212 sodium bicarbonate, 758 Right pulmonary arter surgical strategies, 758 closure, 721 vasopressin, 759 dissection, 720 volatile anesthesia, 758 Right ventricle (RV), 850 postoperative strategies Right ventricle-to-pulmonary artery shunt (Sano shunt), 816 early use of RRT, 759 Right ventricular dependent , 943 extended daily dialysis, 759 Right ventricular dysfunction, 77 KDIGO bundle, 759 Right ventricular outflow tract obstruction (RVOTO), 899, 917 preoperative strategies Right ventricular outflow tract (RVOT) reconstruction, 447 aspirin, 756 Right ventricular outflow tract (RVOT) stenosis, 917 preoperative erythropoietin (EPO) plus iron, 757 Right ventricular outflow tract stenting, 817 renal function optimization, 756, 757 Right ventricular systolic function, 29 statins, 756 Risk, injury, failure, loss, end-stage kidney disease (RIFLE) criteria, 755 Renin-angiotensin-aldosterone system (RAAS), 598, 601 Rittenhouse-Manougian operation, 348 Reparation á l’étage ventriculaire (REV) procedure, 901 Robotic totally endoscopic coronary artery bypass (TECAB), 175–176 Reperfusion, 102 anesthesia, 176 Reperfusion pulmonary edema, 725 beating heart, 180, 181 Respiratory septal shift, 712 cardiopulmonary bypass, 178 Restrictive cardiomyopathy, 714, 1002 evaluation for, 176 Resynchronization for Ambulatory Heart Failure Trial (RAFT) trial, HCR, 181 608 learning curve, 181 REsynchronization reVErses Remodeling in Systolic left vEntricular myocardial protection, 178 dysfunction (REVERSE) trial, 608 outcomes, 182, 183 Retrograde cardioplegia, 96, 104 port sites and cannulation strategy, 180 Retrograde cerebral perfusion (RCP), 538 surgical procedure, 178, 180 Retrograde delivery cardioplegia, 103 ROOBY trial, 140 Retrograde type A dissection (RTAD), 478 Ross procedure, 366, 376, 834 Retroviruses, 272 hemodynamics, 352 Reverse Potts shunt, 815 history, 351 REVIVE II trial, 602 long-term outcomes, 354 Rhabdomyoma, 676, 677 long-term survival, 352–354 Rhabdomyosarcomas, 685 operative steps, 355 Rheumatic mitral valve repair pulmonary autograft, 355, 356 annulus, 391, 392 rationale, 351, 352 associated aortic valve disease, 392 reoperation, 354 chordae and papillary muscles, 391 technical variations, 354 chordal repair, 394, 395 valve-related complications, 352, 354 chordal splitting, 394 Ross–Konno operation, 348 1084 Index

S Society of Thoracic Surgeon (STS) database, 421 Saccular arch aneurysm, 529, 530 Society of Thoracic Surgeons Predictive Risk of Mortality (STS-­ Sacubitril/valsartan, 597, 598 PROM) score, 438 Sandwich technique, 872 Society of Thoracic Surgeons/American College of Cardiology Sano technique, 925 Transcatheter Valve Therapy Registry (STS/ACC TVT), Saphenous vein (SV), 131, 133, 134 438 Saphenous vein graft (SVG), 132, 152, 207, 215 Sodium bicarbonate, 758 SAPIEN 3 THV, 438 Soft tissue flap transposition, 772 SAPIEN valve, 448 SolarGen TMR Ho:YAG Laser System, 262 SAVE-RITA trial, 134 Sondergaard’s groove, 376 Sealant devices, 423 Sones (Cordis®), 5 Seattle Angina Questionnaire, 263 Spinal collateral arterial network (SCAN), 561 Secondary mitral regurgitation, 752–753 Spinal cord injury Secondary right ventricular hypertrophy, 917 Crawford’s classification, 562 , 4, 85, 616, 624 high-risk patients, 562 Selective antegrade cerebral perfusion (SACP), 96 incidence, 561 Selective cerebral perfusion (SCP), 788 independent predictors, 562, 563 Sendai virus (SeV), 272 monitoring, 564 SENIORS clinical trial, 602 multimodal approach, 561, 562 Senning technique, 912 protective strategies Sensitization, 655 CSF drainage, 563 SENTINEL trial, 443 drugs, 563 Septal hypertrophy, 735 hypothermia, 563 Septal myectomy, 735, 736, 743, 996 preconditioning and postconditioning, 564 SHOCK Trial, 229 spinal cord perfusion pressure (SCPP), 563 Short Form Questionnaire 36, 263 schematic diagram of blood supply, 561, 562 Simplified Renal Index (SRI) score, 756 spinal collateral arterial network (SCAN), 561 Simpson’s biplane method, 27 spinal cord perfusion, 561 Single chamber pacemaker, 84 Sporadic cases, 996 Single endarterectomy, 257 Stage II procedure or bidirectional Glenn (cavo-pulmonary) shunt, 929 Single internal thoracic artery and saphenous vein graft (SITA/SVG), Stage III/total cavo-pulmonary connection, 930 214 Standard cardiopulmonary bypass techniques, 919 Single left internal mammary artery (LIMA), 772 Standard median sternotomy, 158 Single photon emission computerized tomography (SPECT), 17, 56, Stanford classification system, 479, 487, 489 57 Stanford/DeBakey methods, 478, 480 Single shot cardioplegia, 96 Starne’s procedure, 976 Single-patch (McGoon) technique, 998 Starr-Edwards valve, 291 Sinuses of Valsalva ST-elevation myocardial infarction (STEMI), 4 anatomic positioning, 567 Stent graft induced new entry (SINE), 502–503 clinical presentation, 568 Stented bioprosthetic valves diagnostic imaging, 568, 569 composite grafts, 299 embryology, 567 composition and design, 299, 300 epidemiology, 568 Crown PRT™ valve, 299–301 medical management, 569 Magna Ease™ valve, 301, 302 noncoronary (posterior) sinus, 567 Mosaic Mitral valve, 301 pathogenesis, 568 Cinch® II implant system, 303 percutaneous closure, 570 durability, 303, 304 stress reduction, 567 hemodynamic performance, 303 surgical management, 570 physiologic fixation, 303 Skeletonized IMA harvesting, 181 Mosaic Ultra™ valve, 301 Slide tracheoplasty, 988, 989 AOA® tissue treatment, 302 Sliding aortoplasty, 998 durability, 302 Small aortic root hemodynamic performance, 302 aortic homografts, 347 Physiologic Fixation process, 302 aortic root enlargement, 348, 349 PERIMOUNT Plus Mitral™ valve, 301–303 aortic root sizing, 346 stented pericardial xenografts, 299 bioprosthetic values, 346, 347 Trifecta valve, 301 commissures, 346 types, 299, 300 conduction system, 346 Stentless valves, 375 enlargement and replacement procedures, 345 Sternal SSI interleaflet triangle, 346 diagnosis, 771 mechanical values, 346 risk factors, 771 patient-prosthesis mismatch, 345 treatment, 772 pulmonary autograft, 348 Sternal wound infection, 204 three-cusp architecture, 346 clinical presentation, 769 Small molecules, 272 deep infections, 770 Index 1085

DSWI cardiac catheterization, 836 incidence, 771 chest radiograph, 835 risk factors, 772 CT, 836 treatment, 772 electrocardiogram, 835 late infections, 773 MRI, 836 late sternal SSI, 771 transthoracic echocardiogram, 835, 836 negative-pressure wound therapy, 772 localised SVAS, 998 pathogenesis long term survival rate, 836 CoNS, 771 management, 836 gram-negative bacteria, 771 outcomes, 998 P. acnes, 771 pathophysiology, 835, 997 S. aureus, 771 point mutations, 834 preventive measures, 774, 775 surgical management sternal SSI diffuse supravalvar aortic stenosis, 998 diagnosis, 771 general principles, 998 risk factors, 771 localised SVAS, 998 treatment, 772 single-patch (McGoon) technique, 998 sternal SSIs, 769 sliding aortoplasty, 998 superficial infections, 770, 771 three-patch (Brom) technique, 998 treatment outcomes, 773 two-patch repair (inverted Y-shaped) Doty technique, 998 wound revision, 772 Supravalvar pulmonary stenosis, 885 Sternocutaneous fistulas, 773, 774 Surgical aortic valve replacement (SAVR), 579 Sternotomy, 150 Surgical rhythm control Steroid therapy, 669 Cox-Maze (CM) procedure, 728, 729 Steroids, 1005 hybrid approach, 730 STICH trial, 753 left atrial appendage exclusion, 730, 731 St. Jude Medical valve, 291, 292, 294 less extensive lesion sets, 729 Stoney technique, 249 Surgical site infections (SSIs), 769 Strain imaging, 24, 25 Surgiflo®, 124, 125 Stress echo, 37 SURVIVE trial, 602 St. Thomas’ Hospital cardioplegic solutions, 104 Svensson’s modification, 574 St. Thomas’ Hospital solution no 1 (STH-1), 104 Swan-Ganz catheters, 9, 11 Studies of Left Ventricular Dysfunction (SOLVD), 598 Sympathetic nervous system (SNS), 598 Stunned myocardium, 55 SYNTAX score, 8 Subaortic myectomy, 736 Systemic anticoagulation, 617 Subclavian artery, 701 Systemic inflammatory response syndrome (SIRS), 71, 101 Subclavian flap aortoplasty, 880 Systemic-pulmonary shunt, 814 Subcommissural annuloplasty (SCA), 331, 364, 365 Systemic vascular resistance (SVR), 70, 73 Subcutaneous implantable cardioverter defibrillator, 87 Systolic Heart Failure Treatment with If Inhibitor Ivabradine Trial Subcutanous fistula, 773 (SHIFT), 601 Subvalvar aortic stenosis (SAS) anatomy, 994 clinical presentation, 994 T diagnosis, 994 Tadalafil, 76 four-stage aetiology, 993, 994 TandemHeart™ devices, 634, 635 indications, 994 Tei index, 28 outcomes, 996 Temporary mechanical circulatory support (MCS) devices pathophysiology, 994 advantages and disadvantages, 634 surgical management CentriMag™ devices, 635, 636 aortoventriculoplasty procedures, 995–996 device comparison, 633 Konno operation, 996 Impella® devices, 632–634 modified Konno Operation, 995 indications, 632 resection of fibrous shelf, 995 patient selection, 632 Ross-Konno procedure, 996 post-operative management, 636 Subvalvar pulmonary stenosis, 885 selection, 631 Sudden cardiac death (SCD), 738 TandemHeart™ devices, 634, 635 Sudden death, 835 weaning, 636, 637 Supravalvar aortic stenosis (SVAS) Tendyne system, 461 anatomy, 834, 996, 997 Tepid hematic cardioplegia, 103 clinical features, 835 Terminal warm blood cardioplegia, 793 clinical presentation, 997 (TOF) diagnosis, 997 anomalous coronary arteries, 918 general principles, 998 associated lesions, 917 incidence, 834 characteristic morphologic features, 917 indications for surgery, 997 with chromosomal abnormalities, 918 investigations initial approaches, 918 1086 Index

Tetralogy of Fallot (TOF) (cont.) partial cardiopulmonary bypass, 520, 521 long-term cumulative incidence, 921, 922 partial left heart bypass, 521 pathophysiology, 918 cerebrospinal fluid drainage, 519 perioperative management, 921 classification, 516, 518 postoperative outcomes, 921 diagnosis, 516, 517 RVOT stenosis, 917 endovascular treatment, 517 surgical management epidemiology, 516 distal main pulmonary artery, 919 etiology, 515, 516 intraoperative transesophageal echocardiography, 919 incision, 519, 520 mildly dysplastic valve, 919 medical management, 516 mild-moderate pulmonary stenosis, 919 morbidity and mortality, 515 moderate pulmonary valve stenosis or dysplasia, 919–921 open surgical treatment, 517 palliative aorto-pulmonary shunts, 918 preoperative planning and set-up, 519, 520 pulmonary valve, 919 preoperative testing, 519 RVOT muscle bundle resection, 919 prognosis, 516 standard cardiopulmonary bypass techniques, 919 repair technique valve-sparing TOF repair, 919 back-bleeding control, 522, 523 valve-sparing transannular reconstruction, 920, 921 cerebrospinal fluid drainage, 522 VSD closure, 919 Dacron tube graft, 523 TGA with intact ventricular septum (TGA-IVS), 898 distal anastomosis, 522, 524 Thoracic aorta endovascular repair (TEVAR) distal aortic perfusion, 522 advantages, 553 extent I thoracoabdominal aortic aneurysm repair, 523 aortic arch landing zones, 553, 554 extent II thoracoabdominal aortic aneurysm repair, 523 in blunt traumatic aortic injury, 556, 557 extent IV thoracoabdominal aortic aneurysm repair, 524 branched and fenestrated grafts, 555 outcomes, 524, 527 cerebrospinal fluid drainage, 554 proximal anastomosis, 522, 524 for complicated type B dissection, 556 Pruitt catheters, 522 complications trifurcation graft, 522, 523 endoleaks, 557, 558 risk factors, 516 neurologic complications, 557 Thoratec PediMag, 807 retrograde type A dissection (RAAD), 557 Three-dimensional (3D) echocardiography, 25 injury, 557 Three-patch (Brom) technique, 998 and open debranching, 555, 556 Thrombelastography (TEG®), 66 neuromonitoring, 555 Thrombin gelatin matrix, 124, 125 for thoracic descending and thoracic abdominal aneurysm, 555 Thrombocytopenia, 617 for uncomplicated type B dissection, 555, 556 Thromboelastometry (RoTEM®), 66 vascular arterial access, 555 Thromboembolic pulmonary hypertension, 718 Thoracic aortic aneurysms Thulium-Holmium-Chromium:YAG, 262 endovascular therapy, 553 Tigerpaw, 731 TEVAR (see Thoracic endovascular aortic repair (TEVAR)) Tissue adhesives, 121 Thoracic aortic calcification (TAC), 582 Tissue Doppler imaging (TDI), 24 Thoracic endovascular aortic repair (TEVAR), 499, 500, 541 Tissue sealants, 121 Thoracic trauma Topical hemostatic agents, 120, 121 aorta Total arterial revascularization (TAR) blunt aortic injury, 700 arterial coronary grafts penetrating aortic injury, 700 inferior epigastric and ulnar arteries, 212 carotid artery LITA, 208, 209 blunt carotid injury, 701 RA grafts, 209–210 penetrating injury, 700, 701 RGEA, 210 clinical signs, 697 RITA, 209 heart arterial graft patency rates, 213 blunt cardiac injury, 699, 700 clinical results coronary injuries, 699 long-term results, 213–214 penetrating cardiac injury, 699 perioperative results, 213 surgical approach, 699 3 versus 2 arterial grafts, 214–215 initial evaluation, 697, 698 conduit patency rates, 213 innominate artery EVH, 208 blunt innominate artery injury, 700 free right internal thoracic artery graft, 210 penetrating injury, 700 in-situ right internal thoracic artery graft, 209 management algorithm, 698 LITA, 207 subclavian artery, 701 off-pump and anaortic coronary artery bypass grafting, 215 Thoracoabdominal aortic aneurysms (TAAA) radial artery grafts, 211 cardiopulmonary bypass saphenous vein grafts, 208, 215 cannulation techniques, 520, 521 sequential radial artery graft, 211 deep hypothermic circulatory arrest (DHCA), 521, 522 SVG, 207 full cardiopulmonary bypass, 521 total arterial/multiple arterial coronary bypass grafting Index 1087

bilateral internal thoracic artery grafts, 212 multi-centered controlled trials, 264 bilateral radial arteries, 213 myocardial perfusion scans, 264 contraindications and limitations, 215 operative technique, 263 controversies in, 215 perioperative mortality rates ranged, 263 left radial artery graft, 213 PMR, 265 parallel anastomosis., 212 quality of life indices, 264 RCA stenosis, 212 Seattle Angina Questionnaire, 263 Total artificial hearts (TAHs), 808–811 Short Form Questionnaire 36, 263 Transannular patch, 944 sole therapy transmyocardial laser revascularization, 263 Transaortic septal myectomy, 735 spinal cord stimulation, 261 Transcatheter aortic valve implantation (TAVI), 421, 422, 427 stem cell therapy, 261 access routes, 440 thoracotomy incision without, 261 heart team concept, 438 transmural channels, 262 high risk, 440, 441 versus medical management, 264 history of, 437 Transposition of the great arteries (TGA) indications and guideline recommendations, 442 balloon atrial septostomy, 900 intermediate risk, 441 discordant ventriculoarterial connections, 897 in landmark trials, 438, 440 epidemiology, 897 low risk trial, 441, 442 physiology, 897, 898 Medtronic CoreValve Evolut R THV, 439 effective systemic blood flow, 898 permanent pacemaker, 442 with intercirculatory mixing at atrial and great artery levels, prohibitive risk, 440 898, 899 risk assessment, 438 LVOT obstruction, 899 SAPIEN 3 THV, 438, 439 patent foramen ovale, 898 stroke, 442, 443 postnatal circulation, 897 valve thrombosis and antithrombotic management, 443, 444 RVOT obstruction, 899 Transcatheter aortic valve replacement (TAVR), 365, 411, 579, 584 TGA with intact ventricular septum, 898 Transcatheter MV implantation (TMVI), 459, 461 preoperative imaging, 899, 900 Transcatheter pulmonary valve replacement (tPVR) preoperative management, 900 balloon-tipped catheter, 449 prostaglandin E1, 900 beneficial effects, 447 surgical techniques biplane main pulmonary artery angiography, 449 arterial switch operation, 901 clinical indications, 447, 448 atrial switch operation, 900 complications, 453 historical perspective, 900 Edwards SAPIEN transcatheter heart valve, 448–452 long-term follow up, 902 empiric IV antibiotics, 451 outflow tract obstruction, 901 femoral approach, 449 timing of surgery, 900 gradual balloon dilation, 451 Transthoracic echocardiography (TTE), 4, 229, 230, 245 hemodynamic and clinical outcomes, 453 Tricuspid regurgitation (TR) intra-procedural heparin, 449 causes, 415, 416 learning curve, 447 ESC/EACTS guidelines, 415 life-long low-dose aspirin, 453 Secondary/functional TR (FTR), 415 Medtronic MELODY transcatheter pulmonary valve, 448–450 surgical techniques, 417, 419 off-pump RVOT ring placement, 454 clover technique, 417 post-dilatation of stent, 451 concomitant tricuspid valve annuloplasty, 419 pre-stenting, 451 De Vega annuloplasty, 417 pretreatment, 449 double orifice valve technique, 418 procedure, 449 functional TR, 418, 419 quantitative assessment, 448 indications, 416 right heart catheterization, 449 isolated tricuspid valve surgery, 416 simultaneous coronary angiography, 449 leaflet tethering, 417, 418 Transcatheter tricuspid valve therapy, 415 left-sided valve surgery, 416 Transcranial Doppler sonography, 95 perioperative outcomes, 419 Transesophageal echocardiography (TEE), 35, 38, 39, 103, 153, prosthetic tricuspid ring annuloplasty, 417 170, 853 right atriotomy, 416 Transforming growth factor beta (TGF-β), 273 ring vs. suture (de Vega) annuloplasty, 419 Transient constrictive pericarditis, 707 suture bicuspidization, 417, 418 Transmyocardial laser revascularization (TMR) trans-tricuspid pacemaker, 419 anti-anginal therapy, 263 tricuspid valve annuloplasty, 419

CO2 laser energy, 262 tricuspid valve replacement, 418 devices, 262 (TS) surgery, 416 Duke Activity Status Index, 263 transcatheter tricuspid valve therapy, 415 exercise tolerance testing, 264 Tricuspid stenosis, 416 external counterpulsation, 261 Tricuspid valve assessment, 34 Ho:YAG laser, 263 Tricuspid valve regurgitation (TR), 714 long-term follow-up, 264 Tricuspid valvuloplasty, 910 1088 Index

Trifecta valve, 301 Upper respiratory tract infections, 981 TriGuard device, 443 Urgent coronary revascularization patients, 763 Triple immunosuppressive therapy, 665 Tris-hydroxymethyl aminomethane (THAM), 96 True aneurysm, 244 V Truncus arteriosus (TA) Valvar pulmonary stenosis, 885 classification systems, 892, 893 Valve Academic Research Consortium (VARC), 582 diagnosis, 893 Valve-sparing transannular reconstruction (VSTAR), 920, 921 morphology, 891, 892 Valvular abnormalities, 640 natural history, 893 Valvular heart disease pathophysiology, 893 CT, 47, 48 surgical treatment etiologies, 291 abnormal valve leaflet resection, 894 mechanical prosthetic valves, 296 allografts, 894 anticoagulation complications, 292 arch reconstruction, 894 bileaflet valves, 291 Chimney graft, 893 bioprosthetic valves, 291, 292 common pulmonary trunk, 893 Bjork-Shiley tilting disk valve, 291 infundibulotomy, 894 CarboMedics mechanical valve, 295 intracardiac repair, 893 current North American (AHA/ACC) and European (ESC/ moderate hypothermic cardiopulmonary bypass, 893 EACTS) guidelines, 292, 293 monocusp reconstruction, 894 Harken Soroff Valve, 291 myocardial protection, 893 Hufnagel Valve, 291 outcomes, 894 On-X valve, 294, 295 outlet VSD, 894 Open Pivot line, 295 PA banding, 893 patient preference, 292 patch technique, 894 Ross procedure, 291 post-operative management, 894 Starr-Edwards Valve, 291 pulmonary bifurcation, 894 St. Jude Medical valve, 291, 292, 294 pulmonary trunk, 894 transcatheter valve, 292 repair techniques, 894 valve selection recommendations, 292 replacement techniques, 894 MRI, 47, 48 truncal valve stenosis or regurgitation, 894 Valvular surgery, 77 Tthoracic aortic calcification, 582 Vascular endothelial growth factor (VEGF), 270 Twelve intrepid system, 461 Vascular endothelial growth factor A (VEGF-A), 273 2D speckle-tracking echo, 27 Vascular rings 2D transesophageal echocardiography (TEE), 28 classification scheme, 981, 982 Two-patch repair (inverted Y-shaped) Doty technique, 998 clinical presentation, 982 Type B aortic dissection (TBAD) diagnostic techniques, 982, 983 complicated type B dissections, 488 indications for operation, 983 DeBakey classification system, 488, 489 Kommerell diverticulum, 981 diagnosis, 489 natural history, 982 etiology, 488 operative techniques imaging modalities, 489 double aortic arch, 983, 984 management right aortic arch, 984–986 ADSORB trial, 490 upper respiratory tract infections, 981 anti-impulse therapy, 490 Vasculogenesis, 273 chronic type B aortic dissection (CTBAD), 492 Vascutek Thoraflex® hybrid stent-graft, 547 complicated ATBAD, 491, 492 Vasoactive peptides, 598 follow-up, 493, 494 Vasopressin, 759 INSTEAD trial, 490 Veno-arterial (VA) extracorporeal membrane oxygenation, 623–627 intramural hematoma (IMH), 492, 493 Venoarterial extra corporeal membrane oxygenation (VA-ECMO), 798 medications, 490 Venous cannulation, 784 penetrating aortic ulcer (PAU), 492, 493 Venous drainage, 94 TEVAR device, 490 Venous switch operation, 909, 910, 912 uncomplicated ATBAD, 491 Venovenous extra corporeal membrane oxygenation (VV-ECMO), uncomplicated TBAD, 490 623–627, 798 physical examination, 489 VENT-AVOID trial, 626 risk factors, 488–490 Ventricular assist devices (VAD), 93, 808–811 Stanford classification system, 487 Ventricular hypertrophy, 794 Stanford classification systems, 489 Ventricular performance, 614 Ventricular remodeling, 749 Ventricular septal defect (VSD), 891, 906 U closure, 910 Undifferentiated pleomorphic sarcomas, 684 DORV Univentricular management, 947 doubly committed VSD, 963 Unresectable cardiac tumors, 1003 non-committed VSD, 962 Index 1089

subaortic VSD, 962 Waterston shunt, 815 subpulmonic VSD, 962 Williams-Beuren syndrome, 835, 996 modified technique, 909 Williams syndrome, 997 Vents sucker, 788 Windkessel effect, 613 Video assisted thoractomy (VATS) approach, 867 WiSE cardiac resynchronization therapy (CRT) system, 87, 88 Viruses, 272 Visceral pericardium, 703 von Willebrand factor (vWF), 764 Y Vouhe operation, 349 Yacoub technique, 315

W Z Waffle procedure, 713 Zenith dissection endovascular system, 499 Warden procedure, 845 Zenith TX2 thoracic aortic endovascular graft, 499